You are on page 1of 129

1|Page

Table of Contents
38. INDIA’S BIOECONOMY REPORT 2022…………25
39. DISTRICT MINERAL FOUNDATION……………..25
Economy
1. Core Sectors……………………………………..…………6 Environment
2. Deep Sea Mining…………………………….……………6
3. India International Bullion Exchange……………7 1. Coastal Clean Up Campaign…………….…………27
4. Seekho aur Kamao Scheme………………………….7 2. Right to a Clean and Healthy environment..27
5. Ujjwal Bharat Ujjwal Bhavishya – Power 3. Living Lands Charter………………..….…………….28
@2047…………….…………….…………………….………8 4. American Bullfrog and Brown Tree Snake....28
6. Financial Inclusion Index…………….………….……9 5. Nagar Van Yojana………………………..…………….29
7. Ocean Thermal Energy Conversion plant 6. The Energy Conservation (Amendment) Bill
(OTEC) …………….…………….………………...…………9 2022…………….…………….…………….……………….29
8. Gold Exchange Traded Funds…………….……….10 7. Ranganathittu Bird Sanctuary……………..….…30
9. ERP (Enterprise Resource Planning) system.10 8. Updated climate pledge…………….…………..….30
10. National Intellectual Property Awareness 9. Bhitarkanika National Park………….…….………31
Mission…………….…………….…………….…..………11 10. Great Barrier Reef…………….……………….………31
11. Digital Lending Norms…………….…………..….…11 11. World Lion Day…………….…………….….………….32
12. Blue Bonds…………….………………………………….12 12. Agasthiyarmalai Elephant Reserve…………….32
13. Atal Pension Yojana…………….…………………….12 13. 11 New Ramsar Sites…………….……………..……33
14. Essential Commodities Act of 1955………….…13 14. Great Indian Bustard…………….…………….…….34
15. Corporate Social Responsibility………………….13 15. Tasmanian Tiger…………….………………….………34
16. Super Vasuki…………….……………………………….14 16. Megalodon…………….…………….…………….…….35
17. Emergency Credit Line Guarantee Scheme...14 17. Snailfish…………….…………….………………..………35
18. Draft Indian Ports Bill, 2022……………………….15 18. Air Quality and Health in Cities………………….36
19. Jal Jeevan Mission-Har Ghar Jal………..……….15 19. Treaty on Ocean Biological Diversity……….…36
20. Rules of Origin…………….…………….………………16 20. Purslane…………….…………….……………….………37
21. Natural Resource Accounting…………………….16 21. Pacific Bluefin Tuna…………….…………….………37
22. Bisphenol-A pilot plant in CSIR-NCL……………17 22. Arth Ganga…………….…………………..…………….38
23. Grameen Udyami Project………………..…………17 23. Battery Waste Management Rules, 2022…..38
24. Foreign Exchange Management (Overseas 24. Seti River…………….…………………...……………….39
Investment) Rules, 2022…………….…….……….18 25. Swachh Sagar, Surakshit Sagar/Clean Coast,
25. Moonlighting…………….…………….………….…….18 Safe Sea Campaign…………….……………..……….39
26. One Nation One Fertiliser Scheme……….…….19 26. Center for Wildlife Rehabilitation and
27. Southern Rice Black-Streaked Dwarf Virus...19 Conservation…………….…………….………………..40
28. Recognition of Prior Learning program……...20
29. Pradhan Mantri Jan-Dhan Yojana………………20
30. Registry to check banking frauds…….…………21 Science and Technology
31. One District One Product (ODOP) Initiative.21
1. AlphaFold & Protein…………….………………..….42
32. ENERGY POVERTY…………….…………….…………22
2. African Swine Fever………………….…………….…42
33. OPIUM MARKET…………….…………….……………23
3. Supercapacitors…………….……………….….………43
34. TECHNICAL RECESSION…………….………….…….23
4. Astrobee…………….……………………….…………….43
35. GLOBAL FINDEX DATABASE 2021……………….24
5. Azaadi SAT…………….…………….……………..…….44
36. NATIONAL APPRENTICESHIP PROMOTION
6. Type Ibn Supernovae(SNe) ……………….………44
SCHEME (NAPS) …………….……………….…………24
7. Exercise Skylight…………….………………….………45
37. OFF-BUDGET BORROWINGS (OBBS) .…………24

2|Page
8. Vasculitis…………….…………….……………..……….45 5. MP’s privileges on arrests……………….…………64
9. Novel Langya Henipavirus…………….……………46 6. Pharmacopoeia Commission for Indian
10. Chronic Fatigue Syndrome…………….…..………47 Medicine & Homoeopathy…………………………65
11. Cornea…………….…………….…………….……………47 7. Foreigners’ Tribunals…………….……….………….66
12. STEVE Phenomenon…………….…………………….48 8. Enhanced Access and Service Excellence…...66
13. Hayabusa-2 probe…………….……………….………48 9. Electricity Amendment Bill,2022………….…….67
14. Lactose Intolerance…………….…………….……….49 10. Criminal Procedure (Identification) Act,
15. Facial Recognition Technology…………….…….49 2022………………………………………………………….67
16. Tomato Flu…………….………………………………….49 11. Fundamental Duties…………….……………..…….68
17. India's first observatory…………….…..………….50 12. Bal Aadhar…………….…………………….…………….68
18. Novae and Supernova…………….…………..…….50 13. Legal Aid Defence Counsel system……….…….69
19. Sickle cell disease…………….…………….…….……51 14. Benami Law of 1988…………….…………………….69
20. CSA6…………….…………….…………….……………….51 15. Floor Test…………….………………….….…………….70
21. Data Sonification…………….……………..………….52 16. Competition (Amendment) Bill, 2022…….….70
22. Dunaliella tertiolecta…………….…………….….…52 17. Constitutional Bench of the Supreme Court.71
23. FIBERISATION…………….…………….……………….53 18. Supreme Court Judges Rules………….………….72
24. BATTERY CERTIFICATION AN QUALITY 19. TELE-LAW SERVICE TO BE MADE FREE OF
CONTROL…………….…………….……………….…….53 COST FOR CITIZENS FROM 2022…………..…….72
25. LUX-ZEPLIN (LZ) DARK MATTER DETECTOR..54 20. PLACES OF WORSHIP ACT………………………….73

Geography International Relations


1. Loktak lake…………….……..……….…………….……55 1. Chabahar Port …………………………….…………….74
2. Odesa Port…………….……………..…….…………….55 2. Mineral Security Partnership……………….…….74
3. Hasdeo Aranya region………………...…………….55 3. Audio Visual Co-production Treaty……….……75
4. Indian Ocean Dipole (IOD) …………………….….56 4. Universal Postal Union…………….….…………….75
5. Nicaragua…………….……………….….…………….…56 5. UNMOGIP…………….…………….…………………….76
6. Rhine River…………….………………………………….57 6. International Labour Organization……….……77
7. Mahanadi River…………….……………….………….57 7. Nagorno-Karabakh dispute………………….…….77
8. Lake Garda…………….…………….………………..….58 8. International Olympiad on Astronomy and
9. Godavari River…………….…………….……..……….58 Astrophysics…………….…………….………………….78
10. Paraguay…………….…………….…………….……..…59 9. Supply Chain Ministerial Forum…………..…….78
11. Tonga Volcano…………….…………….….………….59 10. International Driving Permit…………..……….…79
12. Poyang Lake…………….…………….…………..…….60
13. Anang Tal Lake…………….…………….……….…….60
14. Tigray…………….…………….…………….……….…….61 Society
15. Pileus Cloud…………….…………….………………….61
1. Initiatives to mark 2 years of NEP 2020……..80
16. Puga Valley…………….…………….…………….…….62
2. School Innovation Council……………..………….80
3. Experiential Learning for the 21st Century
Polity Programme…………….…………….…………………..81
4. National Leprosy Eradication Programme….81
1. Electoral Bonds…………….……………………………63 5. SMILE-75 Initiative…………………………………….82
2. Centralized Public Grievances Redress and 6. PMAY-Urban scheme…………….……….………….82
Monitoring System…………….………………….….63 7. Paalan 1000 Campaign…………….………….…….83
3. Waqf Properties…………….………………………….63 8. PEN-PLUS Strategy………………….…………………83
4. Central Vigilance Commission…………..….……64 9. Nikshay Poshan Yojna……………………………….84

3|Page
10. Aashwasan Campaign………………….…………….84 11. Mandala Art…………….…………….……………….104
11. National Forensic Science University…….…..85 12. Pandurang Khankhoje…………….……………….105
12. SEED Scheme…………….…………….………….…….85 13. Mithila Makhana…………….…………….…………105
14. Acculturation…………….…………….…..………….106
15. Yakshagana…………….…………….…………..…….106
Security 16. Garba…………….…………….………………………….106
17. Chola Dynasty…………….…………….…………….107
1. INS Vikrant…………….…………….……………..…….87
18. INDIAN KNOWLEDGE SYSTEMS (IKS)
2. AL NAJAH-IV…………….………………….…………….87
INITIATIVE…………….…………….…………….…….107
3. Exercise VINBAX 2022…………….………………….87
4. MiG 21…………….…………….………………………….88
5. Exercise Pitch Black……………………..…………….88 Miscellaneous
6. Hellfire R9X Missile…………….………………….….88
7. Yuan Wang 5…………….…………….……….……….89 1. ONGC Para Games…………….…………….………108
8. Laser-Guided Anti-Tank Guided Missiles…...89 2. Aridity Anomaly Outlook(AAO) Index……..108
9. Exercise Yudh Abhyas………………….…………….90 3. Central Universities (Amendment)Bill
10. Live-fire exercises…………….…………………….….90 2022………………………………………………………..109
11. Him Drone-a-thon programme………………....90 4. Diammonium Phosphate…………………………109
12. Exercise Vajra Prahar…………….…………….…….91 5. Udyam Portal…………….…………….…….……….110
13. Porcupine Strategy…………….……………….…….91 6. Baumol’s Cost Disease…………….………..…….110
14. Advanced Towed Artillery Gun System……...92 7. World Dairy Summit 2022…………….…….……111
15. Butterfly Mines…………….………………….……….92 8. Khayyam Satellite…………….…………….…….…111
16. Garuda Shield…………….………………..……………93 9. Pyrene…………….…………….…………….……….…111
17. Exercise Udarashakti…………….………….……….93 10. Indian Virtual Herbarium…………………………112
18. Dornier Maritime Surveillance Aircraft………93 11. India ki Udaan Project…………….……………….112
19. F-INSAS System…………….…………….…….………93 12. Operation Yatri Suraksha…………….……..……112
20. TEJAS Fighter Aircraft…………….……….……..….94 13. The Sacrifice Zone…………….……………...……113
21. Zorawar…………….…………….…………….….………95 14. Ongole Cattle Breed……………….…………….…113
22. Exercise VAJRA PRAHAR…………….….………….95 15. The Zaporizhzhia Nuclear Plant……………….114
23. INS Sumedha…………….…………….……….……….96 16. DigiYatra…………….…………….……………….……114
24. Pegasus Spyware…………….……………….……….96 17. Carnelian beads…………….…………….….………114
25. F/A-18 Super Hornet…………….…………….…….96 18. PIN Code…………….…………….……………….……115
26. Light Combat Helicopters…………….………….…97 19. Talc…………….…………….…………….…………..….115
27. THEATRE COMMANDS…………….………….…….98 20. Nereguli Paddy…………….…………….……………116
21. Manthan Platform…………….……………….……116
22. Thucydides Trap…………….………….….…………116
History and Culture 23. Roshni…………….…………….………………..………117
24. Federation Internationale de Football
1. Pingali Venkayya…………….…………………..…….99
Association (FIFA) …………….……………….……117
2. Shrimad Rajchandra Mission…………….…….…99
25. Tilapia Fish…………….…………….……..….……….118
3. Quit India Movement…………….………………….99
26. NIDAAN Portal…………….…………….….…………118
4. Aurobindo Ghose…………….…………..………….100
27. Mudhol hounds…………….……………..….………118
5. Navroz festival…………….…………….…….………101
28. Forever Chemicals…………….……………..………119
6. Traditional Knowledge Digital Library…..….101
29. Aqua Bazar App…………….…………….………..…119
7. Veer Durgadas Rathore…………….……………..102
30. Kerala Savari…………….…………….……………….120
8. Kamakhya Temple…………….…………….………102
31. Puros helmet…………….…………….……..……….120
9. Lord Curzon…………….…………….………….…….102
32. Hunger Stones…………….…………….……….……120
10. Sutr Santati Exhibition…………….……………….103

4|Page
33. Coradia iLint…………….…………………..…………121 39. Zombie Ice…………….…………….………………….122
34. Azadi Quest…………….…………….……….……….121 40. India Council for Research on International
35. Meslin Flour…………….…………….………….…….121 Economic Relations…………….…………….…….123
36. Indian Boiler Act,1923…………….……………….121
37. National Cancer Grid…………….………..……….122
38. Vishnugad Project…………….…………….……….122

5|Page
Economy
1. Core Sectors

Why in News?
India’s eight core sectors’ output growth moderated to 12.7% in June, from 18.1% in May, with all sectors except crude
oil registering an uptick in production.
About

• Eight Core Sectors ➔ Coal, Crude Oil, Natural Gas, Refinery Products, Fertiliser, Steel, Cement, Electricity.
• These comprise 40.27% of the weight of items included in the Index of Industrial Production (IIP).
• The eight core sector industries in decreasing order of their weightage:
• Refinery Products> Electricity> Steel> Coal> Crude Oil> Natural Gas> Cement> Fertilizers.

👉 Index of Industrial Production:

• IIP is an indicator that measures the changes in the volume of production of industrial products during a given
period.
• It is compiled and published monthly by the Central Statistical Organization (CSO), Ministry of Statistics and
Programme Implementation.
• It is a composite indicator that measures the growth rate of industry groups classified under:
• Broad sectors: Mining, Manufacturing, and Electricity.
• Use-based sectors: Basic Goods, Capital Goods, and Intermediate Goods.
• Base Year for IIP is 2011-2012.

2. Deep Sea Mining


Why in News?
The Minister of Science & Technology presented a National Science Award to Indian scientists who conducted the world’s
first locomotive trials of a deep-sea mining system in the central Indian Ocean.
About

• Deep-sea mining is the process of extracting and often excavating mineral deposits from the deep seabed. The deep
seabed is the seabed at ocean depths greater than 200m, and covers about two-thirds of the total seafloor.
• Research suggests deep-sea mining could severely harm marine biodiversity and ecosystems, but India still lacks
the knowledge and means to implement protections.
• Despite this, there is growing interest in the mineral deposits of the seabed. This is said to be due to depleting
terrestrial deposits of metals such as copper, nickel, aluminium, manganese, zinc, lithium and cobalt. Demand for
these metals is increasing to produce technologies like smartphones, wind turbines, solar panels and batteries.

Buoy-based coastal observation and water quality Nowcasting System?

• Developed by ➔ Indian National Centre for Ocean Information Services (INCOIS)


• Purpose ➔ It will make real-time measurements of coastal water quality by measuring 19 parameters including
dissolved oxygen, carbon-di-oxide, nutrients, hydrocarbons and pigments
• Deployed at ➔ Cochin Coast.

What is Nowcasting?

• Nowcasting is weather forecasting on a very short term e.g. 2 hrs.


• In this method, radar and satellite observations of local atmospheric conditions are processed and displayed rapidly
by computers to project weather several hours in advance.
6|Page
3. India International Bullion Exchange(IIBX)
Why in News?
The Prime Minister launched the India International Bullion Exchange(IIBX) at GIFT city in Gandhinagar, Gujarat.
About IIBX

• What is it? ➔ It is India’s first bullion exchange based at Gujarat’s GIFT City (Gujarat International Finance Tec-City).
• Bullion ➔ Bullion refers to physical gold and silver of high purity that is often kept in the form of bars, ingots, or
coins. It can sometimes be considered legal tender and is often held as reserves by central banks or held by
institutional investors.
• Who can trade on IIBX? ➔
o Qualified jewellers will be permitted to import gold through the IIBX. To become qualified jewellers, entities
require a minimum net worth of Rs 25 crore and 90% of the average annual turnover in the last three
financial years through deals in goods categorized as precious metals.
o Apart from qualified jewellers, non-resident Indians and institutions will also be able to participate in the
exchange after registering with the IFSCA.
o Further, jewellers will be able to transact on IIBX as trading members or as clients of a trading member. For
becoming a trading member, a qualified jeweller may establish a branch or a subsidiary in IFSC and apply
to the IFSCA.
• Benefits of Bullion exchange in India ➔
o a) The exchange will be an additional platform for gold imports into the country in addition to banks and
nominated agencies. b) The platform will help with efficient price discovery and provide additional
transparency in transactions. c) It will also give an option to trade gold and silver in the US dollar. d) Since
gift city is a free trade zone, no duty will be paid.
o India is a major player in the global gold market but largely a price taker. The exchange may help increase
India’s role in price discovery in the global market and help India become a major trading hub in Asia.

4. Seekho aur Kamao Scheme


Why in News?
The ‘Seekho Aur Kamao’ scheme has met its target of earmarking 33% of the total trained beneficiaries for female
beneficiaries.

About

• The Seekho aur Kamao (Learn & Earn) Scheme is a 100% Central Sector Scheme for "Skill Development of Minorities",
which is implemented from 12th Five Year Plan.
• The aim of this scheme is to upgrade the skills of minority youth (age group of 14-35 years) and ensure 75%
placements, out of which 50% should be in the organized sector.
• The minimum qualification of trainee should be at least Class V.
• Post placement support of Rs. 2000/- per month is provided to placed trainees for two months as placement
assistance.
• Implementation ➔ This scheme is implemented by the Ministry of Minority Affairs (MoMA) through selected
Project Implementing Agencies (PIAs).
o National Skill Development Corporation (NSDC) has been assigned for the implementation of Seekho aur
Kamao scheme for union territories of J&K and Ladakh.
o The pattern of funding and course will be as per the common norms issued by the Ministry of Skill
Development and Entrepreneurship.
• Beneficiaries ➔ The scheme will be implemented for the benefit of the 6 notified minority communities under
National Commission for Minorities Act 1992.
7|Page
o The six minority communities are Muslims, Christians, Sikhs, Buddhists, Parsis and Jains.
o However, in the States/UTs where some other minority communities notified by respective State/UT
Governments exist, they may also be considered but they will not occupy more than 5% of the total seats.
o The scheme can be taken up anywhere in the country but preference will be given to organizations that
impart training and propose the program for identified minority concentration districts/ towns/ blocks.

5. Ujjwal Bharat Ujjwal Bhavishya – Power @2047


Why in News?
The Prime Minister has participated in the Grand Finale marking the culmination of ‘Ujjwal Bharat Ujjwal Bhavishya –
Power @2047’. He also launched the Revamped Distribution Sector Scheme and National Solar rooftop portal. He also
laid the foundation stone of various green energy projects of NTPC.
About

• It is organized as part of the ongoing ‘Azadi Ka Amrit Mahotsav’.


• Purpose: It showcases the transformation in the power sector achieved in the last eight years. It aims to empower
citizens by improving their awareness and participation in various power-related initiatives, schemes and
programmes of government.

👉 Revamped Based Power Distribution Sector Scheme

• The Scheme aims to improve the operational efficiencies and financial sustainability of DISCOMs/ Power
Departments excluding Private Sector DISCOMs by providing conditional financial assistance for strengthening of
supply infrastructure.
• Objectives of the Scheme ➔
o Reduction of AT&C losses to pan-India levels of 12-15% by 2024-25.
o Reduction of ACS-ARR gap to zero by 2024-25.
o Developing Institutional Capabilities for Modern DISCOMs
o Improvement in the quality, reliability, and affordability of power supply to consumers through a
financially sustainable and operationally efficient Distribution Sector.
• Implementation ➔ Power Finance Corporation(PFC) and Rural Electrification Corporation(REC) have been
nominated as the nodal agencies for the implementation of the scheme.
• Duration of the Scheme ➔ The Scheme would be available till the year 2025-26.

👉 National Solar Rooftop Portal?

• The portal will have the following features:


• Firstly, it will enable online tracking of the process of installation of rooftop solar plants starting from registering the
applications to release of subsidies in residential consumers’ (‘beneficiaries’) bank account after installation and
inspection of the plant.
• Secondly, it will make it easier for a residential consumer to apply and get the solar rooftop solar installed. The
consumers will have the choice to select any vendor registered with the local distribution company, solar modules
of equality and efficiency, solar inverter and other balance of plants and equipment.

👉 Green energy projects launched

• Ramagundam floating solar project in Telangana ➔ It is located in Telangana. It is India’s largest floating solar PV
project with 4.5 lakh ‘Made in India’ solar PV modules.
• Kayamkulam floating solar project in Kerala ➔ It is the second-largest floating solar PV project consisting of 3 lakh
‘Made in India’ solar PV panels floating on water.
• Nokh Solar Project in Rajasthan ➔ It is India’s largest Domestic Content Requirement based Solar project with 1000
MWp at a single location, deploying high-wattage bifacial PV Modules with a tracker system.
8|Page
• Green Hydrogen Mobility Project in Leh ➔ It is a pilot project and aims for five Fuel Cell Buses to be run in and
around Leh. This pilot project would be the first deployment of Fuel Cell Electric Vehicles for public use in India.
• Kawas Green Hydrogen Blending with Natural Gas project in Gujarat ➔ It will be India’s first Green Hydrogen
Blending Project helping in reducing the usage of natural gas.

6. Financial Inclusion Index


Why in News?
The Reserve Bank of India has released the Composite Financial Inclusion Index (FI-Index) for the year ended 31st March
2022.

About

• Released by ➔ Reserve Bank of India.


• It is a comprehensive index incorporating details of banking, investments, insurance, postal as well as the pension
sector in consultation with the government and respective sectoral regulators.
• It was developed by the RBI in 2021, without any ‘base year', and is published in July every year.
• Aim ➔
o To capture the extent of Financial Inclusion across the country.
o The FI-Index is responsive to ease of access, availability and usage of services and quality of services,
consisting of 97 indicators.
• Parameters ➔
o It captures information on various aspects of financial inclusion in a single value ranging between 0 and
100, where 0 represents complete financial exclusion and 100 indicates full financial inclusion.
o It comprises three broad parameters (weights indicated in brackets) viz., Access (35%), Usage (45%), and
Quality (20%) with each of these consisting of various dimensions, which are computed based on a number
of indicators.

👉 Findings

• India’s Financial Inclusion Index has improved to 56.4 from 53.9 in the previous year 2021.
• The improvement has been seen across all its sub-indices (Access, Usage and Equality).

7. Ocean Thermal Energy Conversion plant (OTEC)


Why in News?
Ocean thermal energy conversion plant coming up in Lakshadweep.
About

• Location ➔ It is being established in Kavaratti, the capital of Lakshadweep.


• Ministry ➔ The National Institute of Ocean Technology, an autonomous institute under the Union Ministry of Earth
Sciences (MoES) is establishing an Ocean Thermal Energy Conversion plant.
• Function and capacity ➔
o It has a capacity of 65 kilowatt (kW).
o The plant will power the one lakh litre per day low temperature thermal desalination plant, which converts
seawater into potable water.

👉 Ocean Thermal Energy Conversion

• Ocean thermal energy conversion (OTEC) is a form of energy conversion that makes use of the temperature
differential between the warm surface waters of the oceans, heated by solar radiation, and the deeper cold waters
• OTECs are used to generate power in a conventional heat engine.
9|Page
• Beyond the production of clean power, the OTEC process also provides several useful by-products.
o The delivery of cool water to the surface has been used in air-conditioning systems and in chilled-soil
agriculture (which allows for the cultivation of temperate-zone plants in tropical environments).
o Open-cycle and hybrid processes have been used in seawater desalination, and OTEC infrastructure allows
access to trace elements present in deep-ocean seawater.
o In addition, hydrogen can be extracted from water through electrolysis for use in fuel cells.

8. Gold Exchange Traded Funds


Why in News?
In July 2022, Gold Exchange Traded Funds (ETFs) witnessed a net outflow of Rs 457 crore as investors parked their money
in other asset classes as part of their portfolio rebalancing strategy.
About

• What is it? ➔ Gold ETF, which aims to track the domestic physical gold price, are passive investment instruments
that are based on gold prices and invest in gold bullion.
• Gold ETFs are units representing physical gold which may be in paper or dematerialised form.
• One gold ETF unit is equal to 1 gram of gold and is backed by physical gold of very high purity.
• They combine the flexibility of stock investment and the simplicity of gold investments.
• Benefits ➔
o There is complete transparency on the holdings of an ETF.
o Gold ETFs have much lower expenses as compared to physical gold investments.
o No wealth tax, no security transaction tax, no VAT and no sales tax is levied on ETFs.
o There is no fear of theft as ETFs are safe and secure as units held in Demat Account of the holder.

9. ERP (Enterprise Resource Planning) system


Why in News?
The Ministry of Heavy Industries has launched Automated Online Data Transfer for capturing critical data related to
Domestic Value Addition(DVA) from the PLI applicant’s ERP (Enterprise Resource Planning) system to PLI Auto Portal.
About

• All approved applicants of the PLI Scheme have their own ERP (Enterprise Resource Planning) system.
• ERP is a type of software that organizations use to manage business activities.
• Now the Government has launched the Automated Online Data Transfer system to enable the smooth transfer of
critical data related to Domestic Value Addition(DVA) from the applicant’s existing ERP system to the PLI Auto portal
in a safe environment.
• This facility eliminates that paperwork by bringing in automation. It will also reduce the compliance burden on the
part of the applicants on the one hand, and it will enable faster processing of claims on the other hand.

👉 PLI Scheme for the Auto Sector

• Launched in ➔ 2021.
• Nodal Ministry ➔ Ministry of Heavy Industries.
• Aim ➔ The scheme proposes financial incentives to boost domestic manufacturing of Advanced Automotive
Technology(AAT) products and attract investments in the automotive manufacturing value chain.
• Eligibility ➔ The scheme incentivizes only those eligible AAT products for which a minimum of 50% Domestic Value
Addition (DVA) is achieved.
• This criterion shall reduce imports from outside India, enable deep localization for AAT products in India and enable
Indian Automotive Industry to be an important player in the global supply chain.

10 | P a g e
10. National Intellectual Property Awareness Mission
Why in News?
National Intellectual Property Awareness Mission (NIPAM) has achieved target of imparting Intellectual Property (IP)
awareness and basic training to 1 million students on 31st July 2022, ahead of the deadline which was 15 August 2022.
About

• NIPAM, a flagship program to impart IP awareness and basic training, was launched on 8 Dec 2021 as a part of
“Azadi Ka Amrit Mahotsav'' celebrations.
• The program is being implemented by Intellectual Property Office, the Office of Controller General of Patents,
Designs and Trademarks (CGPDTM), Ministry of Commerce and Industry.
• Aim ➔ 1) To provide awareness on intellectual property and its rights to 1 million students and 2) To inculcate the
spirit of creativity and innovation to students of higher education (classes 8 to 12) and ignite and inspire the students
of college/Universities to innovate and protect their creations.

11. Digital Lending Norms


Why in News?
The Reserve Bank of India (RBI) unveiled a regulatory framework to make digital lending safe in the backdrop of its gaining
traction.
About

• The framework is focused on the digital lending ecosystem comprising RBI Regulated Entities (REs) and the Lending
Service Providers (LSPs).
• LSPs are engaged by Regulated Entities to extend various permissible credit facilitation services.

👉 Norms –

• All loan disbursals and repayments are required to be executed only between the borrower’s bank accounts and
the RE, without any intervention from LSP or any third party.
• Any fees or charges, payable to LSPs in the credit intermediation process shall be paid directly by RE and not by the
borrower.
• A standardized Key Fact Statement (KFS) must be provided to the borrower before executing the loan contract.
• Annual Percentage Rate (APR), an all-inclusive cost of digital loans is required to be disclosed to the borrowers. It
should also form part of KFS.
• It prohibits automatic increases in credit limits without the explicit consent of the borrower.
• It prescribes a cooling-off period during which the borrowers can exit digital loans by paying the principal and the
proportionate APR without any penalty, which shall be provided as part of the loan contract.
• Grievance Redressal ➔ LSPs engaged by Res should have a suitable nodal grievance redressal officer to deal with
FinTech/digital lending-related complaints.
o Such a grievance redressal officer should also deal with complaints against their respective Digital Lending
Applications (DLAs).
• Data Privacy ➔ Data collected by DLAs should be need-based, should have clear audit trails, and should be only
done with the prior explicit consent of the borrower.
o Borrowers could be provided an option to accept or deny the consent for the use of specific data, including
an option to revoke previously granted consent, and an option to delete the data collected from borrowers
by the DLAs/LSPs.
o Any lending sourced through DLAs is required to be reported to Credit Information Companies (CICs) by
REs irrespective of its nature or tenor.

11 | P a g e
o All new digital lending products extended by REs over merchant platforms involving short-term credit or
deferred payments are required to be reported to CICs by the REs.

12. Blue Bonds


Why in News?
SEBI has proposed the concept of blue bonds as a mode of sustainable finance, saying such securities can be utilized for
various blue economy-related activities, including oceanic resource mining and sustainable fishing.
About

• What is it? ➔ A blue bond is a debt instrument issued by governments, development banks or others to raise capital
from investors to finance marine and ocean-based projects that have positive environmental, economic and climate
benefits.
• Like in the case of conventional bonds, investors lend money to a bond issuer, who agrees to repay the interest
every year for the term of the bond plus the capital on a certain day.
• Similarly, in a blue bond, earnings are generated from the investments in sustainable blue economy projects.
Furthermore, the issuance of a blue bond enables investors to fulfil their corporate social responsibilities and
generate benefits for the ocean and humankind.
• The Republic of Seychelles launched the world’s first sovereign blue bond in 2018 raising a total of $15 million to
advance the small island state’s blue economy.
• What are the benefits of Blue Bond? ➔
o a) It offers an opportunity for private sector capital to be mobilized to support the blue economy. b) It helps
in raising awareness about important marine issues while providing much-needed funding to projects. c) It
will catalyze progress towards SDG 14 (Life Below Water).

13. Atal Pension Yojana


Why in News?
Recently, a gazette notification issued by the Ministry of Finance stated that Income tax payers will soon be disallowed
to be a part of the Atal Pension Yojana (APY).
About

• It was launched in 2015.


• Aim ➔ To provide social security for the people working in an unorganised sector as people working in such
sectors mainly belong to a low-income group.
• Eligibility ➔
o Any Indian citizen in the age group of 18-40 years having savings bank account/ post office savings bank
account.
o The co-contribution of the Government of India is available for 5 years & for those who are not covered
by any Statutory Social Security Scheme and are not income tax payers.
• Administrated By ➔ Pension Fund Regulatory and Development Authority (PFRDA).
• Coverage ➔
o The Scheme has been implemented comprehensively across the country covering all states and Union
Territories.
o Atal Pension Yojana (APY) is open to all bank account holders who are not members of any statutory social
security scheme.
• Guaranteed pension ➔
o Under this social security scheme, a subscriber receives a minimum guaranteed pension of 1000 to 5000
per month from the age of 60 years, depending upon his contribution.

12 | P a g e
o The same pension would be paid to the spouse of the subscriber after the demise of subscriber and on
demise of both the subscriber and spouse, the pension wealth as accumulated till age 60 of the subscriber
would be returned back to the nominee.

Recent decision

• Eligibility ➔ Any citizen, who is or has been an income-tax payer, shall not be eligible to join APY.
o the rule will come into effect on October 1, 2022.
o However, they will receive the money accumulated in their respective accounts.
• Income taxpayer is a person who is liable to pay income-tax in accordance with the Income Tax Act, 1961, as
amended from time to time.

14. Essential Commodities Act of 1955


Why in News?
Ministry of Consumer Affairs, Food and Public Distribution has invoked the Essential Commodities Act of 1955 to curb
tur dal prices surging.
About

• Background ➔ To prevent hoarding and black marketing of foodstuffs, the Essential Commodities Act was enacted
in 1955.
• Essential Commodity ➔ There is no specific definition of essential commodities in the Essential Commodities Act,
1955.
o Section 2(A) of the Act states that an “essential commodity” means a commodity specified in the Schedule
of the Act.
• Legal Jurisdiction ➔
o The Act gives powers to the central government to add or remove a commodity in the Schedule.
o The Centre, if it is satisfied that it is necessary to do so in the public interest, can notify an item as essential,
in consultation with state governments.
• Objective ➔ The ECA 1955 is used to curb inflation by allowing the Centre to enable control by state governments
of trade in a wide variety of commodities.
• Impact ➔ By declaring a commodity as essential, the government can control the production, supply, and
distribution of that commodity, and impose a stock limit.

15. Corporate Social Responsibility (CSR)


Why in News?
Coal companies carry out various programmes and activities through their Corporate Social Responsibility (CSR) funds
benefit the peripheral areas around coal mines.
About

• The CSR concept in India is governed by Section 135 of the Companies Act, 2013, Schedule VII of the Act and
Companies (CSR Policy) Rules, 2014.
• The Companies Act encourages companies to spend 2% of their average net profit in the previous three years on
CSR activities.
• It is a management concept whereby companies integrate social and environmental concerns in their business
operations and interactions with their stakeholders.
• Key CSR Areas ➔ Environmental management, Eco-efficiency, Responsible sourcing, Stakeholder engagement,
Labour standards and working conditions, Employee and community relations, Social equity, Gender balance,
Human rights, Good governance, and Anti-corruption measures.

13 | P a g e
• Significance of CSR ➔
o Helps companies to achieve a balance of economic, environmental and social imperatives (“Triple-Bottom-
Line-Approach”);
o Companies can make a valuable contribution to poverty reduction;
o Enhanced access to capital and markets, increased sales and profits.

👉 Scientific Social Responsibility (SSR)

• Government has launched guidelines on Scientific Social Responsibility (SSR), on the lines of Corporate Social
Responsibility (CSR).
• The SSR guidelines seek to:
o Harness the latent potential of the scientific community on voluntary basis,
o Strengthening science and society linkages and
o Making the S & T ecosystem responsive to societal needs.
• The guidelines have provisions to give due weightage to individual SSR activities in performance evaluation of
scientists / knowledge workers.
• The guidelines also include provision of incentivising individual and institutional SSR activities.

16. Super Vasuki


Why in News?
The Indian Railways conducted a test run of the 3.5-km-long freight train, Super Vasuki, with 295 wagons carrying over
27,000 tonnes of coal, as part of the 'Azadi Ka Amrit Mahotsav' celebrations.
About

• The Super Vasuki is a 3.5-km-long freight train run by the South East Central Railway.
• It is the longest and heaviest freight train ever run by the Railways, adding the train takes about 4 minutes to cross
a station.
• It was formed by amalgamating five rakes of goods trains as one unit.
• The Railways plans to use this arrangement (longer freight trains) more frequently, especially to transport coal in
peak demand season to prevent fuel shortages in power stations.
• Significance ➔ Earlier in 2022, coal shortages had pushed the country into a severe power crisis.
o The amount of coal carried by Super Vasuki is enough to fire 3000 MW of power plant for one full day.
o This is three times the capacity of existing railway rakes (90 cars with 100 tonnes in each) that carries about
9,000 tonnes of coal in one journey.

17. Emergency Credit Line Guarantee Scheme


Why in News?
The Union Cabinet has approved the enhancement in the limit of Emergency Credit Line Guarantee Scheme (ECLGS) by
Rs 50,000 crore from Rs. 4.5 Lakh crore to Rs. 5 Lakh crore, with the additional amount being earmarked exclusively for
enterprises in hospitality and related sectors.
About

• Announced as part of the Atma Nirbhar Bharat Package in 2020.


• Objective ➔ to help businesses including MSMEs to meet their operational liabilities and resume businesses in
view of the distress caused by the COVID-19 crisis.
• Providing Member Lending Institutions (MLIs), 100 percent guarantee against any losses suffered by them due to
non-repayment of the ECLGS funding by borrowers.
• Operational domain of the Department of Financial Services (DFS), Ministry of Finance.

14 | P a g e
• Limit ➔
o The overall ceiling initially announced for ECLGS was Rs 3 lakh crore which was subsequently enhanced to
Rs 4.5 lakh crore.
o Loans of about Rs. 3.67 Lakh crore have been sanctioned under ECLGS till 5.8.2022.

18. Draft Indian Ports Bill, 2022


Why in News?
The Government of India has prepared the draft Indian Ports Bill, 2022 to consolidate and amend the laws relating to
ports.
About

• The draft bill seeks to repeal and replace the existing the Indian Ports Act, 1908.
• It seeks to amend the laws relating to ports, for the prevention and containment of pollution at ports, to ensure
compliance with the country’s obligation under the maritime treaties and international instruments to which India
is a party.
• It seeks to empower and establish State Maritime Boards for effective administration, control and management of
non-major ports in India
• It aims to provide adjudicatory mechanisms for redressal of port related disputes and to establish a national council
for fostering structured growth and development of the port sector.
• It will ensure optimum utilisation of the coastline of India, as may be necessary, and to provide for matters ancillary
and incidental thereto, or connected therewith.
• Significance of Ports for India?
o India has a 7,500 km long coastline, 14,500 km of potentially navigable waterways and strategic location
on key international maritime trade routes.
o About 95% of India’s trade by volume and 65% by value is done through maritime transport facilitated by
ports.

19. Jal Jeevan Mission-Har Ghar Jal


Why in News?
Goa and Dadra & Nagar Haveli and Daman & Diu (D&NH and D&D) became the first ‘Har Ghar Jal’ certified State and UT
in the country respectively.
About

• Jal Jeevan Mission is a flagship programme of Government of India launched in 2019.


• Aim ➔ To make provision of potable tap water supply in adequate quantity, of prescribed quality and on regular &
long-term basis to every rural household of the country by 2024.
• The program is implemented by the Government of India in partnership with States/UTs.
• The Jal Jeevan Mission will be based on a community approach to water.
o The mission will include information, education, and communication as key components.
o The mission is meant to create a people's movement for water, making it everyone’s priority.

👉 How are States and UTs certified as Har Ghar Jal?

• The process of certification has been detailed in the Margdarshika of Jal Jeevan Mission:
• First, the field engineer submits a completion certificate regarding the water supply scheme to the Panchayat
during the Gram Sabha meeting.
• The villages confirm through a resolution of the Gram Sabha, that every household is getting a regular supply of
water of prescribed quality and not a single household is left out.

15 | P a g e
• They also confirm that all schools, Anganwadi centres and other public institutions are also getting tap water.

20. Rules of Origin


Why in News?
As per the Central Board of Indirect Taxes and Customs (CBIC), in case of conflict between Customs Rules, 2020
(CAROTAR) and Rules of Origin (under Free Trade Agreement), the provision of FTA on ROO will prevail.
About CAROTAR Rules

• CAROTAR, 2020 set guidelines for enforcement of the ‘rules of origin’ for allowing preferential rate on imports under
Free Trade Agreements.
• They supplement the existing operational certification procedures prescribed under different trade agreements.
• They were notified in August, 2020 by the Ministry of Finance.
• Provisions ➔
o An importer is required to do due diligence before importing the goods to ensure that they meet the
prescribed originating criteria.
o An importer will have to enter certain origin related information in the Bill of Entry, as available in the
Certificate of Origin.
o Importers will have to ensure that imported goods meet the prescribed ‘rules of origin’ provisions for
availing concessional rate of customs duty under Free Trade Agreements (FTAs).
o Importers have to prove that imported products have undergone value addition of at least 35% in the
countries of origin.
o Earlier, merely a country of origin certificate, issued by a notified agency in the country of export was
sufficient to avail the benefits of FTAs.
o This was exploited in many cases, i.e., the FTA partner countries have been claiming to have produced the
goods in question without having the necessary technological capacity for the required value addition.

21. Natural Resource Accounting


Why in News?
The Government Accounting Standards advisory board(GASAB) has come up with a Natural Resource Accounting(NRA)
for tracking and maintaining the natural resources of India.
About Natural Resource Accounting

• GASAB had prepared a concept paper on Natural Resource Accounting (NRA) envisaging short, medium and long-
term goals starting from 2020.
• The first goal was the preparation of asset accounts on mineral and energy resources, more importantly of the non-
renewable and finite resources which would ultimately aid in building resilience to environmental issues like climate
change.
• To date, all 28 States and the Union Territory of Jammu and Kashmir have prepared the Asset Accounts for 2020-
21. The Asset Accounts captured details of 34 major minerals, 58 minor minerals and all four fossil fuels.
• The formats of asset accounts have been designed in keeping with the prescriptions of the System of Environmental-
Economic Accounting central framework(SEEA).
• Note: The UN adopted the System of Economic and Environmental Accounting(SEEA) framework in 2012. It is an
internationally accepted framework for NRA.

👉 Government Accounting Standards Advisory Board(GASAB)

• GASAB was established in the year 2002 by the Comptroller and Auditor General of India (C&AG) with the assistance
of the Ministry of Finance.

16 | P a g e
• Purpose ➔ To formulate and improve standards of government accounting and financial reporting and enhance
accountability mechanisms.
• Chairperson ➔ Deputy Comptroller and Auditor General (Government Accounts).
• Members ➔ The Board has high-level representation from the important accounting heads in Government, Ministry
of Finance, Department of Post, Finance Secretaries of states, RBI and heads of premier accounting & research
organizations.

22. Bisphenol-A pilot plant in CSIR-NCL


Why in News?
The Union Minister inaugurated a Bisphenol A(BPA) pilot plant by setup by the Council of Scientific and Industrial
Research-National Chemical Laboratory(CSIR-NCL) and also launched India’s first truly indigenously developed Hydrogen
Fuel Cell Bus.
About

• Bisphenol A(BPA) is a chemical produced in large quantities for use primarily in the production of polycarbonate
plastics, epoxy resins and other engineering plastics.
• The entire global market for Bisphenol-A is projected to reach 7.1 Million Tons by 2027, growing at a CAGR of 2%
over the analysis period 2020-2027.
• The entire estimated annual demand of 1,35,000 tons in India is imported today. Hence, this pilot plant will enable
import substitution of this important raw material and help in India’s Atmanirbhar initiative.

👉 Hydrogen Fuel Cell Bus

• Developed by ➔ Council of Scientific and Industrial Research (CSIR) and private firm KPIT Limited.
• Working Process ➔ The fuel cell utilizes Hydrogen and Air to generate electricity to power the bus and the only
effluent from the bus is water, therefore making it possibly the most environmentally friendly mode of
transportation.
• Advantages of fuel cells ➔ Lower Cost, Zero Greenhouse gas emissions, Excellent means to eliminate on-road
emissions.

23. Grameen Udyami Project


Why in News?
National Skill Development Corporation(NSDC) in partnership with Seva Bharti and Yuva Vikas Society has launched the
second phase of Grameen Udyami Project to augment skill training in tribal communities for their inclusive and
sustainable growth.
About

• The project is implemented under Sansadiya Parisankul Yojana.


• Aim ➔ To multiskill India’s tribal youth and impart functional skills to them for enabling livelihoods.
• Coverage ➔ The project is being implemented in six states— Maharashtra, Rajasthan, Chhattisgarh, Madhya
Pradesh, Jharkhand, and Gujarat.
• Funded by ➔ National Skill Development Corporation(NSDC)
• The training under the project will be conducted in the following Job roles: Electrician & Solar PV Installation
Technician Plumbing & Masonry 2-Wheeler Repair & Maintenance IT/ITES with e-Governance and Farm
Mechanization.
• Phases ➔ In the first phase of the project, candidates were mobilized from rural and tribal areas of Maharashtra,
Rajasthan, Chhattisgarh, Madhya Pradesh, and Gujarat.
o The phase-II of the pilot project was launched in Ranchi, Jharkhand.

17 | P a g e
• Significance of the project ➔ The project will help in achieving the following objectives: a) Increase in Rural/Local
Economy, b)Enhance employment opportunities, c) Reduce forced migration due to lack of local opportunities and
d)Conservation of natural resources.

24. Foreign Exchange Management (Overseas Investment) Rules, 2022


Why in News?
The Finance Ministry has released the Foreign Exchange Management (Overseas Investment) Rules, 2022. The rules are
aimed at easing rules for domestic firms that want to invest abroad.
About

• The rules are aimed at easing rules for domestic firms that want to invest abroad.
• Annual Performance Report ➔ Any resident in India acquiring equity capital in a foreign entity or overseas direct
investment(ODI) will have to submit an Annual Performance Report (APR) for each foreign entity, every year by
December 31.
o No such reporting shall be required where a person resident in India is holding less than 10% of the equity
capital without control in the foreign entity and there is no other financial commitment other than equity
capital or a foreign entity is under liquidation.
• Overseas Direct Investment(ODI) by Resident Individual ➔ Any resident individual can make an ODI by way of
investment in equity capital or overseas portfolio investment(OPI) subject to the overall ceiling under the
Liberalized Remittance Scheme (LRS) of the Reserve Bank.
o Currently, the LRS permits $2,50,000 in outward investment by an individual in a year.
• Investment in Financial Firms abroad ➔ Indian companies not in financial services can now directly invest in
financial-services firms abroad, such as brokerages, asset management funds, and credit cards under the automatic
route. Banks and insurance firms have been kept out of this. Earlier, such investment was prohibited.
• Strategic Sector ➔ The new rules introduced the concept of “strategic sector”, which gives the government the
powers to permit overseas investment in excess of the limits prescribed under the rules.
o The strategic sector shall include energy, natural resources and such other sectors as may be decided by
the government from time to time in view of the evolving business requirements.
• Prohibitions for overseas investment ➔ Any Indian resident who has been classified as a wilful defaulter or is under
investigation by the CBI, the ED or the Serious Frauds Investigation Office (SFIO) will have to obtain a no-objection
certificate (NOC) from his or her bank, regulatory body or investigative agency before making any overseas financial
commitment or disinvestment of overseas assets.
o If the lenders, the concerned regulatory body or investigative agency fail to furnish the NOC within 60 days
of receiving an application, it may be presumed that they have no objection to the proposed transaction.

25. Moonlighting
Why in News?
Swiggy has launched a Moonlighting Policy allowing its employees to work on their passion projects outside of their work
(calling it a step towards a “remote-first organization”).

About

• Moonlighting is the act of working at an extra job beyond regular working hours, usually without the knowledge of
the employer.
• Since the side job was mostly at nighttime or on weekends, it was referred to as moonlighting.
• Why do people moonlight? ➔
o Moonlighting is a source of gaining extra profits.

18 | P a g e
o Additionally, working in a different role can allow a person to develop new skills, explore related domains
and connect with more people.
o Many universities abroad consider such side projects as valuable and an applicant who performs in
challenging projects alongside their regular 9-5 jobs thus gets an upper hand over others.
• Arguments against moonlighting? ➔
o Legal but may not be ethical: There is no overarching law which prohibits a person from doing multiple jobs.
However, a person with a similar nature of jobs may spark breach of confidentiality issues.

26. One Nation One Fertiliser Scheme


Why in News?
The Ministry of Chemicals and Fertilisers has announced that it has been decided to implement One Nation One Fertiliser
by introducing a “Single Brand for Fertilisers and Logo” under the fertiliser subsidy scheme named “Pradhanmantri
Bhartiya Janurvarak Pariyojna” (PMBJP).

About

• Under ONOF companies are allowed to display their name, brand, logo and other relevant product information
only on one-third space of their bags.
• On the remaining two-thirds space, the “Bharat” brand and Pradhanmantri Bharatiya Jan Urvarak Pariyojana logo
will have to be shown.
• The single brand name for UREA, Di-Ammonium Phosphate DAP, Muriate of potash (MOP) and Nitrogen Phosphorus
Potassium NPK etc. would be BHARAT UREA, BHARAT DAP, BHARAT MOP and BHARAT NPK etc. respectively for all
Fertiliser Companies, State Trading Entities (STEs) and Fertiliser Marketing Entities (FMEs).
• This scheme applies to both public & private sector companies.
• It will bring about uniformity in fertiliser brands across the country.

27. Southern Rice Black-Streaked Dwarf Virus


Why in News?
Recently, Scientists at Punjab Agricultural University(PAU) have found that the disease that has impacted the paddy crop
in Punjab and Haryana was Southern Rice Black-Streaked Dwarf Virus (SRBSDV).

About

• What is it? ➔ It is a viral disease that has infected the paddy crops in Haryana and Punjab causing “dwarfing” of the
plants.
• Named After ➔ It is named after Southern China where it was first reported in 2001.
• Transmitted by ➔ It is transmitted by white-backed plant hopper(WBPH) in a persistent circulative and propagative
manner.
• The long-distance transmission of this virus may occur through WBPH migrating with typhoons and strong
convection winds.
• Treatment ➔ Since there was no corrective measure for this viral disease, farmers have been advised several
measures which include:
o Firstly, Farmers should regularly monitor the crop for the presence of WBPH and a few plants should be
slightly tilted and tapped 2-3 times at the base at weekly intervals.
o Secondly, if WBPH nymphs/adults are seen floating on water, then insecticides can be sprayed towards the
base of the plants.
o Lastly, farmers are advised to follow the transplanting dates advised by PAU since stunting was observed
to be more in the early transplanted crops. It will not only help in managing the viral disease but also save
water.
19 | P a g e
28. Recognition of Prior Learning (RPL) program
Why in News?
Recently, the Union Minister of Education and Skill Development presented certificates to NDMC workers for completing
the Recognition of Prior Learning (RPL) program in Delhi.

About

• Recognition of Prior Learning (RPL) is a process used to evaluate a person’s existing skill sets, knowledge and
experience gained either by working on-the-job or learning over the years.
• Ministry ➔ The Ministry of Skill Development and Entrepreneurship launched Pradhan Mantri Kaushal Vikas Yojana
in the year 2015.
o Recognition of Prior Learning is a component of the PMKVY Scheme.
o It is being implemented under the SANKALP Programme.
• Implementing agency ➔ The implementing agency for the programme is the National Skill Development
Corporation (NSDC).
• Features of RPL ➔
o No fee is charged from a candidate for participating in the RPL program.
o RPL recognizes the value of learning acquired outside a formal setting and provides a government
certificate for an individual’s skills.
o Every successfully certified candidate receives INR 500.
o Candidates receive exposure to concepts of digital and financial literacy.
o Candidates receive accidental insurance coverage for three years at free of cost.
• Aim ➔
o The objective of this Skill Certification Scheme was to enable a large number of Indian youth to take up
industry-relevant skill training that will help them in securing a better livelihood.
o Under this programme the workers will be skilled in multiple trades across construction, electrical,
plumbing, pottery, handicrafts and more.
o To provide opportunities for reducing inequalities based on privileging certain forms of knowledge over
others.

👉 Pradhan Mantri Kaushal Vikas Yojana (PMKVY)

• Launched in 2015, it is a flagship program of the Ministry of Skill Development and Entrepreneurship (MSDE)
implemented by the National Skill Development Corporation (NSDC).
• Under PMKVY, training and assessment fees are paid completely by the Government. Pay-outs are provided to the
Training Providers (TPs) in alignment with the common norms.
• Funding: The scheme is being implemented at the Centre and the State level with a 50:50 allocation of funds and
targets with more enhanced involvement of District Skill Committees (DSC).

29. Pradhan Mantri Jan-Dhan Yojana(PMJDY)


Why in News?
The novel Fijivirus has caused a disease that had dwarfed some non-basmati and basmati plants in Haryana and Punjab.
About

• It was launched in 2014 to ensure universal access to financial services namely, Banking/ Savings & Deposit
Accounts, Remittance, Credit, Insurance, Pension in an affordable manner through the use of technology.
• Objectives ➔
o Ensure access to financial products & services at an affordable cost.
o Use of technology to lower cost & widen reach.
20 | P a g e
• Basic tenets of the scheme ➔
o Banking the unbanked: Opening of basic savings bank deposit (BSBD) account with minimal paperwork,
relaxed KYC, e-KYC, account opening in camp mode, zero balance & zero charges.
o Securing the unsecured: Issuance of Indigenous Debit cards for cash withdrawals & payments at merchant
locations, with free accident insurance coverage of Rs. 2 lakhs.
o Funding the unfunded: Other financial products like micro-insurance, overdraft for consumption, micro-
pension & micro-credit.
• Six Pillars of the Scheme ➔
o Universal access to banking services: Branch and Banking Correspondants.
o Overdraft Facility: Basic savings bank accounts with overdraft facility of Rs. 10,000/- to every eligible adult.
o Financial Literacy Programme: Promoting savings, use of ATMs, getting ready for credit, availing insurance
and pensions, using basic mobile phones for banking.
o Creation of Credit Guarantee Fund: To provide banks some guarantee against defaults.
o Insurance: Accident cover up to Rs. 1,00,000 and life cover of Rs. 30,000 on account opened between 15
Aug 2014 to 31 January 2015.
o Pension scheme for Unorganized sector.
• Extension of PMJDY with New features ➔ The Government has decided to extend the comprehensive PMJDY
program beyond 2018 with some modifications:
o Focus shift from ‘Every Household’ to Every Unbanked Adult
o RuPay Card Insurance – Free accidental insurance cover on RuPay cards increased from Rs. 1 lakh to Rs. 2
lakh for PMJDY accounts opened after 28.8.2018.
o Enhancement in overdraft facilities – OD limit doubled from Rs 5,000/- to Rs 10,000/-; OD upto Rs 2,000/-
(without conditions).
o Increase in upper age limit for OD from 60 to 65 years.

30. Registry to check banking frauds


Why in News?
Recently, the Reserve Bank of India (RBI) considering setting up a fraud registry to create a database of fraudulent
websites, phones and various methods used by fraudsters.

About

• The registry will create a database of fraudulent websites, phones and various methods used by fraudsters.
• Payment system participants will be provided access to this registry for near-real-time fraud monitoring. The
aggregated fraud data will be published to educate customers on emerging risks.
• Benefits ➔ Improve consumer protection amid cases of digital fraud.

👉 One Nation One Ombudsman

• Launched in 2021 as an integrated consumer grievance redressal mechanism for addressing service deficiencies in
banking, NBFCs and digital payment systems.
• To make the alternate dispute redressal mechanism simpler and more responsive to the customers of regulated
entities.
• Under this,18 lakh complaints were received during 2021-22 and as many as 97.9 per cent of cases were cleared
last financial year.

31. One District One Product (ODOP) Initiative


Why in News?

21 | P a g e
The Union Minister for Commerce and Industry has called for the integration of One District One Product (ODOP)
initiative with Open Network for Digital Commerce (ONDC).

About

• ODOP is an approach adopted under the Pradhan Mantri Formalisation of Micro food processing Enterprises
(PMFME) Scheme.
• It will provide for the framework for value chain development and alignment of support infrastructure of the
PMFME scheme. There may be more than one cluster of ODOP products in one district.
o There may be a cluster of ODOP products consisting of more than one adjacent district in a State.
• The States would identify food products for districts keeping in view the existing clusters and availability of raw
material.
• The ODOP could be a perishable produce based or cereal based or a food item widely produced in an area. E.g.,
mango, potato, pickle, millet-based products, fisheries, poultry, etc.
• Certain other traditional and innovative products including waste to wealth products could be supported under the
Scheme.
• For example, honey, minor forest products in tribal areas, traditional Indian herbal edible items like turmeric, amla,
haldi, etc.
• Significance ➔
o Adopting a cluster approach will help in the development of specific agriproducts in districts having a
comparative advantage.
o It would lead to easing in providing common facilities and other support services.

👉 ONDC

• ONDC will provide a common digital space for buyers and sellers with the goal of democratising e-commerce by
transforming it from a platform-centric paradigm to an open network for buying and selling goods and services,
ONDC can certainly transform India's E-commerce sector.

👉 How will integration of ODOP and ONDC initiative help?

• The integration of ONDC with ODOP would help in further expanding the frontiers of ODOP by bringing buyers and
sellers together on a democratic platform.

32. ENERGY POVERTY


Why in News?
At the G-7 Summit in Germany, the Indian Prime Minister highlighted the issue of energy poverty and the need for equal
energy access for the impoverished global south and the rich global north.

About

• Energy poverty, also referred as fuel poverty, is usually defined in two ways as- Energy Accessibility, i.e., lack of
access to electricity, and Energy Affordability, I.e., household dependence on inefficient and polluting energy
sources such as burning of solid biomass.
• Nearly three billion people are energy-poor people in Latin America, Asia, and Africa alone.

22 | P a g e
• The risk of energy poverty in Europe and other parts have also increased due to rising energy prices.

33. OPIUM MARKET


Why in News?
For the first time, central government has allowed a private company, Bajaj Healthcare, to manufacture concentrated
poppy straw (CPS) alkaloids and active pharmaceutical ingredients (APIs) from unlanced poppy capsules.

About

• CPS is a mechanised system under which entire harvest is cut by machine, transferred to factories for alkaloid
extraction and eliminates diversion of legally produced opium.
• Significance of private players involvement
o Boost domestic production of various alkaloids such as morphine and codeine.
o Bring in modern technology to extract alkaloids.
o Reduce imports of opium.
o Offsetting the declining area under cultivation of poppy in India.
• Opium is a natural substance obtained from poppy seeds and its derivatives.
o Alkaloids obtained from opium are used to make cough syrups, cancer medicines and painmanagement
medicines.
o Only 12 countries including India allow its cultivation legally for medicinal use.
o It is used illicitly for smoking, drinking, or eating as pills.
• Due to potential for illicit trade and risk of addiction, opium poppy cultivation is strictly regulated.
o At present, cultivation and processing of poppy and opium is controlled by provisions of Narcotic Drugs and
Psychotropic Substances (NDPS) Act and Rules.

34. TECHNICAL RECESSION


Why in News?
US is expected to avoid a technical recession. A technical recession is a term used to describe two consecutive quarters
of decline in output.

About

• In case of a nation’s economy, it usually refers to back-to-back contractions in real GDP.


• It is most often caused by a one-off event (say COVID-19 pandemic and lockdowns imposed) and is generally
shorter in duration.
• It is mainly used to capture trend in GDP while a ‘recession’ encompasses more broad-based decline in economic
activity that covers several economic variables including employment, household and corporate incomes etc.

23 | P a g e
35. GLOBAL FINDEX DATABASE 2021
Why in News?
World Bank recently released Global Findex Database 2021: Financial Inclusion, Digital Payments, and Resilience in the
Age of COVID-19.

About

• Global Findex database surveyed how people in 123 economies use formal and informal financial services including
use of cards, mobile phones, and internet to make and receive digital payment throughout 2021.
• India’s Specific Findings India is among seven countries home to half the world’s 1.4 billion adults without access
to formal banking.
• Aadhar system contributed to significant increase in financial inclusion, driving account ownership up to 80% of
adults in 2017 from 35% in 2011.
• Internal fraud and leakage from pension payments dropped by 47% when the country transitioned from cash to
sending payments to biometric smart cards.
• Distance to financial institutions, lack of trust, and lack of need, are the factors for bank account inactivity.

About Financial Inclusion

• It that individuals and businesses have access to useful and affordable financial products & services that meet their
needs – transactions, payments, savings, credit, and insurance – delivered in a responsible and sustainable way.
• It helps in achieving ending extreme poverty and promoting shared prosperity, and it is linked to various United
Nations’ 17 Sustainable Development Goals.

36. NATIONAL APPRENTICESHIP PROMOTION SCHEME (NAPS)


Why in News?
Ministry of Skill Development and Entrepreneurship (MSDE) announced that NAPS will be part of Direct Beneficiary
Transfer (DBT) scheme.

About

• Now, government will directly transfer its contribution to bank accounts of apprentices through National Skill
Development Corporation (NSDC), 25% of the stipend payable up to Rs. 1500/- per month.
• NAPS, launched in 2016, to promote apprenticeship training in country and provide financial support to
establishments undertaking apprenticeship.
• It aims to motivate employers to hire apprentices and aid them in discovering right job roles while maximising their
potential through in-depth skill development.
• NAPS has replaced Apprentice Protsahan Yojna (APY) from 2016.

37. OFF-BUDGET BORROWINGS (OBBS)


Why in News?
Centre eased norms for adjusting states' off-Budget borrowings (O-BBs).

About

• Under the new norms, O-BBs done by states up to 2020-21 may not be adjusted and only those done in 2021-22
can be adjusted over up to 4 years till March 2026.
• Earlier this year, Centre has informed states that O-BBs are to be equated with states’ own debt and incremental O-
BBs raised (2020-21 & 2021-22) would be adjusted out of the borrowing ceiling this year.

24 | P a g e
• O-BBs by a state government refer to loans taken by its entities, special purpose vehicles, etc., which are expected
to eventually be serviced through the state government’s own budget, instead of the cash flows or revenues
generated by the borrowing entity.
• Such borrowings bypass the states’ net borrowing ceiling fixed in a FY by routing loans outside state budget.
• Such borrowings have an impact on the revenue deficit and fiscal deficit and thus surpassed the targets set for fiscal
indicators under the state FRBM Act.
• For 2022-23, ceiling has been set at 3.5% of GSDP, along with an additional 0.5% linked to reforms in power sector.
• Under Article 293 (3), state governments are required to take the Centre’s permission for fresh borrowing, if they
are indebted to the Government of India.
• Reasons for increasing O-BBs by states:
• Constrained revenue growth due to the pandemic induced slowdown and increasing revenue expenditure.
• Erosion of autonomous fiscal space due to GST.

38. INDIA’S BIOECONOMY REPORT 2022


Why in News?
The Biotechnology Industry Research Assistance Council (BIRAC) recently released India’s Bioeconomy Report 2022.

About

• The Special Biotech Ignition Grant Call for Northeast Region (BIG-NER) under BIRAC was also launched and financial
support of Rs 50 lakhs each to 25 start-ups and entrepreneurs from Northeast Region to develop biotech solutions
was announced.

What is Bio-Economics?

• According to United Nations Food and Agriculture Organisation (FAO), bioeconomy is the production, utilization,
conservation, and regeneration of biological resources, including related knowledge, S&T, and innovation, to
provide
• sustainable solutions (information, products, processes, and services) within and across all relevant sectors and
enable a transformation to a sustainable economy.

Some of the major areas of Bio-economics include-

• Food systems occupy the largest niche in the bioeconomy which includes sustainable agriculture, sustainable fishing,
forestry, and aquaculture.
• Bio-based products include bioplastics, biodegradable clothing, and other products with eco-design.
• Bioenergy improves the security of energy supply and reduces energy dependence like biomass.

About BIRAC

• BIRAC is a not-for-profit section 8, Schedule B, Public Sector Enterprise.


• It was set up by the Department of Biotechnology (DBT)
• It acts as an Interface Agency to strengthen and empower the emerging Biotech enterprise to undertake strategic
research and innovation, addressing nationally relevant product development needs.

39. DISTRICT MINERAL FOUNDATION


Why in News?
Centre has released the guidelines to tighten the criteria for use of DMF fund to ensure they are used for the specified
purpose, that is redevelopment of areas and people affected by mining activities.

About Key guidelines

25 | P a g e
• DMFs will have to prepare a five-year perspective plan to ensure systematic development of the area affected by
mining-related activities using the fund accrued from miners’ contribution as a percentage of royalty.
• DMFs will conduct a baseline survey through universities or renowned organisations or agencies to formulate a
perspective plan.
• Gram sabha or local bodies may aid in preparation of needs assessment reports.
• Based on the findings and gaps as identified through the baseline surveys, the DMF shall prepare a strategy for five
years and the same shall be included in the perspective plan.

About DMF

• Under the Mines and Minerals (Development and Regulation) Amendment Act 2015, the Central Government
instituted the DMF in 2015.
o Under this, DMF is created in each district that has mining.
o The mining companies shall pay DMF 30% of the royalty amount for leases granted before 2015 and 10%
for leases granted through the auction mechanism post-2015.
• The funds so collected are to be managed by non-profit trusts and are to be used for welfare of mining-affected
population, including tribal and forest-dwelling communities.
o Each district has a separate trust.
• It is a special fund that is not tied to any particular scheme or area of work, and it does not lapse at the end of each
financial year.
o The unused funds get accumulated over the years.

Recent amendments to the DMF

• In March 2020, government issued guidelines suggesting that up to 30% of the funds could be used towards
expenditure related to COVID-19.
• According to MMDR Amendment Act 2021, central government may also give direction regarding the composition
and utilization of the fund while state governments continue to prescribe the constitution and functions of the
DMF.
• In 2021 government issued order that no sanction or approval of any expenditure out of the DMF fund shall be
done at the state level by the state government or any state-level agency.

The Mines and Minerals (Development and Regulation) Amendment Act, 2015:

• It amends the Mines and Minerals (Development and Regulation) Act, 1957 as it regulates the mining sector in
India and specifies the requirement for obtaining and granting mining leases for mining operations.
• It provides for the creation of a District Mineral Foundation (DMF by the state government) and a National Mineral
Exploration Trust (NMET by the central government).
• It creates a new category of mining license i.e. the prospecting license-cum-mining lease, which is a two stage-
concession for the purpose of undertaking prospecting operations, followed by mining operations.

Pradhan Mantri Khanij Kshetra Kalyan Yojana (PMKKKY)

• It is meant to provide for the welfare of areas and people affected by mining related operations, using the funds
generated by DMF.
• The scheme ensures mitigating the adverse impacts, during and after mining, on the environment, health and socio-
economics of people in mining districts; and to ensure long-term sustainable livelihoods for the affected people in
mining areas.
• As per this scheme, High priority areas like drinking water, health care, sanitation, education, skill development,
welfare measures, skill development and environment conservation will get at least 60% share of the funds.
• It was launched by Ministry of mines.

26 | P a g e
Environment
1. Coastal Clean Up Campaign

Why in News?
The Union Minister of Earth Sciences has convened a meeting of Union Ministers and Members of Parliament from
coastal states of the country to discuss the ongoing countrywide Coastal Clean Up Campaign launched by the Ministry of
Earth Sciences.
About

• What is it? ➔ Coastal Clean-Up campaign is a 75-day campaign launched by the government to clean up coastal
beaches and raise awareness about “Swachh Sagar, Surakshit Sagar”.
• Aim ➔ To make awareness about reducing marine litter, minimal use of plastics, segregation at source and waste
management.
• The campaign will culminate on “International Coastal Clean-up Day” (17th September 2022).
• Impact ➔ Through this campaign, a mass behavioural change among the masses is intended by raising awareness
about how plastic usage is destroying our marine life.

👉 What is Eco Mitram?

• It is a mobile app launched to spread awareness about the campaign and also for the common people to voluntary
registration for the beach cleaning activity on the 17th of September 2022.

2. Right to a Clean and Healthy environment


Why in News?
UN declares access to clean, healthy environment as universal human right.
About

• Right to live in a clean and healthy environment was not included in the Universal Declaration of Human Rights,
1948.
• Now, United Nations (UN) in a historic resolution has declared that every person on the planet has the right to live
in a clean and healthy environment.
• The declaration has been adopted by over 160 UN member nations, including India.
• This will encourage countries to incorporate the right to a healthy environment in national constitutions and
regional treaties.
• However the declaration is not legally binding.
• The resolution will help reduce environmental injustices, close protection gaps and empower people in vulnerable
situations, including environmental human rights defenders, children, youth, women and indigenous peoples.
• It also recognises that Climate change and environmental degradation are the most critical threats awaiting
humanity in the future.
• This refers to the three main interlinked environmental threats that humanity currently faces: climate change,
pollution and biodiversity loss.
• A similar resolution was adopted last year by the Human Rights Council.
• It calls upon States, international organisations, and business enterprises to scale up efforts to ensure a healthy
environment for all.

👉 Constitutional Provision in India

• Article 21 ➔ Right to life ( clean environment, free of disease)


27 | P a g e
• Article 48A ➔ (protect the environment and safeguard forests and wildlife)
• Art 51-A(g) ➔ (Duty to protect and improve the natural environment).

3. Living Lands Charter


Why in News?
At the 2022 Commonwealth Heads of Government Meeting (CHOGM) in Kigali, all 54 Commonwealth members have
officially adopted the Living Lands Charter: A Commonwealth Call to action on Living Lands (CALL).
About

• All Commonwealth members have agreed to voluntarily dedicate a ‘living land’ in their respective countries to
future generations.
• This Living Lands Charter is in line with the strategy set for the United Nations Decade on Ecosystem Restoration
(2021-2030).
• The non-binding ‘Living Lands Charter’ mandates that member countries will
o Safeguard global land resources
o Arrest land degradation and
o Act against climate change, biodiversity loss and towards sustainable management.
• It will help to encapsulate the combined effort to hold the global average temperature increase to 1.5 degrees
Celsius.
• The ‘Living Lands Charter’ aimed to support member countries to effectively deliver their commitments under the
three Rio conventions
o UN Convention on Biological Diversity,
o UN Convention to Combat Desertification (UNCCD) and
o UN Framework Convention on Climate Change (UNFCCC).

👉 Commonwealth

• Headquartered in London, the Commonwealth is an international intergovernmental organization of 56 countries.


• It was established by the London Declaration in 1949.
• It is headed by the Monarch of the United Kingdom.
• Membership ➔ The 56 member countries are mostly former territories of the British Empire and dependencies.
• But today any country can join the modern Commonwealth. The membership is based on free and equal voluntary
co-operation.
• The two African countries, Gabon and Togo, were admitted to the Commonwealth of Nations, as its members at the
CHOGM 2022.

4. American Bullfrog and Brown Tree Snake


Why in News?
According to a study, two invasive species namely the American bullfrog and
the brown tree snake cost the world economy an estimated $16 billion
between 1986 and 2020 by causing problems such as crop failure and
triggered costly power failures.

About Brown Tree Snake or Boiga Irregularis

• It is an invasive species that has multiplied uncontrollably on Pacific


islands including Guam and the Marianna Islands, where it was
introduced by U.S. troops in World War II.

28 | P a g e
• The snakes have at times been so abundant that they caused power outages by crawling on electrical equipment.

👉 American bullfrog

• It is a highly invasive species that had the greatest impact in Europe.


• A female bullfrog can lay up to 20,000 eggs at one time compared to native frogs which only lay 2,000 to 5,000 eggs.
They can eat nearly anything, including other bullfrogs.
• In Europe, exploding numbers of American bullfrogs have required ambitious and costly management programmes.
To prevent the spread of them, officials have been forced to install expensive frog-proof fencing around known
breeding sites.

5. Nagar Van Yojana


Why in News?
The Minister of State for Environment, Forest & Climate Change has informed Lok Sabha about Nagar Van Yojana.
About

• Initiative of ➔ Ministry of Environment, Forest & Climate Change.


• Aim ➔ To create Nagar Vans (urban forests) and Nagar Vatikas in the country with an objective to significantly
enhance the tree outside forests and green cover, enhancement of biodiversity and ecological benefits to the urban
and peri-urban areas apart from improving the quality of life of city dwellers.
• Target ➔ The scheme envisages developing 400 Nagar Vans and 200 Nagar Vatikas in the country during the period
of 2020-21 to 2024-25.
• Funding ➔ The finances for the scheme will be paid for by the CAMPA (Compensatory Afforestation Fund (CAF) Act,
2016) funds.

6. The Energy Conservation (Amendment) Bill 2022


Why in News?
Recently, the Ministry of Power has tabled the Energy Conservation (Amendment) Bill 2022 in Lok Sabha. The Bill
proposes to amend the Energy Conservation Act 2001 last amended in 2010.
About

• Energy Conservation Act, 2001 was enacted to provide for efficient use of energy and its conservation and for
matters connected therewith.
• Norms for energy efficiency ➔ The Act empowers the Centre to specify norms and standards of energy efficiency
for appliances, industrial equipment and buildings with a connected load over 100 kiloWatts (kW) or a contractual
demand of more than 15 kilovolt-amperes (kVA).
• BEE ➔ The Act established the Bureau of Energy Efficiency(BEE). The 2010 amendment extended the tenure of the
Director General of the Bureau of Energy Efficiency from three to five years. This Bureau can specify qualifications
required for energy auditors who monitor and review the power consumption of various industries.
• Energy Saving Certificates ➔ The Centre can issue energy savings certificates to those industries which consume
less than their maximum allotted energy.
• However, this certificate can be sold to customers who consume higher than their maximum allowed energy
threshold – providing for a framework for energy trading.
• The Act allows the Centre to prohibit the manufacture, sale, purchase or import of any particular equipment unless
it conforms to specified norms issued six months/ one year before.
• Penalty ➔ In case of any violations under this Act, each offence shall attract a penalty of Rs ten lakh with an
additional penalty of Rs 10,000 for each day the offence continues.

29 | P a g e
• Appeals ➔ Any appeals against any such order passed by the Central or state government will be heard by the
appellate tribunal already established under the Electricity Act, 2003.

👉 Proposed changes to the act

• Defining the minimum share of renewable energy to be consumed by industrial units or any establishment. This
consumption may be done directly from a renewable energy source or indirectly via the power grid.
• Incentivising efforts to use clean energy by issuing carbon saving certificates.
• Strengthening institutions set up originally under the Act, such as the Bureau of Energy Efficiency.
• Facilitating the promotion of green Hydrogen as an alternative to the fossil fuels used by industries.
• Considering additional incentives like carbon credits for the use of clean energy to lure the private sector to climate
action.
• Including larger residential buildings under energy conservation standards to promote sustainable habitats.
Currently, only large industries and their buildings come under the ambit of the Act.

7. Ranganathittu Bird Sanctuary


Why in News?
The Ranganathittu Bird Sanctuary in Karnataka has been declared a Ramsar site underlining its status as a wetland of
international importance.
About

• Also known as the Pakshi Kashi of Karnataka, Ranganathittu Bird Sanctuary is situated in Srirangapatna of Mandya
district, Karnataka.
• It is located on the islands of river Cauvery. It is an example of riverine ecosystem.
• It is an Important Bird Area (IBA) that is identified by Birdlife International and Bombay Natural History Society
(BNHS).
• It is declared a sanctuary in 1940 at the behest of the famous ornithologist Dr. Salim Ali
• Climate ➔ There are no extremities of temperature at Ranganathittu Sanctuary.
o The temperature stays between the 23-29°C range for most parts of the year.
o In June, when the southwest monsoon peaks throughout Karnataka state, heavy to very heavy rainfall,
accompanied by waterlogging and mild flooding, has been reported from the area.
• Significance ➔ Ranganathittu bird sanctuary supports more than 1% of the world’s population of spot-billed
pelicans - as against a global population of nearly 17,000.
o Similarly, it supports a high population of painted storks and mugger crocodiles besides other species of
fish.
o Therefore, the six islands and the water surrounding them would be part of Ramsar site.

8. Updated climate pledge


Why in News?
Recently, India updated its Nationally Determined Contribution (NDC).
About

• As per the Paris Agreement’s provisions, countries must ‘update’ their pledges every five years to make higher
commitments to greenhouse gas (GHG) emissions reductions.

👉 India’s first pledge:

• The country submitted its first pledge in 2015.


• India’s first pledge, also known as a Nationally Determined Contribution (NDC), had three primary targets.

30 | P a g e
o The first was to reduce the emissions intensity of the economy by
33–35 percent below 2005 levels.
o The second was to have 40 percent of installed electric power from
non-fossil-based energy resources by 2030.
o The third target was to create an additional (cumulative) carbon
sink of 2.5-3 gigatonnes of carbon dioxide equivalent (GtCO2e) by
2030 through additional forest and tree cover.

👉 Updated pledge:

• India now stands committed to reducing the emissions intensity of its GDP
by 45 percent by 2030 from its 2005 levels, as per the updated NDC.
• The country will also target about 50 percent of cumulative electric power
installed capacity from non-fossil fuel-based energy resources by 2030.
• To further a healthy and sustainable lifestyle, 'LIFE' 'Lifestyle for
Environment' as a key to combating climate change" has been added to
India's NDC.

9. Bhitarkanika National Park


Why in News?
Experts on saltwater crocodiles say the park has reached a saturation point, which could prove to be a big problem if not
addressed properly.
About

• It has the second-largest mangrove forest in India and is a Ramsar site. It was
declared as a Bhitarkanika National Park in the year of 1988.
• Bhitarkanika is located in the estuary of Brahmani, Baitarani, Dhamra, and
Mahanadi river systems. It is located in Kendrapara district of Odisha.
• It is one of Odisha’s finest biodiversity hotspots and is famous for its
mangroves, migratory birds, turtles, estuarine crocodiles, and countless creeks.
• It is said to house 70% of the country’s estuarine or saltwater crocodiles,
conservation of which was started way back in 1975.

👉 River Brahmani

• It is a river in northeastern Odisha state, eastern India. Formed by the confluence of the Sankh and South Koel
rivers in southern Bihar state, the Brahmani flows for 300 miles.
• It winds generally south-southeast past Bonaigarh and Talcher and then turns east to join northern branches of the
Mahanadi River, which then empties into the Bay of Bengal at Palmyras Point.
• It is one of the few rivers that cut across the Eastern Ghats, and it has formed a minor gorge at Rengali, where a
dam has been built.

10. Great Barrier Reef


Why in News?
According to the Australian Institute of Marine Science (AIMS) report, the highest levels of coral cover within the past 36
years have been recorded in the northern and central parts of Australia’s Great Barrier Reef (GBR).
About

• It is the world’s largest and longest coral reef system. It is located in Australia.

31 | P a g e
• It is a World Heritage Area since 1981 (the world’s first reef ecosystem to be recognised by UNESCO).
• It is home to 400 types of coral, 1500 species of fish and 4000 types of molluscs.

👉 Coral reef

• Corals are marine invertebrates or animals which do not possess a spine (phylum Cnidaria). They are the largest
living structures on the planet.
• A coral reef is an underwater ecosystem characterized by reef-building corals. Reefs are formed of colonies of coral
polyps held together by calcium carbonate.
• Coral Bleaching ➔ Coral bleaching is the process when corals become white due to various stressors, such as
changes in temperature, light, or nutrients. Bleaching occurs when coral polyps expel the algae that live inside their
tissue, causing the coral to turn white. GBR has faced several mass coral bleaching in the recent past (including the
recent one in March 2022).
• Importance of Corals ➔ Coral reefs support over 25% of marine biodiversity even though corals occupy only 1% of
the seafloor. Coral reef systems generate $2.7 trillion in annual economic value through goods and service trade
and tourism and other livelihood activities.
• In India, coral reefs are located in 7 regions ➔ Goa coast, Kerala coast, Palk Bay, Gulf of Kucch, Gulf of Mannar,
Lakshadweep islands, Andaman and Nicobar islands.

11. World Lion Day


Why in News?
World Lion Day is observed on 10th August annually to spread awareness and educate people about lions and their
conservation.
About Lion

• Scientific Name ➔ Panthera leo.


• The lion is divided into two subspecies: the African lion (Panthera leo leo) and the Asiatic lion (Panthera leo persica).
• Asiatic lions are slightly smaller than African lions.
• Role in the Animal Kingdom ➔
o Lions hold an indispensable place in the ecosystem, they are an apex predator of their habitat, responsible
for checking the population of grazers, thus helping in maintaining the ecological balance.
o Lions also contribute to keeping the population of their prey healthy and resilient as they target the
weakest members of the herd. Thus, indirectly helping in disease control in the prey population.
• Protection Status ➔
o IUCN Red List: Vulnerable
o Asiatic Lion - Endangered.
o CITES: Appendix I for populations of India, all other populations are included in Appendix II.
o Wildlife (Protection) Act 1972: Schedule I
• Status in India ➔
o India is home to the majestic Asiatic Lion, who inhabit the protected territory of Sasan-Gir National Park
(Gujarat).
o According to the data from 2020, there are 674 lions in India, which were 523 in 2015.
• Threats ➔ Poaching, genetic inbreeding arising from a single population inhabiting one place, diseases such as
plague, canine distemper or a natural disaster.

12. Agasthiyarmalai Elephant Reserve


Why in News?

32 | P a g e
On World Elephant Day, the Union Environment Ministry has declared a new elephant reserve called Agasthiyarmalai
Elephant Reserve.
About

• Located in ➔ Tamil Nadu.


• This will be the 5th elephant reserve in Tamil Nadu and the 32nd elephant reserve in India.

👉 Elephant Reserves in India

• Till now, India has 31 elephant reserves. In the last three years, Dandeli Elephant Reserve has been notified by
Karnataka, Singphan Elephant Reserve by Nagaland and Lemru Elephant Reserve in Chhattisgarh.
• These efforts brought a total of 76,508 sq km across 14 states in the country under elephant reserves.
• With the announcement of the Agasthiyamalai Tiger Reserve, India is adding yet another 1197 sq km of Protected
Area dedicated for the protection and conservation of elephants in India.

👉 About World Elephant Day

• World Elephant Day is an international annual event on August 12.


• Aim ➔ To create awareness of elephant conservation, and to share knowledge and positive solutions for the better
protection and management of wild and captive elephants.
• Conservation Status ➔ International Union for Conservation of Nature (IUCN) Red List of threatened species:
o African Forest Elephant- Critically Endangered
o African Savanna Elephant- Endangered
o Asian Elephant- Endangered
o Convention of the Migratory species (CMS): Appendix I
o Wildlife (Protection) Act, 1972: Schedule I

👉 Steps Taken

• Gaj Yatra ➔ Awareness campaign to protect elephants.


• MIKE programme (The Monitoring the Illegal Killing of Elephants (MIKE) programme, 2003) ➔ It is an international
collaboration for elephant protection.
• Project Elephant ( centrally sponsored scheme, 1992) ➔ To protect elephants and their habitats.

13. 11 New Ramsar Sites


Why in News?
India has added 11 more wetlands to the list of Ramsar sites to make a total 75 Ramsar sites.
About

• Tampara Lake in Odisha ➔ It is among the most prominent freshwater lakes in Odisha. The lake was called “Tamp”
by the British and subsequently termed “Tampra” by the locals. The lake is an important habitat for vulnerable
species such as Cyprinus carpio, common pochard (Aythya ferina) and river tern (Sterna aurantia).
• Hirakud Reservoir in Odisha ➔ It is the largest earthen dam in Odisha started operating in 1957. The reservoir
provides important hydrological services by moderating floods in the Mahanadi delta.
• Ansupa Lake in Odisha ➔ It is the largest freshwater lake in Odisha. It is an oxbow lake formed by River Mahanadi.
It provides a safe habitat to at least three threatened bird species- Rynchops albicollis (EN), Sterna acuticauda (EN)
and Sterna aurantia (VU) and three threatened fish species- Clarias magur (Clariidae) (EN), Cyprinus carpio
(Cyprinidae) (VU) and Wallago attu (VU).
• Yashwant Sagar in Madhya Pradesh ➔ It is one of the two Important Bird Areas(IBA) in the Indore region. Presently,
it is mainly used for water supply to the city of Indore. It is considered to be a stronghold of the vulnerable Sarus
Crane in central India.
33 | P a g e
• Chitrangudi Bird Sanctuary in Tamil Nadu ➔ It is locally known as “Chitrangudi Kanmoli”. It has been a protected
area since 1989 and has been an ideal habitat for winter migratory birds.
• Suchindrum Theroor Wetland complex in Tamil Nadu ➔ It is a part of Suchindrum-Theroor Manakudi
Conservation Reserve. It is declared an Important Bird Area and lies at the southern tip of the Central Asian flyway
of migratory birds.
• Vaduvur bird sanctuary in Tamil Nadu ➔ It is a large human-made irrigation tank and shelter for migratory birds
as it provides a suitable environment for food, shelter, and breeding ground.
• Kanjirankulam Bird Sanctuary in Tamil Nadu ➔ It is notable as a nesting site for several migratory heron species
that roost in the prominent growth of babul trees there.
• Thane Creek ➔ It is located in Maharashtra. There are several sources of fresh water in the creek of which Ulhas
River is the largest. It has been declared as Thane Creek Flamingo Sanctuary. It is fringed by mangroves on both
banks & comprises around 20% of the total Indian mangrove species.
• Hygam Wetland in Jammu and Kashmir ➔ It falls within the River Jhelum basin and plays a significant role as a
flood absorption basin, biodiversity conservation site, eco-tourism site and livelihood security for the local
communities. It is also recognized as an Important Bird Area (IBA).
• Shallabug Wetland Conservation Reserve in Jammu and Kashmir ➔ Large areas of the wetland dry up between
September and March. The area has extensive reedbeds of Phragmites communis and Typha angustata and rich
growth of Nymphaea candida and N. stellata on open water.

14. Great Indian Bustard
Why in News?
Great Indian Bustards(GIBs) in Rajasthan’s Desert National Park(DNP) have adopted an altogether new habit of laying a
clutch of two eggs at a time after having a diet with additional proteins during the monsoon season.
About

• GIBs are the largest among the four bustard species found in India, the other three being MacQueen’s bustard,
lesser florican, and the Bengal florican.
• GIBs’ historic range included much of the Indian sub-continent but it has now shrunken to just 10 percent of it.
• Among the heaviest birds with flight, GIBs prefer grasslands as their habitats.
• GIBs are considered the flagship bird species of grassland.
• Protection accorded
o Birdlife International: uplisted from Endangered to Critically Endangered (2011)
o Protection under CITES: Appendix I
o IUCN status: Critically Endangered
o Protection under Wildlife (Protection) Act: Schedule I.

👉 Desert National Park

• Desert National Park is a national park situated in the Indian state of Rajasthan. This is one of the largest national
parks.
• The Park is an excellent example of the ecosystem of the Thar Desert. Sand dunes form around 44% of the Park.
• The Chinkara or Indian Gazelle (Gazella bennettii) is a common antelope of this region. The Great Indian Bustard is
also found here in relatively fair numbers.

15. Tasmanian Tiger


Why in News?
Scientists in the US and Australia have embarked on a $15-million project to resurrect the thylacine or Tasmanian Tiger,
a marsupial that went extinct in the 1930s, using gene-editing technology.

34 | P a g e
About

• Tasmanian Tiger (Thylacinus cynocephalus) was a marsupial mammal. It was the only animal in the Thylacinidae
family to survive in modern times.
• Marsupial ➔ Marsupial is a mammal of an order whose members are born incompletely developed and are typically
carried and suckled in a pouch on the mother’s belly.
• Features ➔ The mammal earned its nickname Tasmanian Tiger because of the stripes along its back. It was a slow-
paced carnivorous that usually hunted alone or in pairs at night.
• It had a dog-like head and ate kangaroos, other marsupials, small rodents and birds.
• IUCN Status ➔ Extinct (The last Tasmanian tiger died at Hobart Zoo in 1936).
• Impact of Extinction ➔ The animal was at the top of the food chain and hence played a significant role in balancing
the ecosystem of its habitat by removing the weak animals and maintaining species diversity.
o Hence, its disappearance from the food chain resulted in Trophic Downgrading (causal degradation of an
ecosystem that occurs when higher trophic level animals are removed from the food chain) resulting in loss
or exponential growth of other species.

👉 De-extinction method

• De-extinction or resurrection biology is the method of creating a species that went extinct or is endangered, in order
to revitalize ecological diversity and balance shattered due to reasons ranging from biodiversity loss to climate
change.
• While cloning is the most widely used method of de-extinction, genome editing and selective breeding are also
considered effective ways.
• The Pyrenean ibex, a subspecies of Spanish ibex was one of the first extinct animals that have been resurrected
using somatic cell nuclear transfer(SCNT), even though the baby Ibex died minutes after its birth from a lung defect.

16. Megalodon
Why in News?
Research suggests, ancient Megalodon sharks could eat a whale in a few bites.
About Megalodon

• Scientific Name ➔ Otodus megalodon.


• It is an extinct species of mackerel shark which used to exist millions of years ago.
• It roamed the oceans an estimated 23 million to 2.6 million years ago.
• It was the largest shark to ever swim through Earth's oceans.
• Description ➔
o The giant transoceanic predator would have weighed around 70 tonnes — or as much as 10 elephants.
o It had an average cruising speed faster than sharks today.
o It had the ability to migrate across multiple oceans.
o It was bigger than a school bus at around 50 feet from nose to tail.
o In comparison, the great white sharks of the present can grow to a maximum length of around 15 feet.

17. Snailfish
Why in News?
A study has found a Snailfish that lives in an iceberg habitat in Greenland can survive in icy Arctic waters due to the
presence of antifreeze proteins in its bloodstream.
About

• The name snailfish can refer to any of the more than 400 species found in the family Liparidae.
35 | P a g e
• Sometimes they’re also called sea snails—not to be confused with gastropod sea snails (which are the animals).
• The Snailfish releases biofluorescence, which allows it to glow green and red in the dark arctic waters.
• Snailfish is the only polar fish reported to have biofluorescence.
• Biofluorescence is the ability of an organism to convert blue light into green, red, or yellow light.
o It is rarely found in Arctic fish due to prolonged periods of darkness in the region.
• Habitat ➔
o They are found all over the world, including in Antarctica.
o Although they’re found in shallow waters, the deep-sea species are the ones that stand out.

18. Air Quality and Health in Cities


Why in News?
The report titled ‘Air Quality and Health in Cities’ has been published by the State of Global Air.
About

• The State of Global Air is a collaboration between the Health Effects Institute (HEI) and the Institute for Health
Metrics and Evaluation (IHME).
• The report was released by US-based research organisation Health Effects Institute’s (HEI) State of Global Air
Initiative.

👉 Key findings

• Half of all of cities analysed across the globe had PM2.5 levels above even the least-stringent World Health
Organization (WHO) standards.
• India and Indonesia recorded the most severe increase in PM2.5 pollution while China saw the greatest
improvements.
• Delhi and Kolkata have the highest and second-highest levels of pollution globally in terms of Particulate Matter
(PM) 2.5.
• All 20 cities with the greatest decrease in PM2.5 pollution from 2010 to 2019 were found to be located in China.

19. Treaty on Ocean Biological Diversity


Why in News?
MANAGEMENT (LCCM)
The United Nations organised the Intergovernmental Conference to draft the first-ever treaty on the Ocean’s Biological
Diversity to conserve marine diversity in the High Seas.
About

• The treaty seeks to address the conservation and sustainable use


of marine biodiversity in areas of the ocean which are beyond the
limits of national maritime zones.
• It will also address marine genetic resources, including questions on
benefit-sharing, measures such as area-based management tools,
environmental impact assessments, and the transfer of marine
technology.
• The agreement will be deciding on the rights of companies that
undertake exploration for biological resources in the high seas.
o With advances in biotechnology and genetic engineering,
several companies see potential in exotic microbes and
other organisms – several of them undiscovered – that
abide in the deep ocean and could be used for drugs, vaccines and a variety of commercial applications.

36 | P a g e
• Since marine life is already reeling from the impact of industrial fishing, climate change and other extractive
industries, the treaty seeks to protect our global oceans.

👉 High Seas:

• The ocean surface and the water column beyond the EEZ are referred to as the high seas.
• It is considered as “the common heritage of all mankind” and is beyond any national jurisdiction.
• States can conduct activities in these areas as long as they are for peaceful purposes, such as transit, marine science,
and undersea exploration.
• How are the High Seas Regulated Currently? ➔ The United Nations Convention on the Law of the Sea (UNCLOS)
regulates activities in international waters, including sea-bed mining and cable laying.

👉 United Nations Convention on the Law of the Sea (UNCLOS)

• The ‘Law of the Sea Treaty’, formally known as the United Nations Convention on the Laws of the Sea (UNCLOS) was
adopted in 1982 to establish jurisdictional limits over ocean areas.
• The convention defines a distance of 12 nautical miles from the baseline as Territorial Sea limit and a distance of
200 nautical miles distance as Exclusive Economic Zone limit.
• It provides for technology and wealth transfer from developed to underdeveloped nations and requires parties to
implement regulations and laws to control marine pollution.
• India became a signatory to the UNCLOS in 1982.

20. Purslane
Why in News?
A study has found that a common weed named “Portulaca oleracea”, commonly known as purslane, offers important
clues about creating drought-tolerant crops in a world beset by climate change.
About

• What is it? ➔ Purslane is a succulent plant that grows in many countries because it thrives in poor soil.
• It has an extensive distribution extending from North Africa and Southern Europe through the Middle East and the
Indian subcontinent to Malaysia and Australasia.
• Purslane possesses evolutionary adaptations that allow it to be both highly productive and drought tolerant.

How does Purslane remain drought tolerant according to the study?

• Photosynthesis is the process by which green plants use sunlight to synthesize nutrients from carbon dioxide and
water.
• Over time, different species have independently evolved a range of distinct mechanisms to improve this process.
• For example, corn and sugarcane have evolved ‘C4 photosynthesis’, which allows them to remain productive under
high temperatures.
• Meanwhile, cacti and agaves have evolved ‘CAM photosynthesis’, which allows them to thrive in areas with little
water.
• The study has found that the purslane plant integrates both these two distinct metabolic pathways namely C4 and
CAM to create a novel type of photosynthesis. This allows the weed to endure drought while remaining highly
productive.

21. Pacific Bluefin Tuna


Why in News?
Cabinet approved Categorization of the CDRIas an ‘International Organization’.

37 | P a g e
About

• Appearance ➔ They have black or dark blue dorsal sides, with a greyish-green iridescence.
• Their bellies are dotted with silver or grey spots or bands.
• They have a series of small yellow fins, edged in black, running from the second dorsal fin to the tail.
• They have relatively small eyes compared to other species of tuna.
• Diet ➔ They are predatory and mainly eat squids and fish, such as sardines and anchovies, saury, herring,
pompanos, mackerel, hake, other tunas, and occasionally red crabs and krill.
• Habitat ➔ Most of the U.S. catch of Pacific bluefin tuna is within about 100 nautical miles of the California coast.

👉 Bluefin

• Scientific Name ➔ Thunnus Thynnus


• They are the largest tunas and can live up to 40 years. They migrate across all oceans and can dive deeper than
3,000 feet.
• There are three species of bluefin: Atlantic (the largest and most endangered), Pacific, and Southern.
• Status ➔ Endangered.

22. Arth Ganga


Why in News?
Director General of the National Mission for Clean Ganga spoke about the Arth Ganga model during his virtual keynote
address to the Stockholm World Water Week 2022.
About

• Background ➔ In 2019, PM first introduced the concept of Arth Ganga during the first National Ganga Council
meeting in Kanpur where he urged for a shift from Namami Gange (cleaning of Ganga) to the model of Arth Ganga.
• Focus ➔ The Arth Ganga focuses on the sustainable development of the Ganga and its surrounding areas, by
focusing on economic activities related to the river.
• The Arth Ganga model seeks to use economics to bridge people with the river. It strives to contribute at least 3%
of the GDP from the Ganga Basin itself.
• Features ➔ Under Arth Ganga, the government is working on six verticals:
o Zero Budget Natural Farming: It involves chemical-free farming on 10 km on either side of the river, and
the promotion of cow dung as fertilizer through the GOBARdhan scheme.
o Monetization and Reuse of Sludge & Wastewater: It seeks to reuse treated water for irrigation, industries
and revenue generation for Urban Local Bodies (ULBs).
o Livelihood Generation Opportunities by creating haats where people can sell local products, medicinal
plants and ayurveda.
o Increase public participation by increasing synergies between the stakeholders involved with the river.
o Promote the cultural heritage and tourism of Ganga and its surroundings, through boat tourism, adventure
sports and conducting yoga activities.
o Promote institutional building by empowering local administration for improved water governance.

23. Battery Waste Management Rules, 2022


Why in News?
Recently, the government notified Battery Waste Management Rules, 2022 to ensure environmentally sound
management of waste batteries.
About

• Aim ➔ To ensure environmentally sound management of waste batteries.


38 | P a g e
• These rules will replace the Batteries (Management and Handling) Rules, 2001.
• Batteries covered ➔ The rules cover all types of batteries, viz. Electric Vehicle batteries, portable batteries,
automotive batteries and industrial batteries.

👉 Key provisions of the rules?

• Extended Producer Responsibility(EPR) ➔ Producers (including importers) of batteries are responsible for the
collection and recycling /refurbishment of waste batteries and the use of recovered materials from waste into new
batteries.
o The rules enable the setting up of a mechanism and centralized online portal for the exchange of Extended
Producer Responsibility (EPR) certificates.
• Polluter Pay Principle ➔ Environmental compensation will be imposed for non-fulfilment of EPR targets.
o The funds collected under environmental compensation shall be utilized in the collection and refurbishing
or recycling of uncollected and non-recycled waste batteries.
• Industries for waste Batteries ➔ Promote setting up of new industries and entrepreneurship in collection and
recycling /refurbishment of waste batteries.
• Use of recycled materials for new batteries ➔ The rules prescribe the use of a certain amount of recycled materials
in the making of new batteries.
• Monitoring the implementation of rules ➔ Online registration & reporting, auditing, and committee for monitoring
the implementation of rules and taking measures required for removal of difficulties.

24. Seti River


Why in News?
India is to develop two hydropower projects in Nepal, namely the West Seti Hydropower Project and the Seti River
Hydropower Project.
About Seti River

• The Seti Gandaki River is also known as the Seti River or the Seti Khola.
• The Seti River is a river of western Nepal.
• It originates from the glaciers around the twin peaks of Api and Nampa in
the south facing slopes of the main Himalayas. The area is near the tri
junction of the borders of Nepal, India (Uttarakhand), and China.
• The river finally joins the Karnali or Ghagra River (which is Nepal's longest
river).

👉 About the Project

• The MoU was signed between Investment Board Nepal and National
Hydro Power Corporation Ltd(India) to develop the 1200 MW project.
• Benefits ➔ The projects will produce power all year round. Through its national grid, India will receive the power,
either for its own use or for export.
• Background ➔ In 2017, China Three Gorges International Corporation set up a joint venture to develop the project.
It withdrew from the project in 2018.

25. Swachh Sagar, Surakshit Sagar/Clean Coast, Safe Sea Campaign


Why in News?
Recently, The Union Environment Minister participated in the ‘Swachh Sagar, Surakshit Sagar’ campaign, which aims at
promoting environmental sustainability.
About
39 | P a g e
• Swachh Sagar, Surakshit Sagar/Clean Coast, Safe Sea Campaign is a 75-day citizen-led initiative aimed at improving
coastal and ocean health via community action.
• The campaign began on July 5, 2022 and will culminate on 17th September 2022 – International Coastal Clean-up
Day.
• Objective ➔ To alter and save the environment through behaviour change.
• Goals ➔ The campaign’s three overarching goals are: 1) Consume Responsibly 2) Separate waste at home and 3)
Dispose of waste responsibly.
• Nodal Ministry ➔ The campaign is spearheaded by the Ministry of Earth Sciences (MoES) in partnership with the
Ministry of Environment Forest and Climate Change (MoEFCC), Indian Coast Guard, National Disaster Management
Authority (NDMA) and other institutions.
• Features ➔ The campaign is being taken at 75 beaches along the coastline with average of 75 volunteers for every
kilometre of the coastline. National conferences and workshops on the conservation of the Coastal Ecosystem,
marine environment, safe beaches, and waste management are also being conducted.

👉 Other Related steps

• UN "Coastal Clean Seas" campaign ➔


o India is a signatory to the UN "Coastal Clean Seas" campaign.
o A number of activities have been adopted that have direct relevance with the "Swachh Bharat" vision and
have proved highly useful.
• International Coastal Cleanup Day ➔
o The International Coastal Cleanup Day is celebrated globally on the third Saturday of September, every
year.

26. Center for Wildlife Rehabilitation and Conservation


Why in News?
The CWRC near Kaziranga National Park has completed 20 years of providing emergency care, treatment and
rehabilitation to indigenous wild animals displaced due to various reasons.
About

• It is a joint initiative of the Assam Forest Department, the Wildlife Trust of India (WTI) and the International Fund
for Animal Welfare (IFAW)
• It was established on August 28, 2002.
• Facilities ➔ It currently has two satellite facilities called mobile veterinary services. These are located in eastern
Assam at Guijan in Tinsukia district (Dibru Saikhowa National Park) and in western Assam at Charaikhola
(Chakrashila Wildlife Sanctuary).
• Achievements ➔ It continues to be the only facility in India to have successfully addressed the welfare and
conservation of 357 species including elephant, leopard, rhino, tiger, clouded leopard, black bear, wild buffalo, hog
deer, muntjac, wild boar and monkeys.
o It has so far handled 7,397 animals out of which 4,490 (65%) could be sent back to the wild after proper
care and treatment.
o It has become a model now in the field of wildlife conservation and its needs in Assam.

👉 Kaziranga National Park

• Kaziranga National Park in the northeast Indian state of Assam is a world heritage site, notified by UNESCO in 1985.
• It is the oldest park in Assam along the river Brahmaputra on the North and the Karbi Anglong hills on the South.
• National Highway 37 passes through the park area and tea estates, hemmed by table-top tea bushes.

40 | P a g e
• The park holds the world’s largest population of the greater one-horned rhinoceros and provides refuge to a large
number of wild animals including the endangered Royal Bengal tiger and the Asian elephant.

41 | P a g e
Science and Technology
1. AlphaFold & Protein

Why in News?
DeepMind, a company based in London, announced that it had predicted the three-dimensional structures of more than
200 million proteins using AlphaFold.
About

• AlphaFold is an AI-based protein structure prediction tool.


• It is based on a computer system called deep neural network.
• Working? ➔
o It uses processes based on “training, learning, retraining and relearning”. By using this method, AlphaFold
has now predicted the structures of the entire 214 million unique protein sequences deposited in the
Universal Protein Resource(UniProt) database.
• What are the implications of this development? ➔
o Proteins are the business ends of biology, meaning proteins carry out all the functions inside a living cell.
Therefore, knowing protein structure and function is essential to understanding human diseases.
o Scientists predict protein structures using x-ray crystallography, nuclear magnetic resonance spectroscopy,
or cryogenic electron microscopy.
o These techniques are not just time-consuming, they often take years and are based mainly on trial-and-
error methods.
o Therefore, the development of AlphaFold changes all of that. It is a watershed movement in science and
structural biology in particular.
• Is AlphaFold the only tool for predicting protein structures? ➔
o AlphaFold is neither flawless nor the only AI-based protein structure prediction tool.
o RoseTTaFold, developed at the University of Washington in Seattle, U.S., is another tool. Although less
accurate than AlphaFold, it can predict the structure of protein complexes.

2. African Swine Fever


Why in News?
African Swine Fever has been confirmed for the first time, at a private pig farm in in Kerala, after more than 15 pigs on
the farm had died due to the disease in the last ten days.
About

• African swine fever(ASF) is a highly contagious hemorrhagic viral disease of domestic and wild pigs which is
responsible for serious economic and production losses.
• Caused by ➔ It is caused by a large DNA virus of the Asfarviridae family, which also infects ticks of the genus
Ornithodoros.
• Other manifestations of the disease include ➔ High fever, Depression,, Anorexia, Loss of appetite, Hemorrhages in
the skin, Vomiting and diarrhoea among others.
• It was first detected in Africa in the 1920s.
• The mortality is close to 95% - 100% and since the fever has no cure, the only way to stop its spread is by culling the
animals.
• ASF is not a threat to human beings since it only spreads from animals to other animals.
• ASF is a disease listed in the World Organisation for Animal Health (OIE)’s Terrestrial Animal Health Code.
• Transmission ➔
o Direct contact with infected pigs, faeces or body fluids.
42 | P a g e
o Indirect contact via fomites such as equipment, vehicles or people who work with pigs between pig farms
with ineffective biosecurity.
o Pigs eating infected pig meat or meat products.
o Biological vectors - ticks of the species Ornithodoros.

👉 World Organisation for Animal Health

• WOAH was formerly called the “Office International des Epizooties”.


• OIE is an intergovernmental organisation responsible for improving animal health worldwide.
• It has 182 Member Countries.
o India is one of the member countries.
• OIE develops normative documents relating to rules that Member Countries can use to protect themselves from the
introduction of diseases and pathogens.
• OIE standards are recognised by the World Trade Organization as reference international sanitary rules.
• It is headquartered in Paris, France.

3. Supercapacitors
Why in News?
A team of Indian scientists have developed a new low-cost, pristine, conducting polymer-based electrode/redox-active
electrolyte combination that can give enhanced electrochemical performance and cycling stability to supercapacitors.
About

• What is it? ➔ Supercapacitors are electronic devices which are used to store extremely large amounts of electrical
charge.
• They are also known as double-layer capacitors or ultracapacitors.
• Supercapacitors combine the characteristics of conventional capacitors and batteries to give a sudden kick-start to
devices by providing a large amount of power and sustained energy release.
• The electrode materials of supercapacitors play a vital role in determining the performance and stability of energy
devices.
• Conducting polymers like polyaniline and polypyrrole are excellent candidates for electrode materials owing to
their flexibility, stability and tunable electrical and electrochemical properties. They are also inexpensive, lightweight
and can be synthesized easily.
• However, supercapacitors fabricated using these electrodes fail to sustain the initial electrochemical capacitance
after a few cycles of continuous operation.
• The poor energy density of these devices is another issue that limits the use of these devices in practical applications.

👉 What have the scientists developed now?

• Scientists have found a strategy to improve the performance of polyaniline(PANI)-based supercapacitors.


• They found that electrodes made from polyaniline(PANI) when used with an electrolyte powered with an additive
that boosts redox reactions (redox-additive) can drive these energy storage devices to deliver incredible
performances.

4. Astrobee
Why in News?
For the first time ever, two Astrobee robots have begun working independently on the International Space Station, side
by side with humans.

About

43 | P a g e
• Astrobee is NASA’s new free-flying robotic system.
• Purpose ➔ The robots are designed to complete tasks such as taking inventory, documenting experiments
conducted by astronauts with their built-in cameras or working together to move cargo throughout the station.
o It also consists of a system that serves as a research platform that can be outfitted and programmed to
conduct microgravity experiments.Thus, it will help to learn more about how robotics can benefit
astronauts in space.
• Three Flying Robots of Astrobee ➔ The three free-flying robots are named Honey, Queen, and Bumble. The robots
are shaped like cubes 12.5 inches wide.
o The three robots propel themselves using electric fans that allow them to fly through the microgravity
environment of the International Space Station.
o They “look around” and navigate their surroundings using cameras and sensors.
o When they are running low on charge, they can automatically return to their docking station to begin
recharging.
• Based on ➔ The Astrobee robots are built on the knowledge acquired from operating SPHERES (Synchronised
Position Hold, Engage, Reorient, Experimental Satellite) robots which have been operating on the International
Space Station for over a decade. Once fully commissioned, the Astrobee system will take over for SPHERES as the
space station’s robotic test facility.

5. Azaadi SAT
Why in News?
The Indian Space Research Organization(ISRO) is all set to launch a Small Satellite Launch Vehicle(SSLV) from Sriharikota.
The vehicle will carry an Earth Observation Satellite named EOS-02 and a co-passenger satellite, ‘Azaadi SAT’ into low
earth orbit.
About

• Azaadi satellite has been designed by 750 girl students of government schools in rural areas from Kashmir to
Kanyakumari.
• Purpose ➔ The satellite weighs around 8 kg and has 75 Femto experiments (selfie cameras to click pictures of its
own solar panels and long-range communication transponders).
• It also contains a long-range transponder and a solid-state PIN diode-based radiation counter to measure the
ionizing radiation in its orbit.
• The satellite will also carry a recorded version of the national anthem sung by Rabindranath Tagore, which will be
played in space as a tribute to the country.
• Significance ➔ The satellite is the result of ISRO pushing for girls to take up Science, Technology, Engineering, and
Mathematics(STEM).

👉 EOS-02

• EOS-02 is an Earth Observation Satellite. It will provide information about thermal anomalies in the field of geo-
environmental studies, forestry, hydrology, agriculture, soil and coastal studies.

👉 Small Satellite Launch Vehicle(SSLV)

• It is an all-solid three-stage vehicle with the capability to launch up to 500 kg satellite mass into 500 km Low Earth
Orbit(LEO).
• Advantages of SSLV? ➔ The SSLV would help in a) Reduced Turn-around Time b) Launch on Demand c) Cost
Optimization for Realization and Operation d) Flexibility in accommodating Multiple Satellites and e) Minimum
launch infrastructure requirements.

6. Type Ibn Supernovae(SNe)

44 | P a g e
Why in News?
A team of scientists has deciphered the physical characteristics of a rare class of supernovae called Type Ibn Supernovae
(SNe).
About

• Supernovae are a kind of energetic explosions were the core of massive stars (a few times that of mass of our Sun)
go to a catastrophic phase of explosion liberating huge amounts of energy.

👉 Type Ibn Supernovae(SNe)

• Type Ibn supernovae are a rare class of stripped-envelope supernovae interacting with a helium-rich dense
circumstellar medium(CSM).
• These SNe are unique and essential because they help to explore the typical density, velocity, and composition of
the nearby SN environment.
• They are more luminous than normal supernovae(SNe) which are huge stellar explosions releasing a humongous
amount of energy.

👉 What are Wolf-Rayet stars?

• Wolf-Rayet stars represent a final burst of activity before a huge star begins to die. These stars, which are at least
20 times more massive than the Sun, “live fast and die hard”.

7. Exercise Skylight
Why in News?
The Indian Army recently conducted “Skylight Mega-Exercise”, to enhance its space domain capabilities.
About

• It was first of its kind large-scale exercise.


• Conducted by ➔ Indian Army
• Aim ➔ To test the operational readiness and robustness of its satellite communication systems in the event of an
attack by an adversary.

👉 Does the Indian Army have its own dedicated satellite?

• The Indian Army uses the services of a number of ISRO satellites that connect hundreds of communication terminals
of various types.
• Unlike the Indian Air Force and the Navy, the Indian Army currently does not have a dedicated satellite.

👉 Indian Army Satellite by 2025: GSAT-7B Satellite

• The Defence Acquisition Council has cleared a proposal for a GSAT-7B communications satellite. With this, the Indian
Army is on course to get its own satellite by December 2025.
• Note: GSAT 7 satellites are advanced satellites developed by the Indian Space Research Organization(ISRO) to meet
the communication needs of the defence services.

8. Vasculitis
Why in News?
Actor Ashton Kutcher had a “weird, super rare form of vasculitis” two years ago that “knocked out” his vision, hearing,
and “equilibrium”. He mentioned about it in a recent tweet.
About

45 | P a g e
• Vasculitis is a general term for several conditions that cause inflammation of blood vessels.
• It is also called angiitis (“inflammation within blood vessels”) or arteritis (“inflammation in arteries”).
• It is an auto-immune disease in which the body’s immune system turns on healthy blood vessels, causing them to
swell up, narrow down, stretched, or weak. The blood vessels might close entirely.
• The trigger for vasculitis may be an infection or a drug or blood cancers or immune system diseases, although the
precise reason is often uncertain or unknown.
• Vasculitis can be only a minor problem affecting the skin, or it can be a serious condition that impacts the heart,
kidneys or other vital organs.
• Types of Vasculitis ➔ There are around 20 different
disorders that are classified as vasculitis.
o Although the diseases are similar in some
ways, they often differ with respect to which
organs are affected, which medications are
used to treat them, and other characteristics.
• Treatment ➔ Different types of inflammation cause
different diseases, which have their own symptoms
and treatment protocols.
o Steroids are frequently prescribed, as are
some other medicines that reduce the activity of the immune system.

9. Novel Langya Henipavirus


Why in News?
Almost three years after the novel coronavirus was detected in China, the novel Langya virus (LayV) has been discovered
in the two eastern provinces of China.
About

• Langya Henipavirus (LayV) was discovered in eastern China during surveillance testing of patients who had fever
along with a recent history of animal exposure.
• It is a type of Henipavirus, a category of zoonotic viruses that can jump from animals to humans.
• It is closely related to the Mojiang Henipavirus, which was discovered in southern China.
• Symptoms ➔ Fever, fatigue, cough, nausea, headache are the common symptoms of LayV disease.
o The other symptoms include impaired liver function, impaired kidney function, leukopenia and
thrombocytopenia.
• Transmission ➔ In all likelihood, the new virus has jumped from an animal to humans.
o The LayV virus RNA has been predominantly found in shrews, which may be its natural hosts.
o Among domestic animals, seropositivity was detected in goats and dogs.
o So far, there have been no cases of human-to-human transmission of the virus. However, the possibility of
spread cannot be ignored.
• Detection ➔ A standardized nucleic acid testing method is required to identify the Langya virus.
• Treatment ➔ There is currently no vaccine or treatment for Henipavirus and the only treatment is supportive care
to manage complications.

👉 Henipaviruses

• Henipaviruses are classified as Bio-safety Level 4 (BSL4) pathogens.


• They can cause severe illness in animals and humans, and as of now there are no licensed drugs or vaccines meant
for humans.

46 | P a g e
• The types of Henipaviruses that had been identified prior to this included Hendra, Nipah, Cedar, Mojiang and the
Ghanaian bat virus.
• According to the US CDC, the Cedar virus, Ghanaian bat virus, and Mojiang virus are not known to cause human
disease. But Hendra and Nipah infect humans and can cause fatal illness.

10. Chronic Fatigue Syndrome


Why in News?
A woman from Bengaluru has filed a petition in the Delhi High Court to stop her friend who has been suffering from
Chronic Fatigue Syndrome since 2014 from travelling to Europe to undergo physician-assisted euthanasia.
About

• What is it? ➔ Chronic Fatigue Syndrome is also known as myalgic encephalomyelitis (ME/CFS).
• It is a serious and debilitating disease that affects the nervous system, the immune system and the body’s
production of energy.
• Causes ➔ Its causes are still unknown. However, the potential triggers would include viral or bacterial infection,
hormonal imbalances and genetic predispositions. There is no specific test for the disease and doctors have to rely
on medical examinations, and blood and urine tests.
• Vulnerable Group ➔ ME/CFS can affect anyone from children to adults of all ages. But it is more common in women
and people between 40 and 60 years old.
• Symptoms ➔ The biggest symptom is significantly lowered ability to do activities that were performed before the
illness. This is accompanied by at least 6 months (or longer) of debilitating fatigue that is more severe than everyday
feelings of tiredness. This fatigue is not relieved by sleep or rest and exercising usually makes the symptoms worse.
o The most recognizable symptom is post-exertional malaise(PEM). Patients often describe it as a “crash” in
physical/mental energy following even minor activities like grocery shopping or brushing teeth.
• Treatment ➔ As of yet, there is no specific cure or approved treatment. Instead, doctors recommend ways to deal
with the symptoms of the disease such as ‘pacing’ in which patients learn to balance rest and activity to prevent
crashes caused by exertion.
11. Cornea
Why in News?
Researchers in Sweden have developed a successful alternative — bioengineered cornea implants made of collagen
derived from pig skin.
About

• The cornea is the clear outer layer at the front of the eye. The cornea helps the eye to focus light so that one can
see clearly.
• What happens if the cornea is damaged? ➔ Damage to the cornea is one of the leading causes of blindness across
the world leaving approximately 12.7 million people blind.
• Researchers claim that there is a severe shortage of donated human corneas with only one available for 70 patients.
Logistical and storage difficulties, along with expensive surgical equipment, further burden those living in low- and
middle-income countries.

👉 What have researchers developed to overcome this shortage?

• As a substitute for human corneas, the researchers have developed a successful alternative — bioengineered cornea
implants made of collagen derived from pig skin.
• Benefits: This method is not only cheaper and easier to access than donated corneas but requires a less invasive
procedure and can be stored for a significantly longer period — up to two years.

47 | P a g e
12. STEVE Phenomenon
Why in News?
The cosmic phenomenon known as “Strong Thermal Emission Velocity Enhancement (STEVE) made a surprise
appearance after a huge solar storm hit Earth.
About

• STEVE first appeared in the sky in 2017 and has been appearing often since.
• There is evidence to support the claim that the phenomenon has been visible in the sky since 1705
• It is a rare, mysterious, glowing purple arc that runs east to west and occurs closer to the equator.
• How does it occur? ➔
o The phenomenon is believed to be caused by a 25 km wide ribbon of hot plasma moving at a speed of 6
km/s (compared to 10 m/s outside the ribbon) at an altitude of 450 km
o It has been observed in New Zealand, Canada, Alaska and the United Kingdom during a specific time of the
year - between October to February which lead NASA scientists to believe that its occurrence is related to
seasons.
• How is it different from Aurora Borealis? ➔
o Disturbances in the Earth's magnetosphere due to solar wind cause Auroras.
o These disturbances occur because of enhancements in the speed of the solar wind from coronal holes and
coronal mass ejections. "
o While the skylights, STEVE and the Auroras might seem similar, they are nothing alike.
o To begin with, the STEVE phenomenon is closer to the equator than the Auroras.
o STEVE is different from the usual aurora, but it is made of light and it is driven by the auroral system.

13. Hayabusa-2 probe


Why in News?
Hayabusa-2 probe has brought 5.4 grams of rocks and dust from the asteroid Ryugu.
About

• Hayabusa-2 mission was launched in December 2014.


• It is a six-year voyage to study the asteroid Ryugu.
• In 2018 the spacecraft reached the asteroid and deployed two rovers and a small lander onto the surface.
• In 2019, the spacecraft fired an impactor to create an artificial crater to collect the samples.
• A small capsule containing the rock and dust samples landed safely landed in the South Australian outback in 2020.
• Findings of the study
o In an article published in the journal Nature Astronomy on August 15, scientists from Japan suggest that
water and organic materials might have been brought to our planet from the outer edges of the solar
system.

👉 Asteroid

• Asteroids are rocky objects that orbit the Sun, much smaller than planets. They are also called minor planets.
• There are 994,383 known asteroids.
• It is divided into 3 classes.
o Asteroid between Mars and Jupiter.
o Trojans, which are asteroids that share an orbit with a larger planet.
o Near-Earth Asteroids (NEA), the orbits of the asteriods pass close to the Earth.
• More than 10,000 such asteroids are known, out of which over 1,400 are classified as potentially hazardous
asteroids (PHAs).
48 | P a g e
14. Lactose Intolerance
Why in News?
Studies on the global level suggest that 65% of humanity is lactose-intolerant.
About

• Milk has often been branded as a superfood as it is rich in most of the nutrients necessary for health.
• However, how milk came to be an integral part of the human diet has been a conundrum to scientists because most
of the world can’t digest the product.
• Lactose intolerance means, the individual lacks the gene to break down lactose into adulthood.
• Beyond the age of five, lactose, a sugar present in milk, cannot be naturally broken down in the stomach and this
remains in the gut causing flatulence, acidity and diarrhoea.
• India is among the largest producers of milk and, by country, the largest consumer of it.
• Milk drinking, the story goes, hasn’t been very popular in the roughly 3,00,000-year history of humanity.
• However, in the last 5,000 years, a genetic mutation enabled European pastoralists to produce lactase.
• Drinking milk is actually harmful in those who lacked the gene-variant but only in periods of famine and adverse
environmental conditions.

👉 Lactose persistence

• Lactase, an enzyme that breaks down lactose into a digestible form, well into adulthood, a trait called lactose
persistence.
• The lactase persistence trait is more common in populations that practice cattle herding and dairy farming, and it
is related to genetic selection of individuals with the ability to digest lactose.

15. Facial Recognition Technology


Why in News?
Right to Information (RTI) responses received by the Internet Freedom Foundation reveal that the Delhi Police treats
matches of above 80% similarity generated by its facial recognition technology (FRT) system as positive results.
About

• Facial recognition is an algorithm-based technology which creates a digital map of the face by identifying and
mapping an individual’s facial features, which it then matches against the database to which it has access.
• It can be used for two purposes ➔
o firstly, 1:1 verification of identity wherein the facial map is obtained for the purpose of matching it against
the person’s photograph on a database to authenticate their identity. For example, 1:1 verification is used
to unlock phones. However, increasingly it is being used to provide access to any benefits or government
schemes.
o Secondly, there is the 1:n identification of identity wherein the facial map is obtained from a photograph
or video and then matched against the entire database to identify the person in the photograph or video.
Law enforcement agencies such as the Delhi Police usually procure FRT for 1:n identification.
• The use of FRT presents two issues ➔ issues related to misidentification due to inaccuracy of the technology and
issues related to mass surveillance due to misuse of the technology.

16. Tomato Flu


Why in News?
Lancet has sounded an alarm over the emergence of a new non-life-threatening virus called tomato flu among children
below five years of age.

49 | P a g e
About

• What is it? ➔ Tomato Flu is a viral disease. It is caused by Coxsackie virus A 16. It belongs to Enterovirus family.
• Note ➔ Hand-foot-and-mouth-disease(HFMD) is a frequent febrile rash illness of childhood caused by
enteroviruses(EV).
• Named After ➔ The flu was named on the basis of the eruption of red blisters giving a resemblance to a tomato.
• Symptoms ➔ The primary symptoms observed in children with tomato flu are similar to those of chikungunya,
which include high fever, rashes, and intense pain in joints.
• Transmission ➔ Tomato flu is very contagious and children are at increased risk of exposure to tomato flu as viral
infections are common in this age group and spread is likely to be through close contact.
• Treatment ➔ Tomato flu is similar to chikungunya and dengue as well as hand, foot, and mouth disease. Hence,
the treatment is also similar — isolation, rest, plenty of fluids, and a hot water sponge for the relief of irritation
and rashes.

17. India's first observatory


Why in News?
India's first situational awareness observatory to monitor space activity is going to be set up in Uttarakhand by Digantara.
About

• The space situational awareness (SSA) observatory will assist in tracking any activity in space including that of space
debris and military satellites hovering over the region.
• It aims to track objects as small as 10 cm in size orbiting the earth.
• Significance ➔ It will fill the crucial gap in SSA observations in the region as there is a lack of such facilities between
Australia and southern Africa".
o The observatory will help to monitor events occurring in deep space, especially in the geostationary,
medium-Earth, and high-Earth orbits.
o The observatory will give India indigenous capabilities to monitor space activity over the subcontinent
providing a strategic advantage.
• Global Scenario ➔ At present, the United States is a dominant player in monitoring space debris. It has
observatories in multiple locations with commercial companies providing additional inputs from across the world.

18. Novae and Supernova


Why in News?
Scientists from SN Bose Centre for Basic Science(SNBCBS) studied the thick dust formed around the novae called Nova
V1280 Scorpii. The study of this cosmic dust could unravel the mysteries behind the start of life.
About

• A nova is an explosion from the surface of a white dwarf star in a binary star system. A nova occurs when the white
dwarf, which is the dense core of a once-normal star, steals gas from its nearby companion star. When enough gas
builds up on the surface of the white dwarf it triggers an explosion.
• A supernova is a violent stellar explosion that can shine as brightly as an entire galaxy of billions of normal stars.

👉 Cosmic Dust

• Cosmic dust is also called extraterrestrial dust or space dust. It is dust which exists in outer space or has fallen on
Earth.
• Cosmic dust was once solely an annoyance to astronomers, as it obscures objects they wished to observe.

50 | P a g e
• When infrared astronomy began, the dust particles were observed to be significant and vital components of
astrophysical processes. Their analysis can reveal information about phenomena like the formation of the Solar
System.
• For example, cosmic dust can drive the mass loss when a star is nearing the end of its life, play a part in the early
stages of star formation, and form planets.

19. Sickle cell disease


Why in News?
African health ministers launched a campaign to ramp up awareness, bolster prevention and care to curb the toll of sickle
cell disease.
About

• What is it? ➔ It is a group of inherited red blood cell disorders. Red blood carries oxygen to all parts of the body.
• Healthy red blood cells are round, and they move through small blood vessels to carry oxygen to all parts of the
body.
• In someone who has SCD, the hemoglobin is abnormal, which causes the red blood cells to become hard and sticky
and look like a C-shaped farm tool called a “sickle.”
• The sickle cells die early, which causes a constant shortage of red blood cells.
• Types ➔ There are several types of SCD. The specific type of SCD a person has depends on the genes they inherited
from their parents. People with SCD inherit genes that contain instructions, or code, for abnormal hemoglobin.
• Cause ➔ SCD is a genetic condition that is present at birth. It is inherited when a child receives two genes—one
from each parent—that code for abnormal hemoglobin.
• Prevention ➔ Management of SCD is focused on preventing and treating pain episodes and other complications.
Prevention strategies include lifestyle behaviors as well as medical screening and interventions to prevent SCD
complications.

👉 Recent Initiatives

• The Unmukt Project is being implemented by the Central Government to strengthen screening and timely
management of Sickle Cell Anemia.
• The Tribal Affairs Ministry established the National Council on Sickle Cell Disease and Tribal Health Cell, which would
coordinate with the Ministry of Health and State Governments.
• The Tribal Affairs Ministry, through development of Sickle Cell Support Corner, has instituted a mechanism for
creating a Central Repository of data.

20. CSA6
Why in News?
Scientists have identified a gene called CSA6. This gene can hold the key to prevent fungal infection Candidiasis.
About CSA6

• CSA6 is a gene identified in Candida albicans, a fungal species infamous for causing high rates of morbidity and
mortality under certain immuno-compromised conditions such as AIDS or during cancer treatment.
• This gene can hold the key to prevent the fungal infection Candidiasis which often affects intensive-care unit(ICU)
patients, cancer patients and patients receiving immunosuppressive therapy.

👉 Candidiasis

• White Fungus is also known as Candidiasis. It is a type of fungal infection caused by a yeast (a type of fungus) called
Candida.

51 | P a g e
• Candida normally lives on the skin and inside the body in places such as the mouth, throat, gut without causing any
problems.
o However, candida can cause infections if it grows out of control or if it enters deep into the body (for
example, the bloodstream or internal organs like the kidney, heart, or brain).
• Symptoms ➔ Symptoms of white fungus vary depending on the area affected. Most infections result in minimal
complications such as redness, itching, and discomfort. However, complications may be severe or even fatal if left
untreated in certain populations.
• Prevention ➔
o Patients who are on oxygen or ventilators should be taken care of in terms of hygiene.
o The tubes and equipment should be thoroughly cleaned. Only sterilized water should be used in oxygen
cylinder humidifiers.
• Treatment ➔ It is not life-threatening. The treatment of White Fungus disease is mainly done through antifungal
medicines. However, this changes if a person contracts severe disease and lungs are infected.

21. Data Sonification


Why in News?
NASA has created a 'sonification' of the sound emerging from the black hole at the centre of the Perseus galaxy cluster.
About

• Data Sonification is the translation of astronomical data into sounds.


• This sonification translates the actual sound waves discovered in data from various NASA missions - such as the
Chandra X-ray Observatory, Hubble Space Telescope, and Spitzer Space Telescope.
• There is a misconception that there is no sound in space because most of the space is a vacuum, with no medium
for sound waves to travel.
• However, galaxy clusters enveloping thousands of galaxies within it often have large amounts of sound, providing a
medium for the sound waves to travel.
• In the sonification of Perseus, the sound waves previously identified by astronomers were extracted in radial
directions (outwards from centre).
• After extraction, the signals were synthesised into the range of human hearing by scaling them up to 58 octaves
above their actual pitch.

22. Dunaliella tertiolecta


Why in News?
Researchers from the Indian Institute of Technology (IIT) Guwahati has developed an edible material, using marine
microalga Dunaliella tertiolecta, that can be coated on vegetables and fruits to extend their shelf life.
About

• Dunaliella tertiolecta is a marine microalga or the phytoplankton known for its antioxidant properties and has
various bioactive compounds such as carotenoids and proteins.
• Dunaliella is also used to produce algal oil, which is a non-animal source of omega-3 fatty acid, and is a good source
of biofuel.
• After the oil is extracted, the residue is usually discarded.
• However, the researchers have now used these extracts from the residue in formulating their edible film, in
combination with chitosan.
• Edible Film ➔ It has anti-microbial and anti-fungal properties.
o This material displayed superior antioxidant activity, thermal stability, mechanical strength, total phenolic
content and water vapour barrier property. It also had excellent UV-Vis light-blocking properties.

52 | P a g e
• Coating Technique ➔ The material can be either directly coated on the vegetables and fruits or made into a
vegetable storage pouch.
o In both cases, the shelf life of the vegetables can be extended.
o It is a simple dip coating technique with no significant cost added.
• Benefits ➔ The new coatings can be mass-produced.
o They are very stable to light, heat, and temperature up to 40oC, and can be safely eaten as part of the
product formulation.
o They retain texture, colour, appearance, flavour and nutritional value.
• Safety ➔ To test the biosafety of these coatings, the coatings were tried on the BHK21 cell line, which is a laboratory
standard for the study of various biological processes.
o The tests showed that the new coating materials were nontoxic and could be safely used as edible food
packaging materials.

23. FIBERISATION
Why in News?
In the backdrop of the 5G spectrum auction for telecom airwaves by government, the challenge for requisite
infrastructure in terms of fiberisation challenge is underscored.
About

• Fiberisation is the process of connecting radio towers


with each other via optical fibre cables.
• It represents the part of the network that connects the
core of the network to the edge facilitating large
amounts of data used in 5G services.
• Also, it provides additional bandwidth and stronger
backhaul support.
• In India, currently only 33% of the towers are fiberised,
compared to the 80%-90% in the U.S., Japan and China.
• To transition to 5G, India needs at least 16 times more fibre.

24. BATTERY CERTIFICATION AN QUALITY CONTROL


Why in News?
Amid increasing instances of fires in EVs, Centre has formed a panel to formulate procedures on battery certification and
quality control.
About

• Reasons for fire may include manufacturing defects, external damage, faulty charging or faults in the deployment
in the battery management system, which could result in these batteries becoming a fire risk.
• With growing concern over climate change and a larger aim to reduce its crude import bill, government’s focus is
on pushing passenger vehicle owners to shift to EVs.
• Different types of batteries are available to power EVs such as Li-ion, Solid state, Nickel-Metal Hydride etc.
• However, Li-ion batteries are most preferred because of their high power-to-weight ratio, high energy efficiency,
low self-discharge etc.

Steps Taken for Battery Safety

• Bureau of Indian Standard (BIS) formulated performance standards for EV Batteries.


• Constitution of a Probe Committee to look into fire incidents reasons.

53 | P a g e
• Regenerative braking to increase its range by recovering energy.
• Re-use of EV batteries in factories and home energy battery storage systems.

25. LUX-ZEPLIN (LZ) DARK MATTER DETECTOR


Why in News?
A test run of LZ detector in U.S. has shown it to be the most sensitive dark matter detector yet created.
About

• Visible universe—including Earth, sun, other stars, and galaxies—is made of protons, neutrons, and electrons
bundled together into atoms.
• This ordinary, also called baryonic, matter makes up less than 5 percent of the mass of universe.
• Rest of the universe appears to be made of a mysterious, invisible substance called dark matter (25 percent) and a
force that repels gravity known as dark energy (70 percent).
• Unlike normal matter, dark matter does not interact with electromagnetic force. This means it does not absorb,
reflect or emit light, making it extremely hard to spot.
• Scientists study dark matter by looking at the effects it has on visible objects.

Significance of Dark matter study

• It may account for unexplained motions of stars within galaxies.


• Critical to understanding of evolution of universe and to emergence of stars, planets etc.
• Will help to determine if the universe is open (continues to expand), closed (expands to a point and then collapses)
or flat (expands and then stops when it reaches equilibrium).

About LUX-ZEPLIN (LZ) detector

• It is designed (underground) to capture dark matter in the form of weakly interacting massive particles (WIMPs).
• It consists of a huge titanium tank filled with extremely pure liquid xenon.
• Centre of LZ is one of the purest places on Earth (free of radiation and dust).
• Collaboration of scientists/institutions from U.S., U.K., Portugal, and Korea.

54 | P a g e
Geography

1. Loktak lake

Why in News?
Loktak Lake Authority of Manipur recently issued a notice to remove all floating houses and fishing structures on Loktak
lake.
About

• It’s located about 40 kilometres south of Imphal.


• It's the largest freshwater lake in Northeast India, the pristine Loktak Lake is one of the most popular tourist
attractions in Manipur. 
• Known for its floating circular swamps, which are called phumdis in the local tongue, 
• The lake invites tourists from far and wide for its ethereal beauty. 
• These swamps look almost like islands and are a mass of soil, organic matter, and vegetation. 
• The lake houses the only floating national park in the world, the Keibul Lamjao National Park, which is the last
refuge of the endangered brow-antlered deer or sangai, Manipur's state animal.
•  In addition, the lake shelters about 230 species of aquatic plants, 100 types of birds, and 400 species of fauna like
barking deer, sambar, and Indian python.
• Loktak lake was initially designated as a wetland of international importance under the Ramsar Convention in 1990.
• Later it was also listed under the Montreux Record in 1993.

2. Odesa Port
Why in News?
Following the Supreme Court verdict (2018), upholding the notification of the elephant corridor in the Sigur plateau,
there has been a surge in resorts and commercial establishments mushrooming in the areas.

About

• Odessa Sea Port (UAODS Port) is the largest Ukrainian seaport and one of
the largest ports in the Black Sea basin.
• Its total annual traffic capacity is 40 million tonnes.
• The types of vessels regularly calling at Odessa are Bulk Carrier (100%).
• Along with its younger satellite ports of Chornomorsk (1958) and Yuzhne
(1973), port of Odessa is a major freight and passenger transportation
hub of Ukraine.
• Odesa is the base of a fishing fleet as well as the chief operational hub of
the Ukrainian Navy. Odesa is also an important cultural and educational
centre.

3. Hasdeo Aranya region


Why in News?
Chhattisgarh Legislative Assembly has unanimously passed a private member resolution urging the Centre to cancel the
allocation of all coal mining blocks in the ecologically sensitive Hasdeo Aranya area.

About

• The Hasdeo Aranya forests are called the lungs of Chhattisgarh.

55 | P a g e
• The Hasdeo Aranya (Aranya means forest) lies in the catchment area of the Hasdeo river in North-Central
Chhattisgarh.
• The Hasdeo river is a tributary of the Mahanadi river which originates in Chhattisgarh and flows through Odisha into
the Bay of Bengal.
• The Hasdeo forests are also the catchment area for the Hasdeo Bango Dam built
across the Hasdeo river which irrigates six lakh acres of land, crucial to a State
with paddy as its main crop.
• Besides, the forests are ecologically sensitive due to the rich biodiversity they
offer and due to the presence of a large migratory corridor for elephants.

👉 Private member resolution

• An MLA who is not a Minister — whether he or she happens to be from the


ruling party or not— is a private member.
• A private member resolution can be brought in by a private member and if
passed, it becomes an expression of what the House thinks. This is different
from a private member bill which would become law in case of approval.
• Such private member resolutions were passed by the State Assemblies of Punjab and Kerala during the farm law
agitation where both state legislatures had expressed their displeasure against the then proposed (now withdrawn)
farm laws.

4. Indian Ocean Dipole (IOD)


Why in News?
A negative Indian Ocean dipole event has been declared, increasing the chances of above-average rainfall over much of
Australia.
About

• It is defined by the difference in sea surface temperature between two areas (or poles, hence a dipole) – a western
pole in the Arabian Sea (western Indian Ocean) and an eastern pole in the eastern Indian Ocean south of Indonesia.
• In scientific terms, the IOD is a coupled ocean and atmosphere phenomenon, similar to ENSO but in the equatorial
Indian Ocean.
• It is thought that the IOD has a link with ENSO events through an extension of the Walker Circulation to the west
and associated Indonesian throughflow (the flow of warm tropical ocean water from the Pacific into the Indian
Ocean).
• The IOD affects the climate of Australia and other countries that surround the Indian Ocean Basin, and is a significant
contributor to rainfall variability in this region.
• Positive event linked to it ➔
o Warmer sea surface temperatures in the western Indian Ocean relative to the east
o Easterly wind anomalies across the Indian Ocean and less cloudiness to Australia's northwest
o Less rainfall over southern Australia and the Top End.
• Negative event ➔
o Cooler sea surface temperatures in the western Indian Ocean relative to the east
o Winds become more westerly, bringing increased cloudiness to Australia's northwest
o More rainfall in the Top End and southern Australia

5. Nicaragua
Why in News?

56 | P a g e
Earlier this month Nicaragua shuttered seven radio stations belonging to the Catholic Church and launched an
investigation into the bishop of Matagalpa, Monsignor Rolando Álvarez, accusing him of inciting violent actors “to carry
out acts of hate against the population.”
About

• Nicaragua is the largest country in the Central American isthmus, bordered by Honduras to the northwest, the
Caribbean to the east, Costa Rica to the south, and the Pacific Ocean to the southwest.
• Managua is the country's capital and largest city.
• As of 2015, it was estimated to be the second largest city in Central America.

6. Rhine River
Why in News?
Water levels on the Rhine river are very low because of unusually hot and dry weather, preventing many vessels from
navigating the critical European shipping route fully loaded.
About

• It is a waterway of western Europe culturally and historically one


of the great rivers of the continent and among the most important
arteries of industrial transport in the world.
• It rises in two headstreams high in the Swiss Alps.
• It flows from two small headways in the Alps of east-central
Switzerland north and west to the North Sea, into which it drains
through the Netherlands.
• It is a major route for products ranging from grains to chemicals and
coal.
• It is an important link between industrial producers and global
export terminals in North Sea ports such as Rotterdam and
Amsterdam, while canals and other rivers link the Rhine to the
Danube, making it possible to ship to the Black Sea as well.

7. Mahanadi River
Why in News?
The Indian Meteorological Department (IMD) has forecast a heavy rainfall causing the flood situation in the Mahanadi
River, Odisha.

57 | P a g e
About

• Origin ➔ It rises from a place near Sihawa in Bastar hills in the state of
Chhattisgarh to the south of Amarkantak.
• The Mahanadi River system is the third largest of peninsular India after
Godavari and Krishna, and the largest river of Odisha state.
• The catchment area of the river extends to Chhattisgarh, Madhya Pradesh,
Odisha, Jharkhand and Maharashtra.
• Its basin is bounded by the Central India hills on the north, by the Eastern
Ghats on the south and east and by the Maikala range in the west.
• Major Tributaries ➔ The Seonath, the Hasdeo, the Mand and the Ib joins
Mahanadi from left whereas the Ong, the Tel and the Jonk joins it from right.
• Major Dams/Projects on Mahanadi ➔
o Hirakud Dam: This is the longest dam of India.
o Ravishankar Sagar, Dudhawa Reservoir, Sondur Reservoir, Hasdeo Bango and Tandula are other major
projects.
• Urban Centres ➔ Three important urban centres in the basin are Raipur, Durg and Cuttack.

👉 India Meteorological Department (IMD)

• IMD was established in 1875.


• It is an agency of the Ministry of Earth Sciences.
• It is the principal agency responsible for meteorological observations, weather forecasting and seismology.

8. Lake Garda
Why in News?
Italy’s worst drought in decades has reduced Lake Garda to near its lowest level ever
recorded.
About

• Lake Garda is also known as Benaco. It is the largest lake in Italy.


• Why has Lake Garda reached its lowest level? ➔ Northern Italy saw significantly
lower rainfall for months and snowfall in 2022 was also down 70%, drying up
important rivers like the Po, which flows across Italy’s agricultural and industrial
heartland.
• The parched condition of the Po, Italy’s longest river, caused billions of euros in
losses to farmers who normally rely on it to irrigate fields and rice paddies.
• To compensate for the loss, authorities allowed more water from Lake Garda to
flow out to local rivers.
• But in late July 2022, they reduced the amount to protect the lake and the financially important tourism tied to it.

9. Godavari River
Why in News?
Officials issued the second warning with the flood level crossing 50 feet in Godavari River at Bhadrachalam, Telangana,
and the flow in the river crossing the 13-lakh cusecs mark.

58 | P a g e
About

• Origin ➔ Godavari River rises from Trimbakeshwar near Nasik in Maharashtra


and flows for a length of about 1465 km before outfalling into the Bay of Bengal.
• The Godavari is the largest Peninsular River system. It is also called the Dakshin
Ganga.
• The basin is bounded on the north by the Satmala hills, on the south by the
Ajanta range and the Mahadeo hills, on the east by the Eastern Ghats and on
the west by the Western Ghats.
• Drainage Basin ➔ The Godavari basin extends over states of Maharashtra,
Telangana, Andhra Pradesh, Chhattisgarh and Odisha in addition to smaller
parts in Madhya Pradesh, Karnataka and Union territory of Puducherry.
• Tributaries ➔ Pravara, Purna, Manjra, Penganga, Wardha, Wainganga, Pranhita (combined flow of Wainganga,
Penganga, Wardha), Indravati, Maner and the Sabri.
o The Pravara, Manjira and Maner are right bank tributaries.
o The Purna, Pranhita, Indravathi and Sabari are important left bank tributaries
• Cultural Significance ➔ Kumbh Mela also takes place on the banks of the Godavari River in Nashik.
• Urban Centers ➔ Nagpur, Aurangabad, Nashik, Rajhmundry.
• Important Projects on Godavari ➔ Polavaram Irrigation Project, Kaleshwaram, Sadarmatt Anicut, Inchampalli
project, Sriram Sagar Project (SRSP).

10. Paraguay
Why in News?
External Affairs Minister Dr S Jaishankar has unveiled a bust of Mahatma
Gandhi in Asuncion, Paraguay. The Minister also visited the historic Casa de
la Independencia, from where Paraguay’s Independence movement started
more than two centuries ago.
About

• Paraguay is a landlocked country in South America.


• It is bordered by Argentina to the south and southwest, Brazil to the
east and northeast, and Bolivia to the northwest.
• It is a founding member of Mercosur, the United Nations, the
Organization of American States, the Non-Aligned Movement and the
Lima Group.
• Asunción is the capital and the largest city of Paraguay.

11. Tonga Volcano


Why in News?
Recently, the experts said that the Tonga volcano wave was 9 times taller than the tsunami in Japan.
About

• Location ➔ The volcano is located approximately 70 km from the Tongan capital Nuku'alofa; this distance
significantly minimizes its destructive power.
• 90 metres high ➔ The initial tsunami wave created by Tonga’s underwater Hunga Tonga volcano eruption in 2022
was 90 metres high.

59 | P a g e
• Tsar Bomba ➔ The Tonga volcano eruption unleashed more energy than the Tsar Bomba, the most powerful
nuclear device ever detonated.

👉 Tonga

• It is a Polynesian country and also an archipelago consisting of 171


islands of which 45 are inhabited.
• Located in Oceania, Tonga is an archipelago in the South Pacific
Ocean.
• Boundaries ➔ It is surrounded by Fiji and Wallis and Futuna
(France) to the northwest; Samoa to the northeast; New Caledonia
(France) and Vanuatu to the west; Niue (the nearest foreign
territory) to the east; and Kermadec (New Zealand) to the
southwest.
• Climate ➔ Tonga has a tropical rainforest climate.

12. Poyang Lake


Why in News?
Normally surrounded by water in August, an island in China's Poyang Lake is now fully visible, testifying to the dramatic
impact of a long drought and heatwave on a vital part of the country's irrigation infrastructure.
About

• Poyang Lake or Poyang Hu is China's biggest freshwater lake. It is a


shallow lake.
• It is known as the "kidney" because of the role it plays in regulating the
flow of the Yangtze River in central Jiangxi province.
• Poyang Lake is a key flood outlet for the Yangtze River, which overflows
during summer and can cause extensive damage to crops and property.
In winter, the lake’s water flows back out into the river.
• It is a national nature reserve, which is a rest stop for over 300 species
of migratory birds, including the critically endangered Siberian crane.
• It is home to the endangered Yangtze River or finless porpoise.
• Poyang's hydrological functions have been eroded over the years by
sand mining and the construction of the Three Gorges and other large-scale dams upstream.
• The island that is now fully visible is known as Luoxing Pier.

13. Anang Tal Lake


Why in News?
The US State Department released a Joint Statement of the 2022 Supply Chain Ministerial Forum, focusing on the global
challenge which includes – the COVID-19 pandemic, wars and conflicts and climate change.
About

• It is located in South Delhi. The lake is situated to the north of Jog Maya temple and approximately 500 metres to
the northwest of Qutub Complex.
• The lake is claimed to have been built by a Tomar King, Anangpal II in 1060 AD.
• The lake is said to have been a place of a general resort, but now it is dried up and used for cultivation.
• It is also said Alauddin Khalji, in 1296-1316 AD, utilised the water of this tank when he built (Qutub) minar and
extended the Qutub-ul-Islam mosque.

60 | P a g e
👉 Anangpal Tomar

• Anangpal II was popularly known as Anangpal Tomar. He belonged to the Tomar dynasty. He was the founder of
Dhillika Puri, which eventually became Delhi.
• This is revealed by the stone inscriptions excavated by Lord Cunningham. The inscriptions and coins suggest that
Anangpal Tomar was the ruler of present-day Delhi and Haryana between the 8th-12th centuries.
• He was succeeded by his grandson Prithviraj Chauhan.

14. Tigray
Why in News?
The Director-General of the World Health Organisation has said that racism is the reason behind a lack of international
interest in Ethiopia’s war-stricken northern Tigray region, where the civilians are living in dire conditions.
About

• Tigray is Ethiopia's northernmost region.


• Bordering Eritrea, it is home to most of the country's estimated 7 million ethnic Tigrayans.
• The ethnic group, which accounts for about 6% of Ethiopia's population, have had an outsized influence in national
affairs.

15. Pileus Cloud


Why in News?
Haikou city residents in China were astounded when they observed a rainbow in the sky that was shaped like a cloud.
The phenomena was not a rainbow, but actually a cloud known as pileus cloud.
About

• What is it? ➔ The ‘pileus cloud’ is also known as a ‘cap cloud’ or ‘scarf cloud’.
• Actually, the rainbow-like pileus cloud is not a cloud in itself.
• The pileus cloud is an accessory cloud of small horizontal extent, in the form of a cap or hood above the top or
attached to the upper part of a cumuliform cloud that often penetrates it.
• Several pileus may fairly often be observed in superposition.
• Formation ➔ A pileus is created when the quickly rising air in the cumulus cloud (or cumulonimbus) updraft pushes
against the cooler air above it.
o This causes condensation of the moisture right along the top of the updraft, leading to the formation of the
Pileus.
o When the sun is shining at a perfect angle, diffraction of light takes place between the droplets and ice
crystals in the cloud, giving the Pileus its rainbow-like colors.
o But, these clouds typically only last a few hours, as they are "eaten up" by the cloud that is above them.
o Pileus cloud is absorbed by the lower cloud by convection.
o Pileus isn't very common, but it can be spotted on days when you could anticipate cumulus clouds
developing into thunderstorms.
• Pileus cloud should not be confused with the lenticular cloud. Lenticular clouds stand alone while pileus clouds are
an accessory to cumuliform clouds.

👉 Lenticular Clouds

• Lenticular clouds are stationary clouds that form mostly in the troposphere, typically in parallel alignment to the
wind direction.
• These strange clouds sometimes form downwind of hills or mountains.
• Lenticular clouds are so named because they resemble lentils.
61 | P a g e
• They are easily distinguished from other cloud species by their distinctive UFO shape and smooth edges.

16. Puga Valley


Why in News?
The magistrate of Nyoma subdivision in Ladakh ordered Oil and Natural Gas Corporation (ONGC) to practise the highest
standards of precaution at the geothermal energy plant in Puga Valley.
About

• Puga Valley is situated in the Changthang Valley in the southeastern part of Ladakh, about 22 km away from the
Salt Lake Valley.
• The region is known as a hub for generating geothermal energy, which is notably in the form of sulphur and borax
deposits.
• It has shown an immense amount of mud pools and hot sulphur springs.
• These mud pools and springs are said to have medicinal properties, and can cure skin diseases and even
rheumatism.
• One can also spot sheep, yak, bar-headed geese, brahminy ducks, black-necked cranes and more wildlife here.
• The Puga Valley Geothermal Project will be India’s first geothermal energy project, and will be the world’s highest
geothermal energy project.

62 | P a g e
Polity
1. Electoral Bonds

Why in News?
State Bank of India (SBI) shared data reporting that Donations to political parties through electoral bonds (EBs) have
crossed the Rs 10,000-crore mark.
About

• State Bank of India is authorised to issue and encash these bonds.


• Electoral bonds are purchased anonymously by donors and are valid for 15 days from the date of issue.
• As debt instruments, these can be bought by donors from a bank, and the political party can then encash them.
• These can be redeemed only by an eligible party by depositing the same in its designated account maintained with
a bank.
• The bonds are issued by SBI in denominations of Rs 1,000, Rs 10,000, Rs 1 lakh, Rs 10 lakh and Rs 1 crore.
• The bonds are available for purchase by any citizen of India for a period of ten days each in the months of January,
April, July and October as may be specified by the Central Government.
• Eligibility ➔ Only the political parties registered under Section 29A of the Representation of the People Act, 1951
and have secured not less than 1% of the votes polled in the last general election to the House of the People or the
Legislative Assembly, are eligible to receive electoral bonds.

2. Centralized Public Grievances Redress and Monitoring System


Why in News?
The timeline for redressal of grievances has been reduced from 45 days to 30 days.
About

• CPGRAM is an online web-enabled system developed by the National Informatics Centre (Ministry of Electronics &
IT [MeitY]), in association with the Directorate of Public Grievances (DPG) and Department of Administrative Reforms
and Public Grievances (DARPG) (under Ministry of Personnel, Public Grievances & Pensions)
• It helps in receiving, redressing and monitoring of grievance of the public in an efficient way.

3. Waqf Properties
Why in News?
Ministry of Minority Affairs is implementing various schemes to protect Waqf properties.
About

• Waqf is any movable or immovable property given in the name of God for religious and charitable purposes by a
person professing Islam.
• A waqf can be formed through a deed or instrument, or a property can be deemed waqf if it has been used for
religious or charitable purposes for a long period of time.
• A person creating the waqf cannot take back the property and the waqf would be a continuing entity.

👉 Schemes

• Shahari Waqf Sampatti Vikas Yojana (SWSVY) is a Central Sector Scheme, under which the Ministry of Minority
Affairs provides Grant-in-Aid to Central Waqf Council (CWC).

63 | P a g e
o Under the scheme, interest free loans are provided to State Waqf Boards/ Waqf (SWB) Institutions for
construction of economically viable buildings on the urban Waqf land such as marriage halls, hospitals, cold
storages etc.
• Qaumi Waqf Board Taraqqiati Scheme (QWBTS) - Financial Assistance is provided to SWBs towards computerization,
digitization of records of Waqf properties and GIS Mapping of Waqf properties.

👉 Central Waqf Council (CWC)

• It is a statutory body under the administration control of the Ministry of Minority Affairs.
• It was setup in 1964 as per the provision given in the Waqf Act, 1954.
• It acts as an advisory Body to the Central Government on matters concerning the working of the Waqf Boards and
Auqaf.
• The council consists of chairman and members not exceeding 20 in number and appointed by the Government of
India.
• Union Minister in charge of Waqf is the ex-officio chairman of the council.

4. Central Vigilance Commission


Why in News?
Vigilance Commissioner Suresh N. Patel was sworn in as the Central Vigilance Commissioner (CVC) by President Droupadi
Murmu at the Rashtrapati Bhavan.
About

• It was set up by the Government of India (Ministry of Home Affairs) vide Resolution in 1964 on the recommendation
of the Santhanam Committee.
• It is an apex body for the prevention of corruption and exercising general superintendence over vigilance
administration.
• The Parliament enacted Central Vigilance Commission Act, 2003 (CVC Act) conferring statutory status on the CVC.
• It is an independent body which is only responsible to the Parliament.
• It submits its report to the President of India.
• Functions ➔
o Exercise superintendence over the functioning of the Delhi Special Police Establishment (CBI) insofar as it
relates to the investigation of offences under the Prevention of Corruption Act, 1988.
o CVC has no investigation wing of its own as it depends on the CBI and the Chief Vigilance Officers (CVO) of
central organizations, while CBI has its own investigation wing drawing its powers from Delhi Special Police
Establishment Act, 1946.
• Composition ➔ It is a multi-member Commission consisting of a Central Vigilance Commissioner (Chairperson) and
not more than 2 Vigilance Commissioners (Member).
• Appointment of Commissioners ➔ They are appointed by the President of India on the recommendations of a
committee consisting of Prime Minister, Union Home Minister and Leader of the Opposition in Lok Sabha (if there
is no Leader of Opposition then the leader of the single largest Opposition party in the Lok Sabha).
• Term ➔ Their term is 4 years or 65 years, whichever is earlier.
• Removal ➔ Removal is done by the President on grounds of bankruptcy, unsound mind, infirmity of body or mind,
sentenced to imprisonment for a crime, or engages in paid employment or has acquired financial or other interest
that might affect his judgment.
o He can also be removed for proved misbehaviour or incapacity if the Supreme Court inquiry finds him guilty.
o They can also resign by writing to the President.

5. MP’s privileges on arrests

64 | P a g e
Why in News?
Rajya Sabha Chairman has clarified that members of Parliament do not enjoy any immunity from arrest in criminal cases
when the House is in session, and they cannot avoid summons issued by law enforcement agencies.
About

• Constitutional Provision ➔ Under Article 105 of the Constitution, members of Parliament enjoy certain privileges,
so they can perform their duties without any hindrances.
• One of the privileges is that a member of Parliament cannot be arrested in a civil case 40 days before the
commencement of the session or committee meeting and 40 days thereafter. This privilege is already incorporated
under Section 135A of the Civil Procedure Code,1908.
• However, in criminal matters, members of Parliament are not on a different footing than a common citizen. That
means members of Parliament do not enjoy any immunity from being arrested in a criminal case during the session
or otherwise.

👉 Supreme Court judgements on this issue

• K Anandan Nambiar Case ➔ In this, the Court held that the true Constitutional position is that so far as a valid order
of detention is concerned, a Member of Parliament can claim no special status higher than that of an ordinary citizen
and is as much liable to be arrested, detained or questioned even during the Session.
• State of Kerala Vs K. Ajith and Others ➔ In this case, the Supreme Court observed that privileges and immunities
are not gateways to claim exemptions from the general law of the land, particularly in the case of criminal law
which governs the action of every citizen.

6. Pharmacopoeia Commission for Indian Medicine & Homoeopathy


Why in News?
The Government of India has established the Pharmacopoeia Commission for Indian Medicine & Homoeopathy
(PCIM&H) as a subordinate office under the Ministry of Ayush.

About

• What is it? ➔ PCIM&H is an autonomous body under the aegis of Ministry of Ayush, established since 2010.
• Pharmacopoeia is an officially recognized book of standards as per the Drugs and Cosmetics Act, 1940 and Rules
1945 thereunder.
• Functions ➔
o The Commission is engaged in development of Pharmacopoial Standards for Ayurvedic, Unani, Siddha &
Homeopathic drugs.
o PCIM&H is also acting as Central Drug Testing cum Appellate Laboratory for Indian systems of Medicine &
Homoeopathy.
• Benefits of Merger with PLIM & HPL ➔
o Optimum use of infrastructural facilities, technical manpower and financial resources of the three
organizations for enhancing their standardised outcomes.
o Focused and cohesive development of standards of AYUSH drugs and publication of pharmacopoeias and
formularies.

👉 Ayurveda, Siddha and Unani Drugs Technical Advisory Board

• ASUDTAB is a statutory body under the provisions of Drugs and Cosmetics Act, 1940.
• It advises the central and state governments in regulatory matters of Accelerated Shelf-Life Testing (ASLT) drugs.
• ASLT is an indirect method of measuring and estimating the stability of a product by storing the product under
controlled conditions that increase the rate of degradation occurring in the product under normal storage conditions.

65 | P a g e
7. Foreigners’ Tribunals
Why in News?
The Gauhati High Court has asked the Centre and the Assam government to decide collectively whether the ministerial
staff for 200 additional Foreigners’ Tribunals would be appointed.
About

• Foreigners’ Tribunals (FT) are quasi-judicial bodies established as per the Foreigners’ Tribunal Order, 1964 and the
Foreigners’ Act, 1946.
• It is unique to Assam (due to Assam’s National Register of Citizens).
• Establishment ➔ In the beginning, the powers to constitute tribunals were vested only with the Centre.
▪ The Ministry of Home Affairs (MHA) has amended the 1964 Order to empower district magistrates in all
States and UTs to set up tribunals to decide whether a person staying illegally in India is a foreigner or not.
• Powers of Tribunal ➔ According to the Foreigners’ Tribunal Order, 1964 the Foreigners’ Tribunal,
▪ Shall have the powers of a civil court while trying a suit under the Code of Civil Procedure, 1908.
▪ Can summon and ask for the attendance of any person and examine him/her on oath.
▪ Can ask anyone to produce the required documents,
▪ Can commission examining any witness, as and when required.
• Working ➔ Under the provisions of Foreigners’ Act, 1946 and Foreigners Tribunal Order, 1964, only Foreigner
Tribunals have the right to declare a person as a foreigner.
▪ The FT is for those who have been left out in the final NRC list or have been marked as ‘D’ (‘doubtful’). The
ones falling under this category have the right to appeal to the Foreigners Tribunal.
▪ Thus, non-inclusion of a person’s name in the NRC does not by itself amount to him/her being declared a
foreigner.
▪ However, if declared a foreigner or placed under the doubtful category ‘the burden of proof lies with the
accused’.
▪ If a person allegedly fails to prove their citizenship before the Foreigners’ Tribunal after the State police’s
Border wing marks him or her as an illegal immigrant, they will be marked as a declared foreigner (DF).

8. Enhanced Access and Service Excellence


Why in News?
Similar to the Enhanced Access and Service Excellence (EASE) for Public Sector Banks (PSBs), the Centre has asked the
Indian Banks’ Association (IBA) to prepare a viability plan for Regional Rural Banks (RRBs).
About

• EASENext is well-positioned to channel reforms with a specific focus on customer-centric initiatives and emphasised
on customer-first strategy and focus on employee development.
• The EASENext reforms should bring ease for customers as well as for employees.
• EASENext would comprise 2 major initiatives ➔
o EASE 5.0 (Common reform agenda for Public Sector Banks) and
o Bank specific strategic 3-year roadmap (Based on individual bank's business priorities).
• EASE 5.0 ➔ The Enhanced Access and Service Excellence is a common reform agenda for Public Sector Banks (PSBs)
aimed at institutionalizing clean and smart banking.
o It commits PSBs to tech-enabled, simplified and collaborative banking to further the agenda of customer-centric
digital transformation.
o Under EASE 5.0, PSBs will continue to invest in new-age capabilities and deepen the ongoing reforms to
respond to evolving customer needs, changing competition, and the technology environment.

66 | P a g e
o EASE 5.0 will focus on digital customer experience, and integrated and inclusive banking, with emphasis on
supporting small businesses and agriculture.
• 3-year roadmap ➔ All PSBs will also create a bank-specific three-year strategic roadmap. It will entail strategic
initiatives beyond EASE 5.0.
o The initiatives will be across diverse themes - business growth, profitability, risk, customer service, operations,
and capability building.
• EASE index is prepared by the Indian Banking Association, along with Boston Consulting Group. It is commissioned
by the Union Ministry of Finance.

9. Electricity Amendment Bill,2022


Why in News?
Recently, the government has tabled the Electricity (Amendment) Bill 2022 in the Lok Sabha soon after which it was
referred to the parliamentary standing committee on energy for wider consultation with stakeholders.
About

• The Bill amends the Electricity Act, 2003. The Act regulates the electricity sector in India. It sets up the Central and
State Electricity Regulatory Commissions (CERC and SERCs) to regulate interstate and intrastate matters,
respectively.

👉 Key provisions of the Bill

• Multiple discoms in the same area ➔ More than one power distributor can operate in an area and they will be
allowed to use the power distribution infrastructure of other suppliers. This is aimed at boosting competition and
giving more choice to the consumers.
• Tariffs ➔ The Bill makes provision for “mandatory” fixing of minimum as well as maximum tariff ceilings by the
“appropriate commission” to avoid predatory pricing by power distribution companies and to protect consumers.
• Timely Tariff Revisions ➔ The Bill has several provisions to ensure graded and timely tariff revisions that will help
provide state power utilities with enough cash to be able to make timely payments to power producers.
• Cross-subsidy Balancing Fund ➔ The Bill adds that upon grant of multiple licenses for the same area, the state
government will set up a Cross-subsidy Balancing Fund. Cross-subsidy refers to the arrangement of one consumer
category subsidizing the consumption of another consumer category.
o Any surplus with a distribution licensee on account of cross-subsidy will be deposited into the fund. The fund
will be used to finance deficits in cross-subsidy for other discoms in the same area or any other area.
• Renewable purchase obligation(RPO) ➔ The Act empowers SERCs to specify renewable purchase obligations(RPO)
for discoms. RPO refers to the mandate to procure a certain percentage of electricity from renewable sources.
o The Bill adds that RPO should not be below a minimum percentage prescribed by the central government.
Failure to meet RPO will be punishable with a penalty between 25 paise and 50 paise per kilowatt of the
shortfall.
• Strengthen Regulators ➔ The Bill seeks to strengthen payment security mechanisms and give more powers to
regulators.

10. Criminal Procedure (Identification) Act, 2022


Why in News?
The Criminal Procedure (Identification) Act, 2022 has come into force after being passed by the Parliament in April 2022.
It replaces the Identification of Prisoners Act, 1920.
About

67 | P a g e
• It provides Legal sanction to the police to take physical and biological samples of convicts as well as those accused
of crimes.
• The police as per section 53 or section 53A of the Code of Criminal Procedure (CrPC), 1973, can collect Data.
o Data that can be collected: Finger-impressions, Palm-Print impressions, Footprint impressions,
Photographs, Iris and Retina scan, Physical, Biological samples and their analysis, Behavioural Attributes
including signatures, Handwriting or any other examination
o CrPC is the primary legislation regarding the procedural aspects of criminal law.
• Any person convicted, arrested or detained under any preventive detention law will be required to provide
"measurements" to a police officer or a prison official.
• National Crime Records Bureau (NCRB) will store, preserve, share with any law enforcement agency and destroy
the record of measurements at national level. The records can be stored up to a period of 75 years.
• It aims to ensure the unique identification of those involved with crime and to help investigating agencies solve
cases.

11. Fundamental Duties


Why in News?
Chief Justice of India said Fundamental Duties in the Constitution are not merely to serve a "pedantic or technical"
purpose, but they were incorporated as the key to social transformation.
About

• The idea of Fundamental Duties is inspired from the Constitution of Russia


(erstwhile Soviet Union).
• These were incorporated in Part IV-A of the Constitution by the 42nd
Constitutional Amendment Act, 1976 on the recommendations of the
Swaran Singh Committee.
• Originally 10 in number, one more duty was added through the 86th
Constitutional Amendment Act, 2002.
• All the eleven duties are listed in Article 51-A of the Constitution (the sole
Article in Part-IV-A).
• Like the Directive Principles of State Policy, Fundamental duties are also
non-justiciable in nature.

👉 Supreme Court’s Stand on Fundamental Duties

• The Supreme Court’s Ranganath Mishra judgment 2003 held that


fundamental duties should not only be enforced by legal sanctions but
also by social sanctions.
• In AIIMS Students Union v. AIIMS 2001, it was held by the Supreme Court that fundamental duties are equally
important as fundamental rights.
o Though fundamental duties are not enforceable like fundamental rights they cannot be overlooked as duties in
Part IV A.
o They are prefixed by the same word fundamental which was prefixed by the founding fathers of the Constitution
to ‘right’ in Part III.

12. Bal Aadhar


Why in News?
Over 7.9 million children aged up to five were enrolled under the Unique Identification Authority’s Bal Aadhaar initiative
from April to July.

68 | P a g e
About

• Baal Aadhaar is a blue-coloured variant of the Aadhaar Card. It is issued for children below the age of 5 years.
• How is Bal Aadhar different from Aadhar Card?
• Biometrics to establish uniqueness for the Bal Aadhar is not collected unlike in the case of Aadhaar. A facial image
of a child is instead taken for enrolment.
• Moreover, biometric authentication of the parent/guardian and a proof of relationship document, preferably a
birth certificate, are collected at the time of enrolment.
• What happens to the Baal Aadhar after the children turn five?
o On attaining the age of five, the child is required to furnish his or her biometrics at an Aadhaar Seva Kendra
to complete a process called mandatory biometric update(MBU).
o The MBU process goes through a de-duplication process. After completion of this process, the child is issued
a normal Aadhaar without any change in the Aadhaar number.

13. Legal Aid Defence Counsel(LADC) system


Why in News?
NALSA Executive Chairman has launched the Legal Aid Defence Counsel (LADC) system in 365 districts in 22 states across
India.
About

• Launched by ➔ National Legal Services Authority (NALSA).


• Purpose ➔ To provide free legal aid to poor people facing criminal cases to defend themselves during trial.
• Significance ➔
o In India, where 70% of the population is below the poverty line, the number of cases with NALSA, especially on
the criminal side, is only about 10-12%.
o The rest of the marginalized population was engaging private counsels even at the cost of losing their assets –
selling their jewellery and mortgaging their houses.
o Hence, to extend legal aid to these marginalized populations, the LADC system has been launched.

👉 National Legal Services Authority (NALSA)

• The National Legal Services Authority (NALSA) has been constituted under the Legal Services Authorities Act, 1987
to provide free Legal Services to the weaker sections of the society and to organize Lok Adalats for amicable
settlement of disputes. The Chief Justice of India is the Patron-in-Chief.
• In every State, State Legal Services Authority has been constituted to give effect to the policies and directions of
the NALSA and to give free legal services to the people and conduct Lok Adalats in the State. The State Legal Services
Authority is headed by Hon’ble the Chief Justice of the respective High Court who is the Patron-in-Chief of the State
Legal Services Authority.

14. Benami Law of 1988


Why in News?
Recently, the Supreme Court struck down one of the provisions of the Benami Transactions (Prohibition) Act of 1988
which provides for the punishment of a maximum jail term of three years or a fine or both for those indulging in benami
transactions.
About

• In 2016, the Government of India enacted the Benami Transactions (Prohibition) Amendment Act of 2016.
• The law amended the original Benami Act of 1988.

69 | P a g e
• Section 3 (2) introduced through the amendment mandates a punishment of three years imprisonment for those
who have entered into Benami transactions between September 5, 1988, and October 25, 2016. That is, a person
can be sent behind bars for a Benami transaction entered into 28 years before the section even came into existence.
• Section 5 said that any property which is the subject matter of a benami transaction shall be liable to be confiscated
by the Central Government.
• In 2019, the Calcutta High Court ruled that the Benami Transactions (Prohibition) Amendment Act, 2016 cannot be
applied retrospectively. The Central government then appealed to the Supreme Court against this judgement.

👉 Supreme Court verdict on these amendments

• The Supreme Court has declared Sections 3(2) and 5 introduced through this amendment as unconstitutional and
held that these provisions cannot be applied retrospectively.
• The court also held that the amendment also violated Article 20(1) of the Constitution. Article 20(1) mandates that
no person should be convicted of an offence which was not in force “at the time of the commission of the act charged
as an offence”.
• The court dismissed the government’s version that forfeiture, acquisition and confiscation of property under the
2016 Act was not in the nature of prosecution and cannot be restricted under Article 20.

15. Floor Test


Why in News?
Bihar Chief Minister has won the floor test in the Assembly.
About

• A floor test is a measure to check whether the executive is enjoying the confidence of the legislature.
• It is a constitutional mechanism under which a Chief Minister appointed by the Governor can be asked to prove
majority on the floor of the Legislative Assembly of the state.
• How is it conducted? ➔
o As per the Constitution, the Chief Minister is appointed by the Governor of the state.
o When a single party secures the majority of the seats in the house, the Governor appoints the leader of the
party as the Chief Minister.
o In case the majority is questioned, the leader of the party which claims majority has to move a vote of
confidence and prove majority among those present and voting.
o The Chief Minister has to resign if they fail to prove their majority in the house.
o This happens both in the parliament and the state legislative assemblies.
o In situations when there are differences within a coalition government, the Governor can ask the Chief Minister
to prove majority in the house.
• What is composite floor test? ➔
o There is another test, Composite Floor Test, which is conducted only when more than one person stakes claim
to form the government.
o When the majority is not clear, the governor might call for a special session to see who has the majority.
o The majority is counted based on those present and voting. This can also be done through a voice vote where
the member can respond orally or through division voting.
o Some legislators may be absent or choose not to vote.
o In division vote, voting can be done through electronic gadgets, ballots or slips.
o The person who has the majority will form the government. In case of tie, the speaker can also cast his vote.
• Governors’ discretion ➔ When no party gets a clear majority, the governor can use his discretion in the selection
of chief ministerial candidate to prove the majority as soon as possible.

16. Competition (Amendment) Bill, 2022

70 | P a g e
Why in News?
The Competition (Amendment) Bill, 2022 was introduced in the Lok Sabha. The Bill seeks to amend the Competition Act,
2002.
About

• Regulation of combinations based on deal value ➔ Any acquisition, merger or amalgamation may constitute a
combination. The act currently says parties indulging in merger, acquisition, or amalgamation need to notify the
Commission of the combination only on the basis of ‘asset’ or ‘turnover’. The Bill proposes to add a ‘deal value’
threshold.
• Time limit for approval of combinations ➔ The Act specifies that any combination shall not come into effect until
the CCI has passed an order or 210 days have passed from the day when an application for approval was filed,
whichever is earlier. The Bill reduces the time limit in the latter case to 150 days.
• Gun Jumping ➔ Parties should not go ahead with a combination prior to its approval. If the combining parties close
a notified transaction before the approval, or have consummated a reportable transaction without bringing it to the
Commission’s knowledge, it is seen as gun-jumping.
o The penalty for gun-jumping was a total of 1% of the asset or turnover. This is now proposed to be 1% of
the deal value.
• Addresses the issue of the Hub and Spoke Model ➔ A Hub-and-Spoke arrangement is a kind of cartelisation in
which vertically related players act as a hub and place horizontal restrictions on suppliers or retailers (spokes).
o Currently, the prohibition on anti-competitive agreements only covers entities with similar trades that
engage in anti-competitive practices.
o To combat this, the Bill broadens the scope of ‘anti-competitive agreements’ to catch entities that facilitate
cartelisation even if they are not engaged in identical trade practices.
• Leniency Plus ➔ The Bill allows the commission to give an additional waiver of penalties to an applicant who
discloses the existence of another cartel in an unrelated market, provided the information enables the Commission
to form a prima facie opinion about the existence of the cartel.
• Settlement and Commitment in anti-competitive proceedings ➔ Under the Act, CCI may initiate proceedings against
enterprises on grounds of: (i) entering into anti-competitive agreements, or (ii) abuse of dominant position.
• The Bill permits CCI to close inquiry proceedings if the enterprise offers ➔ (i) settlement (may involve payment), or
(ii) commitments (may be structural or behavioral in nature).
• Appointment of Director General ➔ The Act empowers the central government to appoint a Director General to
CCI. The Bill amends this to empower the CCI to appoint the Director General, with prior approval of the
government.
• Penal Provisions ➔ The penalties and penalty guidelines are proposed to be amended.
o For any false information filed, a penalty of five crores will be imposed, and for failure to comply with the
Commission directions, a penalty of ₹10 crores will be imposed.
o For an appeal to be heard by the National Company Law Tribunal(NCLT) against the Commission’s order,
the party will have to deposit 25% of the penalty amount.

17. Constitutional Bench of the Supreme Court


Why in News?
Justice U U Lalit(49th Chief Justice of India) assured there will be at least one Constitution Bench functioning throughout
the year in the Supreme Court.
About

• What is it? ➔ A Constitution Bench is a bench of the Supreme Court having five or more judges on it.
• These benches are not a routine phenomenon.

71 | P a g e
• A vast majority of cases before the Supreme Court are heard and decided by a bench of two judges (called a Division
Bench), and sometimes of three.

👉 Circumstances for Constitutional Bench to Exist:

• Article 145(3) ➔ Article 145(3) provides, “The minimum number of Judges who are to sit for the purpose of deciding
any case involving a substantial question of law as to the interpretation of this Constitution or for the purpose of
hearing any reference under Article 143 shall be five.”
• Article 143 ➔
o When the President seeks the Supreme Court’s opinion under law under Article 143 of the Constitution.
o As per the provision, the President of India has the power to address questions to the Supreme Court,
which he deems important for public welfare.
o The Supreme Court upon reference advises the President by answering the query. However, such referral
advice by the apex court is not binding on the President, nor is it ‘law declared by the Supreme Court’.
• Conflicting Judgments ➔
o When two or more three-judge benches of the Supreme Court have delivered conflicting judgments on the
same point of law, necessitating a definite understanding and interpretation of the law by a larger bench.
o The Constitution benches are set up on ad hoc basis as and when the above-mentioned conditions exist.

18. Supreme Court Judges Rules


Why in News?
Recently, the Centre amended the Supreme Court Judges Rules.
About

• Chauffeurs and domestic help ➔ It provides chauffeurs and domestic help for retired Chief Justices of India and
Supreme Court judges for their entire lifetime.
• Secretarial assistants ➔ Retired CJIs would also get secretarial assistants. The staff would be paid the salary and
allowances of regular employees of the Supreme Court.
• Security cover ➔ The benefit of 24-hour security cover has been extended to five years for retired Chief Justices
and three years for retired judges of the Supreme Court.
• Bills ➔ Former CJIs and retired judges of the top court can get their monthly mobile phone and Internet bills
reimbursed to the extent of 4,200.
• Rent-free Type VII accommodation ➔ A retired CJI is also entitled to a rent-free Type VII accommodation, other
than the designated official residence, in New Delhi for six months immediately after retirement.
• Ceremonial lounges of airports ➔ The amended Rules mandate that retired Chief Justices and judges should be
extended courtesies as per protocol at ceremonial lounges of airports.
• Exceptions ➔ These post-retirement benefits would be available only if the retirees were not getting similar
facilities from any High Court or government body.

19. TELE-LAW SERVICE TO BE MADE FREE OF COST FOR CITIZENS FROM 2022
Why in News?
A Memorandum of Understanding has been signed between Department of Justice and National Legal Services Authority
(NALSA) on Integrated Delivery of Legal Services.
About

• As per the agreement, NALSA will provide services of 700 lawyers, in each district exclusively for Tele-Law program.
• These empanelled lawyers would also act as referral lawyers and assist in strengthening the mechanism for dispute
avoidance and dispute resolution at the pre-litigation stage.

72 | P a g e
• Launched in 2017 by the Department of Justice (Ministry of Law & Justice), Tele-Law initiative is a reliable and
efficient e-interface and pre-litigation tool.
• Tele–Law mainstreams legal aid to the marginalized seeking legal help by connecting them with the Panel Lawyers
through the tele/video-conferencing infrastructure available at Common Service Centres (CSCs) across 1 lakh Gram
Panchayats.

20. PLACES OF WORSHIP ACT


Why in News?
SC has refused a petition by a sect of Jain community filed under Article 32 of Constitution to enforce Places of Worship
(Special Provisions) Act 1991 against alleged conversion of its religious places by another sect.
About

• Act 1991 prohibits conversion of any place of worship and to provide for maintenance of religious character of any
place of worship as it existed on 15th day of August, 1947.

Key provisions of Act

• Section 3: Bars conversion, in full or part, of a place of worship of any religious denomination into a place of worship
of a different religious denomination — or even a different segment of same the religious denomination.
• Section 4(2): Any suit or legal proceeding with respect to conversion of religious character of any place of worship
existing on August 15, 1947, pending before any court, shall abate — and no fresh suit or legal proceedings shall be
instituted.

Significance

• Through it, the state has enforced a constitutional commitment and operationalised its constitutional obligations
to uphold equality of all religions and secularism.
• Preserves non-retrogression as an essential feature of India’s secular values i.e, there must not be any regression of
rights.

73 | P a g e
International Relational
1. Chabahar Port

Why in News?
Union Shipping Minister Sarbanand Sonowal inaugurated the Chabahar Day conference in Mumbai.
About

• Chabahar Port is a seaport in Chabahar located in southeastern Iran, on


the Gulf of Oman.
• It serves as Iran's only oceanic port, and consists of two separate ports
named Shahid Kalantari and Shahid Beheshti, each of which has five
berths.
• The Chabahar port is a key pillar of India's Indo-Pacific vision to connect
with Eurasia with Indian Ocean Region. The port will also be part of the
International North South Transport Corridor network connecting
India.
• Iran has given special incentives to increase trade cooperation activities
between India and Iran through Chabahar port.
• The Chabahar port is considered a gateway to golden opportunities for
trade by India, Iran and Afghanistan with central Asian countries.

👉 International North-South Transport Corridor

• The International North-South Transport Corridor


(INSTC) project was originally decided between
India, Iran, and Russia in 2000 in St Petersburg, and
subsequently included 10 other central Asian and
west Asian countries:
o Azerbaijan Armenia, Kazakhstan, Kyrgyz
Republic, Tajikistan, Turkey, Ukraine,
Belarus, Oman, Syria, and Bulgaria are
observers. 
o Pakistan, Turkmenistan and Afghanistan
are not party to the INSTC agreement but
are interested in using the transport
corridor.
• It envisions a 7,200-km-long multi-mode network
of ship, rail, and road routes for transporting freight, aimed at reducing the carriage cost between India and Russia
by about 30% and bringing down the transit time from 40 days by more than half.
• The route primarily involves moving freight from India, Iran, Azerbaijan, and Russia. 
• Objective ➔ The objective of the corridor is to increase trade connectivity between major cities such as Mumbai,
Moscow, Tehran, Baku, Astrakhan, etc.
• Significance ➔  
• It would be provided as a viable and fairer alternative to China’s Belt and Road Initiative (BRI).
• Further, it will enhance regional connectivity.

2. Mineral Security Partnership


Why in News?
74 | P a g e
India is working through its diplomatic channels to fetch an entry into the Mineral Security Partnership.
About

• What is it? ➔ It is a US-led partnership to secure supply chains of critical minerals, aimed at reducing dependency
on China.
• Aim ➔ To catalyze investment from governments and the private sector to develop strategic opportunities.
• Members ➔ US, Australia, Canada, Finland, France, Germany, Japan, the Republic of Korea, Sweden, United
Kingdom, and the European Commission.
• Significance of the alliance ➔ The MSP will focus on the supply chains of minerals such as Cobalt, Nickel, Lithium,
and also the 17 ‘rare earth’ minerals.
• The alliance is also seen as primarily focused on evolving as an alternative to China, which has created processing
infrastructure in rare earth minerals and has acquired mines in Africa for elements such as Cobalt.

👉 Why is India not part of MSP?

• According to experts, the reason India would not have found a place in the MSP grouping is because India does not
bring any expertise to the table. In the group, countries like Australia and Canada have reserves and also the
technology to extract them, and countries like Japan have the technology to process REEs.

👉 Rare Earth Metals

• Rare earth elements (REEs) are a group of seventeen chemically similar metallic elements on the periodic table. It
comprises 15 lanthanides elements (lanthanum to lutetium), plus scandium and yttrium. REEs are classified as light
RE elements (LREE) and heavy RE elements (HREE).
• Some REEs are available in India — such as Lanthanum, Cerium, Neodymium, Praseodymium and Samarium, etc.
• Others such as Dysprosium, Terbium, and Europium, which are classified as HREEs, are not available in Indian
deposits in extractable quantities.
• Hence, there is a dependence on countries such as China for HREEs, which is one of the leading producers of REEs
with an estimated 70% share of the global production.

3. Audio Visual Co-production Treaty


Why in News?
The Union Cabinet has approved the signing of an Audio Visual Co-production Treaty between India and Australia, which
is aimed at facilitating joint production of films between the two countries.
About

• Audio visual co-production treaties are enabling documents which facilitate the co-production of films between
both countries.
• Under such umbrella agreements, private, quasi-government or governmental agencies enter into contracts to
produce films together.
• India has so far signed 15 audio-visual co-production treaties with other countries.
• According to the co-production treaty, the respective contributions of the producers of the two countries may vary
from 20% to 80% of the final total cost of the jointly produced work.
• Significance of this treaty with Australia ➔
o The treaty will boost ties with Australia, lead to an exchange of art and culture and showcase the soft power
of our country.
o This will also generate employment among artistic, technical as well as non-technical personnel engaged in
audio-visual co-production, including production and post-production work.

4. Universal Postal Union

75 | P a g e
Why in News?
Recently, The Union Cabinet chaired by the Prime Minister has approved the ratification of amendments to the
Constitution of the Universal Postal Union(UPU).
About

• It was established by Treaty of Bern in 1874.


• Purpose ➔ It is the primary forum for cooperation between postal sector players. It helps to ensure a truly universal
network of up-to-date products and services.
• Headquarters ➔ Bern, Switzerland.
• Significance ➔ It is a United Nations specialized agency. It is also the second-oldest international organization
worldwide.
• Members ➔ Any member country of the United Nations may become a member of the UPU.
o Any non-member country of the United Nations may become a UPU member provided that its request is
approved by at least two-thirds of the member countries of the UPU.
o The UPU now has 192 member countries. India has been a member of the UPU since 1876.

👉 Amendments approved by the Government of India

• The 27th UPU Congress held at Abidjan (Cote d’Ivoire) adopted several amendments to the constitution of UPU.
• These amendments ensure further legal clarity and stability to the Acts of the Union, resolve many long-standing
discrepancies in the text and accommodate provisions for ‘acceptance or approval’ of the Acts in consistency with
Vienna Conventions on Law of Treaties, 1969.

5. UNMOGIP
Why in News?
The United Nations (UN) has appointed Rear Admiral Guillermo Pablo Rios of Argentina as the Head of Mission and Chief
Military Observer for the United Nations Military Observer Group in India and Pakistan (UNMOGIP).
About

• The first group of UN military observers arrived in the mission area on 24 January of 1949 to supervise the ceasefire
between India and Pakistan in the Indian state of Jammu and Kashmir.
• These observers, under the command of the Military Adviser appointed by the UN Secretary-General, formed the
nucleus of the United Nations Military Observer Group in India and Pakistan (UNMOGIP).
• Functions of UNMOGIP ➔
o Following renewed hostilities of 1971, UNMOGIP has remained in the area to observe developments
pertaining to the strict observance of the ceasefire of 17 December 1971 and report thereon to the UN
Secretary-General.
o The Karachi Agreement of July 1949 firmed up the role of UN-level military observers and permitted
supervision of the Ceasefire Line established in Jammu and Kashmir.

👉 Indian contentions with UNMOGIP

• India officially maintains that the UNMOGIP’s role was “overtaken” by the Shimla Agreement of 1972 that
established the Line of Control (LoC).
o In the Shimla Agreement, India and Pakistan agreed to move the ceasefire line to the Line of Control and
to resolve their disputes bilaterally, without the intervention of a third party.
o Kashmir and the Pakistan-sponsored terrorism within now is largely an internal matter of India.
• Since 1972 India has not gone to UNMOGIP with complaints against Pakistan.

76 | P a g e
• In 2014, India requested that UNMOGIP cease operations in Kashmir, and the Ministry of External Affairs (MEA)
reiterated in 2017 that UNMOGIP has no mandate to monitor the situation in Kashmir.
• Pakistan, on the other hand, does not accept the Indian argument and continues to seek cooperation from the
UNMOGIP.
• As a result of these divergent policies, Pakistan continues to lodge complaints with the UNMOGIP against alleged
Indian ceasefire violations.

6. International Labour Organization


Why in News?
International Labour Organisation (ILO) has released a report titled “Global Employment Trends for Youth 2022: Investing
in transforming futures for young people”.
About

• Establishment ➔ Established in 1919 by the Treaty of Versailles as an affiliated agency of the League of Nations.
• Became the first affiliated specialized agency of the UN in 1946.
• It is the only tripartite United Nation (UN) agency. It brings together governments, employers and workers of 187
member States (India is a member), to set labour standards, develop policies and devise programmes promoting
decent work for all women and men.
• Received the Nobel Peace Prize in 1969.
• Headquarters ➔ Geneva, Switzerland
• Reports ➔ World of Work Report, World Employment and Social Outlook Trends 2022, World Social Protection
Report, Social Dialogue Report, Global Wage Report.

👉 Findings Related to India

• The youth employment participation rate declined by 0.9 % over the first nine months of 2021 relative to its value
in 2020, while it increased by 2 % for adults over the same time period.
• The situation is particularly severe for very young people aged 15-20 years.
• Indian young women experienced larger relative employment losses than young men in 2021 and 2022.

7. Nagorno-Karabakh dispute
Why in News?
The conflict between Armenia and Azerbaijan over Nagorno-Karabakh has been at the centre of three major wars and
multiple clashes for decades. The recent flare-up began after Azerbaijan claimed that it had captured the territory in
Karabakh in retaliatory action.
About

• Nagorno-Karabakh is a disputed territory


internationally recognized as part of
Azerbaijan but mostly governed by the
Republic of Artsakh, de facto independent
state with an Armenian ethnic majority
backed by neighbouring Armenia.
• Origin of the conflict ➔ Modern-day
Armenia and Azerbaijan became part of the
Soviet Union when it formed in the 1920s.
Nagorno-Karabakh was an ethnic-majority

77 | P a g e
Armenian region, but the Soviets gave control over the area to Azerbaijani authorities.
• It was only as the Soviet Union began to collapse in the late 1980s that Nagorno-Karabakh’s regional parliament
officially voted to become part of Armenia.
• Azerbaijan sought to suppress the separatist movement, while Armenia backed it. This led to full-scale war.
Armenian forces gained control of Nagorno-Karabakh and areas adjacent to it before a Russian-brokered ceasefire
was declared in 1994.
• After that deal, Nagorno-Karabakh remained part of Azerbaijan, but since then has mostly been governed by a
separatist, self-declared republic run by ethnic Armenians and backed by the Armenian government.

8. International Olympiad on Astronomy and Astrophysics


Why in News?
India secured the third position by winning five medals at the 15th International Olympiad on Astronomy and
Astrophysics (IOAA)-2022 held in Kutaisi, Georgia.
About

• It is governed by a set of Statutes, initially derived from those of the International Physics Olympiad, and
subsequently modified (through a voting process described in the statues) to better reflect the needs of the
competition.
• All participants agree to abide by the terms of the Statutes.
• According to the statutes, the governing body of the IOAA organisation is the International Board (IB), sometimes
also called the International Board Meeting.
• It aims to promote the growing interest in Astronomy and related STEM subjects, especially through the general
education of young people
• It also aims to enhance the development of international contacts among different countries in promoting
Astronomy and Astrophysics in schools.

9. Supply Chain Ministerial Forum


Why in News?
The US State Department released a Joint Statement of the 2022 Supply Chain Ministerial Forum, focusing on the global
challenge which includes – the COVID-19 pandemic, wars and conflicts and climate change.
About

• It is hosted by United States.


• Aim ➔ To advance work to reduce and end near-term supply chain disruptions and collaborate to build supply
chain resiliency to avoid future disruptions.
• Partner Countries ➔ Australia, Brazil, Canada, the Democratic Republic of the Congo, European Union, France,
Germany, India, Indonesia, Italy, Japan, Mexico, Netherlands, Republic of Korea, Singapore, Spain, United Kingdom
and the United States.

👉 Statement issued by the Supply Chain Ministerial Forum

• The shocks to global supply chains came from pandemics, wars and conflicts, extreme climate impacts and natural
disasters.
• To address these disruptions, the world needs to further strengthen supply chains, end near-term disruptions and
build long-term resilience.
• For this, all the partner countries aim to follow various global supply chain principles including transparency,
diversification, security and sustainability.

78 | P a g e
10. International Driving Permit
Why in News?
Recently, The Ministry of Road Transport and Highways has issued a notification for greater facilitation of citizens in the
issue of International Driving Permit (IDP) across the country.
About

• International Driving Permit (IDP) or International Driving Licence (IDL) is accepted in over 150 countries.
• It is an official travel document that is authorised by the United Nations issued to tourists travelling to foreign
countries.
• It is an official translation of your driver's license issued in your home country.
• The document essentially states that you possess a valid driver's license.
• It translates your license documents into many languages so authorities abroad are able to verify and understand
your driving credentials.
• The document is issued from various zonal offices. The validity of this permit is for one year.
• The applicants are required to get the licence from the country visiting within one-year period.
• India’s Status ➔ India is a signatory to Convention on International Road Traffic of 1949 (Geneva Convention).
o So, India is required to issue International Driving Permit as provided under this Convention, for the
acceptance of the same on reciprocal basis with other countries.

79 | P a g e
Society
1. Initiatives to mark 2 years of NEP 2020

Why in News?
The Government of India has launched several initiatives on the second anniversary of the National Education Policy(NEP)
2020.

About

• Kalashala initiative ➔ It will be launched in 750 schools. It envisions educating children about the various art forms
of India & help them discover the country’s rich cultural heritage. The school students will learn about various art
forms through lecture demonstrations by a visiting artist.
• National innovation and entrepreneurship policy for schools in order to promote a learning environment where
creativity, ideation, innovation, problem-solving and entrepreneurship skills of students are nurtured, irrespective
of their age.
• Bharatiya Games Initiative ➔ Under this, the Government will introduce 75 Bharatiya games like ‘gilli danda’,
‘kabaddi’ in schools under the Indian Knowledge Systems(IKS) initiative.

👉 Indian Knowledge Systems(IKS)

• Indian Knowledge System(IKS) is an innovative cell under Ministry of Education(MoE) at AICTE, New Delhi.
• Purpose ➔ It is established to promote interdisciplinary research on all aspects of IKS. Such as preserving and
disseminating IKS for further research and societal applications.
• Activities ➔ It will actively engage in spreading the rich heritage of our country and traditional knowledge in the
field of Arts and literature, Agriculture, Basic Sciences, Engineering & Technology, Architecture, Management,
Economics etc.

2. School Innovation Council


Why in News?
The School Innovation Council was launched and has been introduced to all schools of all the states.
About

• The School Innovation Council (SIC) is an initiative taken by The Ministry of Education’s Innovation Cell (MIC) and
The All India Council for Technical Education (AICTE).
• SIC is a council of teachers, students, and experts from industry and academia to conduct year-round activities for
students and teachers on Innovation and Entrepreneurship.
• SIC will enable mindset change, awareness, and training on Ideation, Innovation and Entrepreneurship, design
thinking, Intellectual Property Rights, start-up finance, and HR among School teachers and students.
• SIC will promote out of box thinking in school education as envisioned in National Innovation and National Education
Policy 2020.
• It will also enable the ranking system for schools on the level of innovation-oriented activities.
• To implement the SIC council in all schools across the nation, SIC portal has been developed where schools can
register themselves.
• SIC will link schools with Innovation Councils established by MIC at Higher Education Level to further provide
exposure to school students.

👉 School Innovation Ambassador Training Program

80 | P a g e
• School Innovation Ambassador Training program (SIATP) was launched to strengthen the mentoring capacity of
teachers for cultivating and handholding innovative and ingenious ideas from students.
• The SIATP program has been conceptualised by MIC and AICTE to train the school teachers across country.
• Under SIATP, teachers undergo 72 hours of training, and those who qualify all the following five modules with a
minimum 50% as passing marks are recognized as “Innovation Ambassadors”.
o Design Thinking & Innovation;
o Idea generation & Idea hand-holding;
o Finance/Sales/HR;
o Intellectual Property Rights (IPR);
o Entrepreneurship and Prototype/ Product Development.
• The “Innovation Ambassadors” are competent to nurture the young school students on Ideation, IPR, product
development, design thinking, problem-solving, critical thinking and skills of entrepreneurship.

3. Experiential Learning for the 21st Century Programme


Why in News?
The National Education Society for Tribal Students (NESTS), Ministry of Tribal Affairs and Central Board of Secondary
Education (CBSE) in collaboration with TATA Trusts and others have launched “Experiential Learning for 21st Century
Programme”.
About

• Experiential Learning for the 21st Century Programme has been launched for the Eklavya Model Residential
School(EMRS) Principals and Teachers.
• Purpose ➔ It is conceptualized as an online programme for educators i.e teachers and principals to help them adapt
classroom learning to real-life experiences.
• The program was offered free of cost to all the selected teachers and principals. The selected teachers were trained
as “Teacher Leaders” who shall be espousing the Experiential Learning Pedagogy to all EMRS teaching fraternity in
a phased manner.

👉 Eklavya Model Residential Schools

• The Ministry of Tribal Affairs in the year 1997-98 launched the scheme.
• Aim ➔ To impart quality education to ST children in remote areas. It will also enable them to avail opportunities in
high and professional educational courses. And further get employment in various sectors.
• Grants ➔ The government of India provides grants for the construction of schools and recurring expenses to the
State Governments under Article 275 (1) of the Constitution.
• Target ➔ Every block with more than 50% ST population and at least 20,000 tribal persons, will have an Eklavya
Model Residential School by the year 2022.

👉 National Education Society for Tribal Students(NESTS)

• NESTS is an autonomous organization established under the Ministry of Tribal Affairs(MoTA).


• Aim ➔ To provide high-quality education to the tribal students in their own environment.

4. National Leprosy Eradication Programme


Why in News?
The Government is implementing the National Leprosy Eradication Programme (NLEP) with the goal of making India
leprosy free.
About

81 | P a g e
• Status ➔ Centrally Sponsored Scheme.
• Nodal Ministry ➔ Ministry of Health and Family Welfare.
• Nodal Mission ➔ National Health Mission(NHM).
• Aim ➔ To detect the cases of leprosy at an early stage and to provide complete treatment, free of cost in order to
prevent the occurrence of disability.

👉 Major Initiatives under the mission

• Leprosy Case Detection Campaign(LCDC) in high endemic districts.


• Leprosy Awareness Campaign on 30th January every year.
• NIKUSTH – A real-time leprosy reporting software implemented across India.
• Strengthening of Disability Prevention and Medical Rehabilitation (DPMR) services.
• Certificates, awards and performance-linked financial incentives are given to the districts for achieving leprosy
elimination.
• Contact tracing is done and Post Exposure Prophylaxis(PEP) with a Single dose of Rifampicin (SDR) is given to the
eligible contacts of the index case in order to interrupt the chain of transmission.

👉 India’s achievements in Leprosy eradication

• India has achieved the elimination of leprosy as a public health problem as per WHO criteria of less than 1 case per
10,000 population at the National level in 2005. However, there are few districts within the States where leprosy is
still endemic.

5. SMILE-75 Initiative
Why in News?
The Ministry of Social Justice & Empowerment has launched the “SMILE-75 Initiative”.
About

• Launched by ➔ Ministry of Social Justice & Empowerment under the SMILE: Support for Marginalized Individuals
for Livelihood and Enterprise Scheme.
• Aim ➔ To make cities/towns and municipal areas begging-free and make a strategy for comprehensive
rehabilitation of the persons engaged in the act of begging through the coordinated action of various stakeholders.
• Under the initiative, seventy-five (75) Municipal Corporations in collaboration with NGOs and other stakeholders
will cover several comprehensive welfare measures for persons who are engaged in the act of begging.
• These measures will focus extensively on rehabilitation, provision of medical facilities, counselling, awareness,
education, skill development, economic linkages and convergence with other Government welfare programmes etc.

👉 SMILE scheme

• ‘Support for Marginalised Individuals for Livelihood and Enterprise’ (SMILE) scheme is set to provide welfare and
rehabilitation to the transgender community and the people engaged in the act of begging.
• This umbrella scheme includes two sub-schemes ➔
o Central Sector Scheme for Comprehensive Rehabilitation for Welfare of Transgender Persons and
o Central Sector Scheme for Comprehensive Rehabilitation of persons engaged in the act of Begging’.

6. PMAY-Urban scheme
Why in News?
The Union Cabinet approved an extension to the Pradhan Mantri Awas Yojana-Urban up to December 31, 2024 so that
the houses sanctioned under the scheme can be completed.

82 | P a g e
About

• The PMAY-U, a flagship Mission of GoI being implemented by Ministry of Housing and Urban Affairs (MoHUA), was
launched on 25th June 2015.
• Aim ➔ To provide all-weather pucca houses to all eligible beneficiaries in the urban areas of the country through
States/UTs/Central Nodal Agencies.
• Coverage ➔ The scheme covers the entire urban area of the country, i.e., all statutory towns as per Census 2011
and towns notified subsequently, including Notified Planning/ Development Areas.
• Verticals ➔ The scheme is being implemented through four verticals:
o Beneficiary Linked Construction (BLC) → Subsidy for beneficiary-led individual house construction
/enhancement.
o Credit Linked Subsidy Scheme (CLSS) → Promotion of affordable housing for weaker sections through credit
subsidy.
o Affordable Housing in Partnership (AHP) → Creation of houses by both public and private sectors.
o In-Situ Slum Redevelopment (ISSR) → Rehabilitation of slum dwellers with the participation of private
developers using land as a resource.
• Features of the Scheme ➔
o Type → It is a Centrally Sponsored Scheme (CSS), except for Credit Linked Subsidy Scheme (CLSS), which is
a Central Sector Scheme.
o The Mission promotes women’s empowerment by providing the ownership of houses in the name of a
female member or in a joint name.

7. Paalan 1000 Campaign


Why in News?
The Union Minister of State for Health has virtually launched the Paalan 1000 National Campaign and Parenting App to
reduce child mortality.
About

• Focus ➔ Paalan 1000 campaign focuses on the cognitive development of children in the first two years of their life.
• Features ➔ The app will provide practical advice to caregivers on what they can do in their everyday routine and
will help clear doubts.
• The app combines coaching for parents, families and other caregivers with services designed to meet the family’s
basic needs.
• Significance ➔ The campaign is aligned with the mission of the Rashtriya Bal Swasthya Karyakram(RBSK) which
emphasizes responsive care and focused interventions during the first 1,000 days of a child.

8. PEN-PLUS Strategy
Why in News?
The African health ministers had adopted the PEN-PLUS strategy against non-communicable diseases.
About

• In Africa, the NCDs are mostly treated in the major cities. This exacerbates health inequities, as it puts care out of
reach for most rural, peri-urban and low-income patients.
• To overcome this Africa has adopted a new strategy called PEN-PLUS.
• The PEN-PLUS strategy is a Regional Strategy to address Severe NCDs at the First-Level Referral Health Facilities by
increasing the access to the essential medicines, technologies, diagnostics and treatment.
• This will bridge the access gap in treatment, as NCDs account for most of the out-of-pocket spending of patients in
Africa.
83 | P a g e
• The strategy urges countries to put in place standardised programmes to tackle chronic and serious NCDs by
ensuring accessibility in district hospitals.
• The PEN-PLUS strategy has been successfully implemented and scaled up in Liberia, Malawi and Rwanda.

9. Nikshay Poshan Yojna


Why in News?
Only two-thirds of people living with tuberculosis benefitted from the Union government’s Nikshay Poshan Yojana (NPY),
sole nutrition support scheme, in 2021, which raises major public health concern.
About

• The NPY was launched in 2018 by the Ministry of Health and Family Welfare.
• It aims to support every Tuberculosis (TB) Patient by providing a Direct Benefit Transfer (DBT) of Rs 500 per month
for nutritional needs.
• Since its inception around Rs 1,488 crore has been paid to 5.73 million notified beneficiaries.
• Performance ➔
o As per India TB Report 2022, only 62.1 % of 2.1 million notified cases across the country received at least
one payment in 2021.
o In Delhi, which has the highest burden of all forms of TB at 747 cases per 100,000 people, only 30.2 % of
patients have got at least one DBT.

👉 Tuberculosis (TB)

• TB is caused by a bacterium called Mycobacterium tuberculosis, belonging to the Mycobacteriaceae family


consisting of about 200 members.
• In humans, TB most commonly affects the lungs (pulmonary TB), but it can also affect other organs (extra-pulmonary
TB).
• Transmission ➔ TB is spread from person to person through the air. When people with lung TB cough, sneeze or
spit, they propel the TB germs into the air.
• Symptoms ➔ Common symptoms of active lung TB are cough with sputum and blood at times, chest pains,
weakness, weight loss, fever and night sweats.
• Treatment ➔ TB is a treatable and curable disease. It is treated with a standard 6-month course of 4 antimicrobial
drugs that are provided with information, supervision and support to the patient by a health worker or trained
volunteer.

10. Aashwasan Campaign


Why in News?
The Ministry of Tribal Affairs and TB division of the Ministry of Health and Family Welfare has organized a national
conclave at the National Tribal Research Institute (NTRI) to disseminate the learnings of the 100-day Aashwasan
Campaign under ‘Tribal TB Initiative’.
About

• Organized by ➔ Ministry of Tribal Affairs and Ministry of Health and Family Welfare under ‘Tribal TB Initiative’.
• Under the campaign, door-to-door screening for TB was undertaken in more than 68,000 villages.
• Based on this campaign, the government has now zeroed in on 75 tribal districts, where a three-pronged strategy
will be implemented to make these districts TB-free. The three-pronged strategy is ➔
o Generating demand for TB services through continued engagement with community influencers
o Improving the delivery of TB services by enhancing the TB testing and diagnosis infrastructure.

84 | P a g e
o Decreasing the risk of transmission and decreasing the pool of infections through active case-finding
campaigns.

👉 What is Tribal TB Initiative?

• Tribal TB is a joint initiative of the Ministry of Tribal Affairs and Central TB Division, Ministry of Health, supported
by USAID as a technical partner and Piramal Swasthya as implementing partner.
• Aim ➔ To improve the cascade of TB care and support services among Tribal Populations in India.

11. National Forensic Science University


Why in News?
Recently, the Union Home and Cooperation Minister addressed the First convocation of National Forensic Science
University (NFSU).
About MTP Act

• Establishment ➔ It was established by the Government of India in 2020 with the objective of fulfilling the acute
shortage against the increasing demand for forensic experts in the country and around the world.
• The National Forensic Sciences University, with the status of an Institution of National Importance, is the world’s
first and only University dedicated to Forensic, behavioral, cybersecurity, digital forensics, and allied Sciences.
• Apart from Gujarat, its campuses have been opened in Bhopal, Goa, Tripura, Manipur and Guwahati.
• Vision ➔
o To fulfill the acute shortage of Forensic Experts in the Country and the World.
o To make the World a Better and Safer place to live.
o To carry out Research in the area of Forensic Science, Crime Investigation, Security, Behavioral Science and
Criminology.
• Mission ➔
o Education through Investigation.
o To impart High-Quality Education of International Standards.
• New Centres of Excellence ➔ A new campus and three Centres of Excellence have been added in the University:
o The Centre of Excellence in DNA.
o The Centre of Excellence in Cyber Security.
o The Centre of Excellence in Investigative and Forensic Psychology.

12. SEED Scheme


Why in News?
The Ministry of Social Justice Ministry has so far received 402 online applications from across the country to avail benefits
under Scheme for Economic Empowerment of Denotified/Nomadic /SemiNomadic(SEED).
About

• The scheme is meant for the upliftment of Denotified, Nomadic and Semi-Nomadic Tribes (DNTs/NTs/SNTs).
• Aim ➔ To provide 1) free competitive exam coaching to these students 2) provide health insurance to families 3)
uplift clusters of these communities through livelihood initiatives and 4) provide financial assistance for housing.
• Implementation ➔ The scheme will be implemented through an online portal that will issue a unique ID to each
applicant so they can apply and track the status of their application online.

👉 The De-notified, Nomadic and Semi-Nomadic Tribes

• They are the most neglected, marginalised and economically and socially deprived communities.

85 | P a g e
• Most of them have been living a life of destitution for generations and still continue to do so with an uncertain and
gloomy future.
• They somehow escaped the attention of our developmental framework and thus are deprived of the support unlike
Scheduled Castes and Scheduled Tribes.

86 | P a g e
Security
1. INS Vikrant
Why in News?
India’s first indigenous aircraft carrier Vikrant was handed over to the Indian Navy.
About

• INS Vikrant is India’s first indigenous aircraft carrier(IAC-1).


• Designed by the Indian Navy’s Directorate of Naval Design (DND) and built by Cochin Shipyard Limited(CSL).
• Named After ➔ Vikrant is named after the Majestic-class aircraft carrier, which was operated by the Indian Navy
from 1961 to 1997.
• Features ➔
o It has a top speed of around 28 knots and a cruising speed of 18 knots with an endurance of about 7,500
nautical miles.
o The ship can accommodate an assortment of fixed-wing and rotary aircraft.
o It has over 2,300 compartments, designed for a crew of around 1,700 people, including specialised cabins
to accommodate women officers.
• Significance ➔ With INS Vikrant, the Indian Navy will have two functional aircraft carriers– the other being INS
Vikramaditya.
• Moreover, with this launch, India has joined a select band of countries such as the USA, UK, France, Russia, Italy and
China that have niche capabilities in designing and building an Aircraft Carrier.

2. AL NAJAH-IV
Why in News?
India and Oman will carry out a 13-day military exercise with a focus on counter-terror cooperation.
About

• The fourth edition of India-Oman joint military exercise ‘AL NAJAH-IV’ between contingents of Indian Army and the
Royal Army of Oman is scheduled to take place at the Foreign Training Node of Mahajan Field Firing Ranges from
August 1 to 13.
• A 60-member team from the Royal Army of Oman has arrived at the site. The Indian Army will be represented by
troops from the 18 Mechanised Infantry Battalion.
• The previous edition of the exercise was organised in Muscat in March 2019.

3. Exercise VINBAX 2022


Why in News?
The 3rd Edition of Vietnam India Bilateral Army Exercise “Ex VINBAX 2022” is scheduled to be conducted at Chandimandir,
Haryana.
About

• What is it? ➔ It is a bilateral military exercise between India and Vietnam.


• Purpose ➔ The exercise is commonly defined as the India and Vietnam army’s collaboration, which helps reinforce
the India-Vietnam defence and security cooperation and endorses India-Vietnam relations.
• Theme of the exercise ➔ “Employment and deployment of an Engineer Company and a Medical Team as part of
United Nations Contingent for Peacekeeping Operations”.

👉 India-Vietnam defence relations

87 | P a g e
• India and Vietnam signed a protocol on defence cooperation in 1994. The partnership was thereafter elevated to a
strategic level in 2007.
• A pact on boosting bilateral defence cooperation was also signed in 2009.
• The strategic partnership was elevated to a Comprehensive Strategic Partnership in 2016 and defence cooperation
is a key pillar of this partnership.
• Vietnam is also an important partner in India’s Act East policy and the Indo-Pacific vision.

4. MiG 21
Why in News?
A MiG-21 Bison aircraft of the Indian Air Force (IAF) crashed in Barmer, Rajasthan, killing the two pilots aboard the trainer
version of the fighter aircraft.
About

• The Mikoyan-Gurevich MiG 21 is a supersonic jet fighter and interceptor aircraft, designed by the Mikoyan-Gurevich
Design Bureau in the Soviet Union.
• MiG is a product of the Soviet Union which entered into service in 1959.
• Approximately 60 countries on four continents have flown the MiG-21, and it still serves many nations six decades
after its maiden flight.
• India inducted the MiG-21 in 1963 and got full technology transfer and rights to license-build the aircraft in the
country.
• Russia stopped producing the aircraft in 1985, while India continued operating the upgraded variants.

5. Exercise Pitch Black


Why in News?
Indian Force will join 16 other nations later this month for Exercise Pitch Black, hosted by the Australian Air Force.
About

• Exercise Pitch Black is a biennial exercise hosted by the Australian Air Force.
• Aim ➔ To practice Offensive Counter Air(OCA) and Defensive Counter Air(DCA) combat in a simulated war
environment.
• Participating countries ➔ The participating countries are Australia, Canada, France, Germany, Indonesia, India,
Japan, Malaysia, Netherlands, New Zealand, the Philippines, South Korea, Singapore, Thailand, UAE, U.K and U.S.

👉 Other Exercises:

• Bilateral ➔ Ex AUSTRA HIND (Army), Ex AUSINDEX (Navy).


• Multilateral ➔ Malabar Exercise (With the Navies of USA, India, Australia and Japan).

6. Hellfire R9X Missile


Why in News?
The US military used its ‘secret weapon’ — the Hellfire R9X missile – to kill Al Qaeda chief Ayman al-Zawahiri.
About

• Hellfire R9X missile is also known as AGM-114 R9X or Ninja Missile.


• It is a US-origin missile known to cause minimum collateral damage while engaging individual targets.
• The weapon does not carry a warhead and instead deploys razor-sharp blades at the terminal stage of its attack
trajectory.

88 | P a g e
• This helps it to break through even thick steel sheets and cut down the target using the kinetic energy of its
propulsion without causing any damage to the persons in the general vicinity or to the structure of the building.
• The blades pop out of the missile and cut down the intended target without causing the massive damage to the
surroundings which would be the case with a missile carrying an explosive warhead.

7. Yuan Wang 5
Why in News?
India is closely monitoring the movement of a Chinese “spy ship” that is on its way to Sri Lanka and will dock at its
Hambantota port around August 11.
About

• The ‘Yuan Wang 5’, a Chinese research and survey vessel, is en route to Hambantota, a strategically important deep-
sea port developed mostly using loans from Beijing.
• ‘Yuan Wang’-class ships are used to track satellite, rocket and intercontinental ballistic missile (ICBM) launches.
• China has around seven of these tracking ships that are capable of operating throughout the Pacific, Atlantic, and
Indian oceans. The ships supplement Beijing’s land-based tracking stations.

8. Laser-Guided Anti-Tank Guided Missiles


Why in News?
Defence Research and Development Organisation (DRDO) has successfully test fired indigenously developed Laser-
Guided Anti-Tank Guided Missiles (ATGM).
About

• ATGMs are primarily designed to hit and destroy heavily armored military vehicles.
• The missiles can be transported by a single soldier, to larger tripod-mounted weapons, which require a squad or
team to transport and fire, to vehicle and aircraft mounted missile systems.
• Developed by ➔ Two Pune-based facilities of the DRDO’s Armament and Combat Engineering (ACE) Cluster — the
Armament Research and Development Establishment (ARDE) and High Energy Materials Research
Laboratory(HEMRL) — in association with Instruments Research and Development Establishment (IRDE), Dehradun.
• Range ➔ The missile has the capability to defeat the protected armoured vehicles in ranges from 1.5 to 5 kilometers.
• This type of guided missiles rely on an electro-optical imager (IIR) seeker, a laser or a W-band radar seeker in the
nose of the missile.
• These are ‘fire-and-forget’ missiles where the operator can retreat right after firing as there is no more guidance
required.

👉 Laser-Guided ATGM:

• The all-indigenous Laser Guided ATGM employs a tandem High Explosive Anti-Tank (HEAT) warhead to defeat
Explosive Reactive Armour (ERA) protected armoured vehicles.
• The ATGM has been developed with multi-platform launch capability and is currently undergoing technical
evaluation trials from the 120 mm rifled gun of MBT Arjun.

👉 Some other Anti-tank Missiles

• Helina ➔ It has a maximum range of seven kilometers and has been designed and developed for integration on the
weaponized version of the ALH (Advanced Light Helicopter).
• The Nag ➔ It is a third-generation fire-and-forget missile developed for mechanized formations to engage heavily
fortified enemy tanks.

89 | P a g e
• MPATGM ➔ It stands for Man-Portable Anti-Tank Guided Missile which has a range of 2.5 kilometers, with fire-
and-forget and top attack capabilities for infantry use.
• SANT ➔ It is a Smart Stand-off Anti-Tank Missile being developed for launch from the Mi-35 Helicopter for the Air
Force’s anti-tank operations.

9. Exercise Yudh Abhyas


Why in News?
India and the US will hold the 18th edition of the Exercise “Yudh Abhyas” in Uttarakhand’s Auli.
About

• What is it? ➔ It is a military exercise between India and the US.


• Aim ➔ To enhance understanding, cooperation and interoperability between the armies of India and the US with
a number of complex drills being planned.
• Started in ➔ The exercise was started in 2004 under the US Army Pacific Partnership Program.
• The exercise is hosted alternately between both countries.

👉 Other exercises between India and the US

• Exercise Tiger Triumph ➔ It is a tri-service military exercise between India and the US.
• Exercise Vajra Prahar ➔ It is a Special Forces joint military training exercise. It has been conducted alternately in
India and the US since 2010.
• Exercise Cope India ➔ It is a series of bilateral Air Force exercises between India and the United States.

10. Live-fire exercises


Why in News?
China has begun its live-fire exercise near Taiwan by launching at least 11 ballistic missiles into the country’s coast. This
is being done after the US House speaker visited Taiwan.
About

• They are exercises primarily used by military personnel in which live ammunition is used to create training conditions
that are as close to real combat scenarios as possible.
• During live-fire training, soldiers are placed in simulated combat situations and are given the opportunity to use
their weapons and equipment (like ships, aircraft, tanks and drones).
• Such exercises are invaluable in maintaining the combat readiness of troops, the cohesiveness of units, and
instilling confidence in their ability to use their weapons and equipment correctly.
• The exercises also involve testing the effectiveness of vehicles, weapon platforms and weapons systems so that
any design flaws can be resolved before the weapons are fully operational.

11. Him Drone-a-thon programme


Why in News?
The Indian Army in collaboration with the Drone Federation of India has launched the ‘Him Drone-a-thon’ programme.
About

• It is launched by Indian Army in collaboration with the Drone Federation of India


• Aim ➔ To catalyze and provide focused opportunities to the Indian drone ecosystem to develop path-breaking
drone capabilities for meeting the requirements of frontline troops.

90 | P a g e
• Features ➔ Under the programme, the following categories of drones will be developed initially - 1) Logistics/ Load
carrying Drone in High Altitude Areas, 2) Autonomous Surveillance/ Search & Rescue, 3) Drone and Micro/ Nano
Drones for Fighting in Built Up Areas.
• Activities planned under the programme ➔ The activities planned include a) interactions & ideation between users,
development agencies, academia, b) visits by development agencies to operational locations to understand the
ground perspective & requirements & c) on-ground trials and actual conduct & evaluation of drone products.

12. Exercise Vajra Prahar


Why in News?
The 13th Edition of the Indo-US Joint Special Forces exercise “Ex Vajra Prahar 2022” commenced at the Special Forces
Training School(SFTS), Bakloh in Himachal Pradesh.
About

• Exercise Vajra Prahar is a Special Forces joint military training exercise.


• It takes place alternately in India and the US since 2010.
• Aim ➔ To enable sharing of best practices and experiences in areas such as, joint mission planning capabilities and
operational tactics.
• Significance ➔ The exercise is a significant step in strengthening the traditional bond of friendship between the
special forces of both Nations as well as improve bilateral defence cooperation between India and the USA.

👉 Other Exercises of India with the USA:

• Exercise Yudh Abhyas (Army).


• Cope India (Air Force).
• Red Flag (USA’s multilateral air exercise).
• Malabar Exercise (trilateral naval exercise of India, USA and Japan).

13. Porcupine Strategy


Why in News?
Recently, China launched aggressive and unprecedented military exercises near Taiwan in response to US House Speaker
Nancy Pelosi’s visit to the island that China claims as part of its territory.
About

• The “porcupine doctrine” was proposed in 2008 by US Naval War College research professor William S Murray.
• It is a strategy of asymmetric warfare focused on fortifying a weak state’s defences to exploit the enemy’s
weaknesses rather than taking on its strengths.
• It is about building defences that would ensure that Taiwan could be attacked and damaged but not defeated, at
least without unacceptably high costs and risks.

👉 Three defensive layers in porcupine strategy?

• The outer layer is about intelligence and reconnaissance to ensure defence forces are fully prepared.
• Behind this come plans for guerrilla warfare at sea with aerial support from sophisticated aircraft provided by the
US.
• The innermost layer relies on the geography and demography of the island.

👉 What is the need for such a strategy?

91 | P a g e
• China enjoys overwhelming military superiority over Taiwan. Over the past decade, China has developed far more
accurate and precise weapon systems to target Taiwan and has been vocal about its intention to “reunite” the island
with the mainland, by force or coercion if needed.

14. Advanced Towed Artillery Gun System


Why in News?
In a first, the indigenous Advanced Towed Artillery Gun System (ATAGS) will be used along with the usual 25-Pounders
for the 21-gun salute during the Independence Day celebrations at Red Fort this year.
About

• The Advanced Towed Artillery Gun System (ATAGS) is an indigenous 155mm, 52-calibre artillery gun system.
• It was jointly developed by the Defence Research and Development Organisation (DRDO) in partnership with the
private industry (Bharat Forge and Tata Group).
• The ATAGS has demonstrated a range of over 45 km, and an official termed it as the “most consistent and accurate
gun in the world”.
• It has a capability to program and fire future Long Range Guided Munitions (LRGM) to achieve precision and deep
strike.
• The system is configured with an all-electric drive that will ensure maintenance free and reliable operation over
longer periods of time.

15. Butterfly Mines


Why in News?
The UK Ministry of Defence, in its intelligence assessment of the ongoing war in Ukraine, has and sounded an alarm on
the possible use of PFM-1 series ‘Butterfly Mines’ by the Russian military in Donetsk and Kramatorsk.
About

• The PFM-1 and PFM-1S are two kinds of anti-personnel landmines that are commonly referred to as ‘Butterfly
mines’ or ‘Green Parrots’.
• These names are derived from the shape and colour of the mines.
• The main difference between the PFM-1 and PFM-1S mine is that the latter comes with a self destruction
mechanism which gets activated within one to 40 hours.
• The ‘Butterfly mine’ has earned a reputation for being particularly attractive to children because it looks like a
coloured toy.
• It is very sensitive to touch and just the act of picking it up can set it off. Because of the relatively lesser explosive
packed in this small mine, it often injures and maims the handler rather than killing them.
• These mines are also difficult to detect because they are made of plastic and can evade metal detectors.

👉 Ottawa Convention

• It is the 1997 Convention on the use, stockpiling, production, and transfer of Anti-Personnel landmines (APLs) and
on their Destruction.
• Usually referred to as Anti-Personnel Mine Ban Treaty, this international agreement bans APLs.
• This Convention requires states-parties,
o To destroy their stockpiled APLs within 4 years and
o To eliminate all APL holdings, including mines currently planted in the soil, within 10 years.
• Some key current and past producers and users of landmines, including the US, China, India, Pakistan, and Russia,
have not signed the treaty.

92 | P a g e
16. Garuda Shield
Why in News?
The United States, Indonesia and other countries are conducting joint combat exercises on the Indonesian island of
Sumatra.
About

• Garuda Shield has been an annual military training since 2009.


• Participation of several other countries, including Australia and Japan, makes the 2022 edition the largest ever.
• The exercises encompass army, navy, air force and marine drills.
• It is designed to strengthen capability, interoperability, trust and cooperation in support of a free and open Indo-
Pacific.

17. Exercise Udarashakti


Why in News?
An Indian Air Force (IAF) contingent has left for Malaysia to participate in a bilateral exercise ‘Udarashakti’ with the Royal
Malaysian Air Force (RMAF).
About

• What is it? ➔ It is a bilateral air force exercise between India and Malaysia.
• Purpose ➔ The exercise will give a chance to both countries to share and learn best practices and an opportunity
for both to discuss mutual combat capabilities.
• The exercise will witness different aerial combat exercises between the two air forces to augment security in the
region and to help in strengthening defense cooperation between the two countries.

👉 Other Exercises between India and Malaysia

• Exercise Harimau Shakti ➔ It is a bilateral joint military exercise between India and Malaysia.

18. Dornier Maritime Surveillance Aircraft


Why in News?
India handed over a Dornier maritime surveillance aircraft to the Sri Lanka Navy to further boost bilateral defence
partnership.
About

• Manufacturer by ➔ It is being manufactured by state-run aerospace major Hindustan Aeronautics Limited (HAL).
• Capabilities ➔ Dornier 228 Maritime Patrol Aircraft is a Short Take-Off and Landing (STOL).
• It is a multirole light transport aircraft used by the Indian Navy for electronic warfare missions, maritime
surveillance, and disaster relief systems and has been in production since 1981.
• Significance of the move ➔
o Security requirement: The aircraft is being provided to Sri Lanka from the inventory of the Indian Navy to
help the country meet its immediate security requirement.
o Countering Chinese: The handing over of the Dornier aircraft by India comes a day before Chinese ship 'Yuan
Wang 5' docs at the Southern port of Hambantota.

19. F-INSAS System


Why in News?

93 | P a g e
Defence Minister Rajnath Singh handed over two new weapons, the Nipun mines, the Landing Craft Assault (LCA) and
the F-INSAS system, to the Army.
About F-INSAS System

• Full-Form ➔ F-INSAS stands for Future Infantry Soldier As A System.


• It is a programme for infantry modernisation aimed at enhancing the operational capability of the soldier.
• Under this programme, soldiers are being equipped with modern systems which will be lightweight, all-weather-
all-terrain, economical and would require low maintenance.
• The full gear of the F-INSAS system includes an AK-203 assault rifle and a multimode hand grenade. Apart from this,
soldiers are also provided with ballistic helmets and ballistic goggles for protection against small projectiles and
fragments, along with a bullet-proof vest.
• F-INSAS also comes with hands-free, secured advanced communications set for real-time exchange of information
with the command post and fellow soldiers for enhanced situational awareness.

👉 What are Nipun Mines?

• Nipun mines are indigenously designed and developed anti-personnel mines termed by the DRDO as ‘soft target
blast munition’. These mines are meant to act as the first line of defence against infiltrators and enemy infantry.
• Developed by ➔ Armament Research and Development Establishment, a Pune-based DRDO facility, and Indian
industry.
• Note ➔ Anti-personnel mines are meant to be used against humans as against anti-tank mines that are aimed at
heavy vehicles. They are smaller in size and can be deployed in large numbers.

20. TEJAS Fighter Aircraft


Why in News?
During the meeting, Argentina expressed interest in the Made in India TEJAS fighter aircraft for the Argentine Air Force.
About

• Tejas – meaning ‘radiant’.


• Manufactured by ➔ state-owned Hindustan Aeronautics Limited (HAL).
• HAL developed its first supersonic fighter – HAL HF-24 Marut – in the 1960s and phased them out in 1990. After
that, it developed its supersonic fighter — Tejas.
• Features ➔
o Tejas is a single engined, light weight, highly agile, multi-role supersonic fighter. It has quadruplex digital
fly-by-wire Flight Control System (FCS) with associated advanced flight control laws.
o The aircraft with delta wing is designed for ‘air combat’ and ‘offensive air support’ with ‘reconnaissance’
and ‘anti-ship’ as its secondary roles.
o It can refuel in the air and be ready for war again.
o These aircraft can fly as fast as the speed of sound, i.e., Mach
1.6 to 1.8, up at an altitude of 52,000 feet
• Objectives ➔ To develop LCA for the IAF and simultaneously reduce the
gap in the field of aeronautical technology available in India and the
advanced nations of the West.

👉 Argentina

• Boundaries ➔ It is a country in the southern half of South America.


Southwest: Chile, North: Bolivia and Paraguay, Northeast: Brazil, East:
Uruguay and the South Atlantic Ocean, South: Drake Passage.
• Capital ➔ Buenos Aires.
94 | P a g e
• Geographical Features ➔
o Andes Mountain Range
o Highest point: Aconcagua (It belongs to the Andes mountain range.
o Major rivers: Paraná, Uruguay.
o Islands: Argentina claims sovereignty over a part of Antarctica, the Falkland Islands and South Georgia and
the South Sandwich Islands.
o Monte Desert: In Argentina, a smaller desert above the Patagonian desert.

21. Zorawar
Why in News?
The Indian Army is prioritising the procurement of the indigenous Indian light tank named ‘Zorawar’ for deployment in
the mountains.
About

• The Armoured Fighting Vehicle-Indian Light Tank (AFV-ILT) ‘Zorawar’ will be deployed to outmanoeuvre the
movement of Chinese forces during the standoff in eastern Ladakh.
• It will be designed to operate from High Altitude Area, the marginal terrain to the Island territories and will be highly
transportable for rapid deployment to meet any operational situation.
• It will have equal firepower as the present tank, including missile firing. The power to weight ratio will make it very
agile.

👉 Other Procurements

• The Indian Army intends to procure a new-generation 'Future Tank' platform namely FRCV (Future Ready Combat
Vehicle), in a phased manner. The expected induction timeline is by 2030.
• It will come along with performance-based logistics, transfer of technology, engineering support package and other
maintenance and training requirements.
• Futuristic Infantry Combat Vehicle (FICV) will replace BMP-II.
• Also, Light Armoured Multi-Purpose Vehicle (LAMV) with enhanced mobility and protection for Recce Platoon are
to replace the existing modified Maruti Gypsy.

👉 Swarm Drone Systems

• Indian Army has inducted indigenously produced Swarm Drones Systems aiming to employ them in both offensive
and defensive operations.
• These drones will provide a decisive edge to the tactical commanders employing them.
• Swarm Drones being inducted into the Mechanised Forces will provide an edge to Indian Army in meeting future
security challenges.
• Make-II case ➔ The Indian Army has also initiated a Make-II case, Autonomous Surveillance and Armed Drone
Swarm (A-SADS) which includes a version for the High Altitude Area.

22. Exercise VAJRA PRAHAR


Why in News?
The 13th edition of India-USA Joint Special Forces Exercise VAJRA PRAHAR 2022 culminated at Bakloh in Himachal
Pradesh.
About

• What is it? ➔ Exercise Vajra Prahar is a Special Forces joint military training exercise between India and US.

95 | P a g e
• Aim ➔ To share the best practices and experiences in areas such as joint mission planning and operational tactics
as also to improve interoperability between the Special Forces of both nations.
• The exercise takes place alternately in India and the US since 2010.

👉 Other exercises between India and the US

• Yudh Abhyas ➔ Military exercise.


• Tiger Triumph ➔ Tri-service military exercise.
• Cope India ➔ Bilateral Air Force exercise.

23. INS Sumedha


Why in News?
Recently, INS Sumedha visited Port Klang, Malaysia during her return passage from Australia where she participated in
the Azadi Ka Amrit Mahotsav celebrations.
About

• INS Sumedha is an indigenously built Naval Offshore Patrol Vessel deployed for multiple roles independently and
in support of Fleet Operations.
• She is part of the Indian Navy's Eastern Fleet based at Visakhapatnam and functions under the operational command
of the Flag Officer Commanding-in-Chief, Eastern Naval Command.
• She is the third ship of the indigenous Naval Offshore Patrol Vessel (NOPV) Project to be inducted into the Indian
Navy.
• The ship has been designed and built by Goa Shipyard Limited.
• The primary role ➔ to undertake EEZ surveillance, anti-piracy patrols, fleet support operations, providing maritime
security to offshore assets and carry out escort operations for high value assets.

24. Pegasus Spyware


Why in News?
Recently, The Supreme Court-appointed technical committee found no conclusive evidence on use of the Pegasus
spyware in phones examined by it.
About

• What is it? ➔ Pegasus is the hacking software – or spyware. It has the capability to infect billions of phones running
either iOS or Android operating systems.
• By whom? ➔ It is developed, marketed and licensed to governments around the world by the Israeli company NSO
Group.
• Techniques ➔
• The earliest version of Pegasus discovered in 2016, infected phones through spear-phishing – text messages or
emails that trick a target into clicking on a malicious link.
• Pegasus infections can also be achieved through “zero-click” attacks, which do not require any interaction from the
phone’s owner in order to succeed.
• These will often exploit “zero-day” vulnerabilities, which are flaws in an operating system that the mobile phone’s
manufacturer does not yet know about and so has not been able to fix.
• How can one detect Pegasus Spyware? ➔ Amnesty International researchers have developed a tool called Mobile
Verification Toolkit (MVT) which is aimed to identify if the Pegasus spyware has targeted your phone.

25. F/A-18 Super Hornet


Why in News?
96 | P a g e
Ahead of India commissioning its first Indigenous Aircraft Carrier (IAC) Vikrant, the US aviation major Boeing has
proposed the Indian Navy to choose its F/A 18 Super Hornet fighter jet.
About

• The F/A-18 Super Hornet Block III is the world’s most advanced, combat proven, multi-role frontline naval fighter
that offers unique and differentiated capabilities and full compatibility with Indian Navy carriers.
• It has been designed and built for carrier operations, and is fully compliant with INS Vikramaditya and INS Vikrant
aircraft carrier.
• F/A-18 will be able to operate on the deck, in the hangar and on the lifts of the Indian Navy’s aircraft carriers.
• It will help further the interface between manned and unmanned systems in a carrier environment.
• The Super Hornet’s Precision Landing Mode software is specially designed to reduce pilot workload while landing
on the Indian Navy’s Vikramaditya carrier by maintaining the proper glide slope and proper approach speeds.
• It has fully redundant systems and is independent of the carrier’s optical landing system.
• F/A-18 Super Hornet is in single-seater (E-Variant) and two-seater variant (F-Variant), and both variants are carrier
compatible to perform the full range of combat missions and can fully operate from the carrier deck.
o The two-seater is also a capable trainer aircraft - both ashore and carrier borne.

👉 INS Vikrant

• INS Vikrant is India’s first indigenous aircraft carrier(IAC-1).


• Designed by the Indian Navy’s Directorate of Naval Design (DND) and built by Cochin Shipyard Limited(CSL).
• Named After ➔ Vikrant is named after the Majestic-class aircraft carrier, which was operated by the Indian Navy
from 1961 to 1997.
• Operation Capabilities ➔ It has a maximum designed speed of 28 knots (about 52 km/h) with an endurance of 7500
NM.
o The ship will be capable of operating 30 aircraft including MiG-29K fighter jets, Kamov-31 Air Early Warning
Helicopters, MH-60R Seahawk multi-role helicopters, as well as the Advanced Light Helicopters (ALH), and
the Light Combat Aircraft (LCA).
o Using a novel aircraft-operation mode known as Short Take Off But Arrested Recovery (STOBAR), the IAC
is equipped with a ski-jump for launching aircraft, and a set of three 'arrestor wires’ for their recovery
onboard.

26. Light Combat Helicopters


Why in News?
The Indian Air Force (IAF) is set to formally raise its first unit of indigenous Light Combat Helicopters(LCHs) in Jodhpur in
the first week of October.
About

• Light Combat Helicopter(LCH) is an Indian multi-role attack helicopter designed and manufactured by the Hindustan
Aeronautics Limited(HAL).
• The LCH has been ordered by the Indian Air Force (IAF) and the Indian Army.
• Features ➔ LCH has several stealth features and has armour protection, night attack capability, and crash-worthy
landing gear to give it better survivability.
o It is powered by two Shakti engines and has a maximum take-off weight of 5,800 kg.
o With a maximum speed of 268 km per hour ,it has a range of 550 km.
o Armed with air-to-air and air-to-ground missiles, LCH also has a 20 mm gun and 70 mm rockets.

👉 What are the roles LCH can perform?

97 | P a g e
• The LCH is well suited for anti-tank role wherein it can fly low and fast to attack enemy armour columns and destroy
them.
• It is also suitable for scout roles wherein it can fly ahead of advancing columns of the Army and detect enemy
presence.
• It is also suitable for air defence roles and destruction of enemy air defence assets. It can also be used in urban
warfare missions and combat search and rescue operations.

27. THEATRE COMMANDS


Why in News?
Minister of Defence announced the setting up of ‘Joint Theatre Commands’ (JTC) of the tri-services.
About Theatre Commands

• Theaterisation or Theatre Command means putting specific number of personnel from the three services —army,
navy and air force— under a common commander in a specified geographical territory.
• At present, the Indian Army, navy and air force
each has multiple commands that are vertically
split in terms of their command structure.
• The army and air force have seven commands
each while the navy has three commands.
• Idea of Theatre commands has been proposed
by both the Kargil review committee as well as
D B Shekatkar committee.
• While the details are not immediately clear,
reports say that single service commands that currently exist would be combined into just 4 geographical
commands.
• Also, Air space and cyber war would be the 5th and 6th theatre commands.

Other such steps in Defence Modernisation:

• Creation of Chief of Defence Staff (CDS)


• Proposal for Integrated Battle Groups (IBG)
• Launching Agnipath Scheme.

98 | P a g e
History and Culture
1. Pingali Venkayya

Why in News?
The Ministry of Culture organized “TirangaUtsav” to celebrate the contributions of Pingali Venkayya to the nation on the
occasion of his 146th Birth Anniversary.
About Pingali Venkayya

• Pingali Venkayya was a freedom fighter, designer of India’s National Flag and a follower of Gandhian principles.
• He was born on 2nd August 1876 into a Telugu Brahmin family in Bhatlapenumarru, Madras Presidency, British India
(today’s Machilipatnam in Andhra Pradesh).
• He was also known as Patti Venkayya because of his research into Cambodia Cotton. Patti means ‘cotton’, which
was very important for Machilipatnam, a former port city that became famous for its Kalamkari handloom weaves.

👉 Contribution to National Flag

• Venkayya designed several models of national flag starting from the one designed in 1921 which was approved by
Mahatma Gandhi at a Congress meeting in Vijayawada.
• This version consisted of two red and green bands; the two bands represented the two major religious communities
— the Hindus and the Muslims. The flag also had a charkha which represented Swaraj. On the advice of Mahatma
Gandhi, Venkayya added a white band which represented peace.
• In 1931, concerns were raised about the religious aspect of the flag. Keeping that in mind, a Flag Committee was set
up. The committee replaced the red with saffron and changed the order of the colours with saffron on top followed
by white and then green. The charkha was placed on the white band in the middle.
• The colours now stood for qualities and not communities; the saffron for courage and sacrifice, white for truth and
peace and green for faith and strength. The charkha stood for the welfare of the masses.
• Post-Independence, a national flag committee under President Rajendra Prasad replaced the charkha with the
Ashok Chakra.

2. Shrimad Rajchandra Mission


Why in News?
The Prime Minister of India inaugurated and laid the foundation stone of various projects of the Shrimad Rajchandra
Mission worth more than 300 crore rupees at Dharampur in Gujarat.
About

• Shrimad Rajchandra Mission Dharampur is a spiritual movement for inner transformation through wisdom,
meditation and selfless service.
• Founded by Pujya Gurudevshri Rakeshji, the organisation works through 196 centres in five continents.

👉 Shrimad Rajchandraji

• He was a self-realised saint, a reformer of Jainism, and a remarkable poet-philosopher of the late 19th century.
• He gave the world a rich heritage that guides generations of seekers.
• He was a visionary who laid the foundations of spirituality for a new era.
• Honoured as Yugpurush, He gave the world a rich heritage that continues to guide generations of seekers, in a short
span of 34 years.
• His life and works are an invitation to turn within and discover the eternal truths.

3. Quit India Movement


99 | P a g e
Why in News?
On 8th Aug 2022, India completed 80 years of Quit India Movement also known as August Kranti.
About

• On 8th August 1942, Mahatma Gandhi called to end British rule and launched the Quit India Movement at the
session of the All-India Congress Committee in Mumbai.
• Gandhiji gave the call “Do or Die” in his speech delivered at the Gowalia Tank Maidan, now popularly known as
August Kranti Maidan.
• Aruna Asaf Ali popularly known as the 'Grand Old Lady' of the Independence Movement is known for hoisting the
Indian flag at the Gowalia Tank Maidan in Mumbai during the Quit India Movement.
• The slogan ‘Quit India’ was coined by Yusuf Meherally, a socialist and trade unionist who also served as Mayor of
Mumbai.
• Meherally had also coined the slogan “Simon Go Back”.
• What caused the Movement? ➔
o Failure of Cripps Mission: The immediate cause for the movement was the collapse of Cripps Mission. Under
Stafford Cripps, the mission was sent to resolve the Indian question of a new constitution and self-
government. It failed because it offered India not complete freedom but the Dominion Status to India, along
with the partition.
o Indian Involvement in World War II without prior consultation: The British assumption of unconditional
support from India to the British in World War II was not taken well by the Indian National Congress.
o Shortage of Essential Commodities: The economy was also in shatters as a result of World War II.
• Demands of the Movement? ➔
o The demand was to end the British rule in India with immediate effect to get the cooperation of Indians in
World War-II against fascism.
o There was a demand to form a provisional government after the withdrawal of the Britishers.

4. Aurobindo Ghose
Why in News?
150th birth anniversary of Aurobindo Ghose will be observed on 15th August.
About

• Birth ➔ August 15, 1872.


• At the age of seven he was taken to England for education.
• Popularly known as yogi Rishi Aurobindo. A revolutionary, nationalist, poet, educationist and philosopher.
• Contributions ➔
▪ He became a state service officer on his return to India in 1893.
▪ He worked as a professor at Baroda College.
o Revolutionary:
▪ He also joined a revolutionary society and took a leading role in secret preparations for an uprising
against the British Government in India.
▪ He was the first political leader in India to openly put forward, in his newspaper Bande Mataram,
the idea of complete independence for the country.
▪ He was the first proponent of ‘Purna Swaraj’ more than 20 years before the Indian National
Congress’ declaration in 1930.
o Literary works:
▪ He was also a journalist and his first philosophical magazine called Arya was published in 1914.
▪ Among his many writings are The Life Divine, The Synthesis of Yoga and Savitri.

100 | P a g e
• Death ➔ Aurobindo Ghose died on December 5, 1950.

5. Navroz festival
Why in News?
Navroz is celebrated at the time of the vernal equinox (marking the start of spring) in the Northern Hemisphere. It is
being celebrated in India on 16th August in India.
About

• Navroz is also known as Parsi New Year.


• In Persian, ‘Nav’ stands for new, and ‘Roz’ stands for the day, which literally translates to ‘new day’.
• It is celebrated in March globally (as per the Iranian calendar), Navroz arrives 200 days later in India and is celebrated
in the month of August as the Parsis here follow the Shahenshahi calendar which doesn’t account for leap years.
• The Persian king Jamshed is credited with having created the Shahenshahi calendar.
• Navroj is on the list of UNESCO Intangible Cultural Heritage of Humanity of India.

👉 Zoroastrianism

• It is one of the earliest known monotheistic faiths, created over 3,500 years ago in ancient Iran by Prophet
Zarathustra
• It was the official religion of Persia from 650 BCE until the emergence of Islam in the 7th century. With the coming
of Islam numerous Zoroastrians fled to India (Gujarat) and Pakistan, thus the name Parsis (‘Parsi’ is Gujarati for
Persian).
• India has one of the largest single groups of Parsis (currently about 61000) in India. The world population is estimated
at 2.6 million Zoroastrians.
• In India, Parsis are one of the notified minority communities.

👉 Other new year

• Chaitra Shukla Pratipada (beginning of the new year of the Vedic (Hindu) calendar)
• Gudi Padwa and Ugadi
• Navreh (the lunar new year in Kashmir)
• Sajibu Cheiraoba (celebrated by Meiteis (Manipur))
• Cheti Chand (by Sindhis)
• Losoong (Sikkimese new year).

6. Traditional Knowledge Digital Library


Why in News?
The Union Cabinet chaired by the Prime Minister has approved the “Widening access of the Traditional Knowledge Digital
Library(TKDL) database to users, besides patent offices”.

About

• Established in 2001 by Council of Scientific and Industrial Research (CSIR) and the Department of Indian Systems of
Medicine and Homeopathy (ISM&H, now Ministry of AYUSH).
• Purpose ➔ It is a first-of-its-kind database on Indian systems of medicine for preventing bio-piracy and
misappropriation of our traditional knowledge.
• Features ➔ The TKDL currently contains information from existing literature related to Indian Systems of
Medicine(ISM) such as Ayurveda, Unani, Siddha, Sowa Rigpa and Yoga.
• The information is documented in a digitized format in five international languages which are English, German,
French, Japanese and Spanish.
101 | P a g e
• TKDL provides information in languages and formats understandable by patent examiners at Patent Offices
worldwide, so as to prevent the erroneous grant of patents.
• Who can access the TKDL database? ➔
o Until now, access to the complete TKDL database is restricted to 14 Patent Offices worldwide for the
purposes of search and examination.
o But from now on the TKDL database can also be accessed by users. This would be through a paid
subscription model with a phase-wise opening to national and international users.
• Significance ➔ This will help drive research and development and innovation based on India’s valued heritage
across diverse fields.

7. Veer Durgadas Rathore


Why in News?
Recently, the Defence minister of India unveiled the statue of Veer Durgadas Rathore in Jodhpur.
About

• He was the Rathore Rajput General of the Kingdom of Marwar and had defied Aurangzeb.
• A famous warrior credited with spearheading the fight against the Mughals.
• Commanded the Rathore forces during the Rajput War (1679–1707).
• Played a major role in the Rajput Rebellion (1708–1710) which became one of the main reasons for the decline of
the Mughal Empire.
• He was elected as the leader of the revolt along with Raja Jai Singh II of Jaipur.
• Regarded as a symbol of social harmony, honesty, bravery and devotion.
• Fought against divisive elements in society.

8. Kamakhya Temple
Why in News?
The Northeast Frontier Railway (NFR) is planning to construct a ropeway from the Kamakhya Railway Station to the
Kamakhya temple in Guwahati.
About

• It is situated on the Nilachal Hill in western part of Guwahati city in Assam.


• It is one of the oldest of the 51 Shakti Peethas in India.
• The main temple is surrounded by individual temples dedicated to the ten Mahavidyas:
• Kali, Tara, Sodashi, Bhuvaneshwari, Bhairavi, Chhinnamasta, Dhumavati, Bagalamukhi, Matangi and Kamalatmika.
• It is a centrepiece of Tantrik Shaktism cult in India
• Being the centre for Tantra worship this temple attracts thousands of tantra devotees in an annual festival known as
the Ambubachi Mela. Another annual celebration is the Manasha Puja.
• Temple Architecture ➔
o It had been modelled out of a combination of two different styles, namely, the traditional nagara or North
Indian and saracenic or Mughal.
o Thus, being an unusual combination which came into its existence on this famous sakti shrine of India, it
has been named as the Nilachala Style of Architecture.
o The reason behind development of this new type by transgression of the orthodox nagara tradition is stated
in the Darang-raj vamsavali, a compilation of 17th century A.D.

9. Lord Curzon
Why in News?

102 | P a g e
The Bardhaman municipality in West Bengal has decided to erect a statue of an erstwhile maharaja in front of the
landmark Curzon Gate in the city.
About

• Born in 1859, George Nathaniel Curzon was a British conservative politician who was educated at the elite
institutions of Eton and Oxford.
• He served as Under-Secretary of State for India (1891-1892), and for Foreign Affairs (1895-1898).
• He was the Viceroy of India from the year 1899 to 1905.
• Of all the Viceroys of India, Curzon is possibly the most criticised.
• Actions taken by Curzon as viceroy of India ➔
o In 1899, he passed the Calcutta Municipal Amendment Act which reduced the number of elected
representatives in the Calcutta Corporation.
o He was also one of the more openly imperialist of viceroys, and a man who saw Britain’s rule over India as
critical to the survival of empire.
o In 1900, Curzon famously stated, “We could lose all our [white settlement] dominions and still survive, but
if we lost India, our sun would sink to its setting.”
o Curzon created a separate Muslim majority province of the North-West Frontier Province.
o He also sent a British expedition to Tibet and established a separate police service in India.
o He was also responsible for establishing the Archaeological Survey of India, in order to study and protect
historical monuments.
o In 1904, he passed the Universities Act (1904) that placed Calcutta University under government control,
and the Indian Official Secrets Amendment Act that reduced the freedom of the press even further.
o He is the one who partitioned Bengal in 1905, and triggered a wave of Bengali nationalism that contributed
to the wider Indian national movement.

👉 Partition of Bengal

• Calcutta was the capital of the British Raj, and Bengal Presidency was one of the largest provinces in India.
• It encompassed the present day West Bengal, Bangladesh, Bihar, parts of Chhattisgarh, Odisha, and Assam.
• For long, the British had maintained that Bengal was too large to efficiently manage and administer.
• It was also believed that with Calcutta as the nerve centre of the educated nationalists, the resistance to colonial
rule would only increase.
• In July 1905, Curzon announced the partition of Bengal into two provinces.
• East Bengal and Assam, with a population of 38 million, was predominately Muslim, while the western province,
called Bengal, and was reduced to 55 million people, primarily Hindus.
• In opposition to the partition, nationalist leaders organized a campaign of boycott of British goods and institutions,
and encouraged the use of local products.
• After a formal resolution was passed at a meeting in Calcutta in August 1905, the Swadeshi movement began.
• The Swadeshi movement and boycott was not restricted to Bengal, and spread to other parts of the country,
including Punjab, Maharashtra, and parts of the Madras Presidency.
• In 1905, Curzon resigned and returned to England after losing a power struggle with the commander-in-chief of the
British Army, Lord Kitchener.
• The protests continued after his exit, and the colonial government in 1911 announced the reunification of Bengal,
and the capital of the Raj was shifted from Calcutta to Delhi.

10. Sutr Santati Exhibition


Why in News?
The Union Minister of Culture has inaugurated the Sutr Santati Exhibition.

103 | P a g e
About

• Organized by ➔ Ministry of Culture and the National Museum in collaboration with Abheraj Baldota Foundation.
• Purpose ➔ Sutr Santati literally means the continuity of yarn. It is a textile exhibition representing 75 traditional
Indian textiles designed to foster Indian pride using traditional techniques using indigenous natural yarns.
• This exhibition will bring to the forefront all the craftspeople from various clusters pan India involved in creating
every piece.

👉 National Museum

• National Museum was established in 1949. It holds a variety of articles ranging from the pre-historic era to modern
works of art.
• Origin ➔ The blueprint for establishing the National Museum was prepared by the Maurice Gwyer Committee in
May 1946.
• Nodal Ministry ➔ Ministry of Culture.
• Located in ➔ New Delhi.

11. Mandala Art


Why in News?
Residents of Liverpool are amazed over a Mandala the size of one and a half football pitches in length created by artist
James Brunt with materials such as leaves and rocks.
About

• Mandala literally means “circle” or “centre” in Sanskrit. It is defined by a geometric configuration that usually
incorporates the circular shape in some form.
• Mandala patterns are a centuries-old motif that is used to depict the cosmos and have been adapted by artists the
world over.
• It can also be created in the shape of a square. But a mandala pattern is essentially interconnected.
• Origin of Mandala Art? ➔
o It is believed to be rooted in Buddhism, appearing in the first century BC in India. Over the next couple of
centuries, Buddhist missionaries travelling along the silk road took it to other regions.
o In Hinduism, the mandala imagery first appeared in Rig Veda(1500 – 500 BCE).
• What is the meaning of Mandala motifs? ➔
o It is believed that by entering the mandala and moving towards its center, one is guided through the cosmic
process of transforming the universe from one of suffering to that of joy.
o In Hinduism, a mandala or yantra is in the shape of a square with a circle at its centre.
o Elements in Mandala Art: There are various elements incorporated within the mandala, each of which has
its own meaning. For instance, the eight spokes of the wheel (the dharmachakra) represent the eightfold
path of Buddhism, the lotus flower depicts balance, and the sun represents the universe.
o Facing up, triangles represent action and energy, and facing down, they represent creativity and knowledge.

👉 Mandala in Modern Art

• Mandala continues to appear in Thangka paintings. But it also has a central place in the practice of mainstream
artists associated with the tantric and neo-tantric spiritual movements.
• In the 1960s Sohan Qadri and Prafulla Mohanty gained widespread recognition for their works that were imbibed
with tantric symbolism, such as mandalas that are also used in the rituals of tantric initiation.
• Geometric compositions also dominated the works of artists such as Biren De, GR Santosh, Shobha Broota, and
famously SH Raza, who visualized the Bindu as the centre of his universe and the source of energy and life.

104 | P a g e
12. Pandurang Khankhoje
Why in News?
Lok Sabha Speaker will be travelling to Mexico where he will unveil statues of Swami Vivekananda and Maharashtra-born
freedom fighter and agriculturalist Pandurang Khankhoje (1883-1967).
About

• Pandurang Khankhoje (1884 – 1967) was an Indian revolutionary, scholar and agricultural scientist.
• He was one of the founding members of the Ghadar Party, established by Indians living abroad in 1914, mostly
belonging to Punjab. Its aim was to lead a revolutionary fight against the British in India.

👉 What was the connection between Pandurang Khankhoje and Mexico?

• Pandurang Khankhoje decided to go abroad for further training in revolutionary methods and militaristic strategy.
He joined the Mount Tamalpais Military Academy in the US.
• At the military academy, he met many people from Mexico. The Mexican Revolution of 1910 had led to the
overthrow of the dictatorial regime and this inspired Khankhoje.
• Along with the Indian workers, militant action was planned by Khankhoje in India, but the outbreak of the First
World War halted these plans. He then reached out to Bhikaji Cama in Paris, and met with Vladimir Lenin in Russia
among other leaders, seeking support for the Indian cause.
• However, as he was facing possible deportation from Europe and could not go to India, he sought shelter in Mexico.
• In Mexico, he was appointed as a professor at the National School of Agriculture. He researched corn, wheat, pulses
and rubber, developing frost and drought-resistant varieties and was part of efforts to bring in the Green Revolution
in Mexico.

13. Mithila Makhana
Why in News?
Recently, Bihar’s Mithila Makhana has been awarded the Geographical Indication (GI) Tag by the Union Government.
About

• Botanical name ➔ Mithila Makhana is locally known as Makhan in Mithila. Its Botanical name is Euryale Ferox Salisb.
• Region associated ➔ This special variety of Aquatic Fox Nut is cultivated in the Mithila region of Bihar and adjoining
areas of Nepal.
• Micro-nutrients ➔ Fox Nuts are rich in protein and fiber and have various micronutrients such as magnesium,
calcium, phosphorus and iron.
• Significance ➔
o The Maithili Brahmin community extensively uses and distributes Makhana during the Kojagara Puja
festival.
o This move is expected to help growers get the maximum price for their premium produce.
• Fifth Product ➔ This is the fifth product from Bihar which is awarded the GI Tag.
o Bhagalpur’s Jardalu Mango, Katarni Dhaan (rice), Nawada’s Magahi Paan and Muzaffarpur’s Shahi Litchi
have got GI tag prior to this.

👉 GI Tag

• GI or Geographical Indication Tag is used for products which have specific geographical origin or have qualities that
can be attributed specifically to the region.
o A GI is primarily an agricultural, natural or a manufactured product (handicrafts and industrial goods)
originating from a definite geographical territory.

105 | P a g e
• Paris Convention ➔ It is a part of the intellectual property rights that comes under the Paris Convention for the
Protection of Industrial Property.
• Act in India ➔ In India, Geographical Indications registration is administered by the Geographical Indications of
Goods (Registration and Protection) Act of 1999.
• Protection ➔ Once a product gets this tag, any person or company cannot sell a similar item under that name.
• Validity ➔ This tag is valid for a period of 10 years following which it can be renewed.
• Benefits of GI Tag ➔
o It provides legal protection to Indian Geographical Indications which in turn boost exports.
o It promotes the economic prosperity of producers of goods produced in a geographical territory.

14. Acculturation
Why in News?
Recently the term “Acculturation “ was in the news.
About

• The concept of acculturation was coined in 1880 by American geologist John Wesley Powel. He defined it as the
psychological changes induced in people due to cross-cultural imitation, resulting from the interaction with
different cultures.
• At present ,it is defined as the process in which a person or group from one culture comes in contact with another
culture, adopting the values and practices of the other while still retaining their own distinct identity.
• Sociologists understand acculturation as a two-way process, wherein the minority culture adopts aspects of the
majority to fit in and the culture of the majority is also influenced by that of the minority.
• Outcomes of acculturation ➔ Acculturation affects various aspects of life, including dietary patterns, fashion, art,
architecture, work culture, and literature.
o Acculturation allows us to learn and understand new aspects of various cultures and appreciate their
differences.

15. Yakshagana
Why in News?
This newscard is an excerpt of the original article published in TH.

About

• Yakshagana is a traditional theater, developed in Dakshina Kannada, Udupi, Uttara Kannada, Shimoga and western
parts of Chikmagalur districts, in the state of Karnataka and in Kasaragod district in Kerala.
• It emerged in the Vijayanagara Empire and was performed by Jakkula Varu.
• It combines dance, music, dialogue, costume, make-up, and stage techniques with a unique style and form.
• Towards the south from Dakshina Kannada to Kasaragod of Tulu Nadu region, the form of Yakshagana is called as
‘Thenku thittu’ and towards north from Udupi up to Uttara Kannada it’s called as ‘Badaga Thittu‘.
• It is sometimes simply called “Aata” or āṭa (meaning “the play”). Yakshagana is traditionally presented from dusk to
dawn.
• Its stories are drawn from Ramayana, Mahabharata, Bhagavata and other epics from both Hindu and Jain and other
ancient Indic traditions.

16. Garba
Why in News?

106 | P a g e
India has nominated the dance form Garba to be inscribed on UNESCO’s intangible cultural heritage list. If approved, it
will become the first intangible cultural heritage of Gujarat to be listed by UNESCO.
About

• Garba is a form of dance which originates from the state of Gujarat in India. Traditionally, it is performed during the
nine-day Hindu festival Navarātrī.
• The word garba comes from the Sanskrit word for womb and so implies gestation or pregnancy — life.
• Traditionally, the dance is performed around a clay lantern with a light inside, called a Garbha Deep (“womb lamp”).
This lantern represents life and the fetus in the womb in particular. The dancers thus honour Durga, the feminine
form of divinity.
• Garba is performed in a circle reflecting the cycle of life. As the cycle of time revolves from birth, to life, to death
and again to rebirth, the only thing that is constant is the Goddess, the idol of whom is placed stationary.

17. Chola Dynasty


Why in News?
Recently, the Idol Wing-CID has submitted documents to U.S. officials through the Central government to retrieve six
exquisite Chola-era bronze idols.

About

• The reign of the Cholas began in the 9th century when they defeated the Pallavas to come into power.
• This rule stretched over for over five long centuries until the 13th century.
• The Early periods of the Chola rule saw the onset of the Sangam literature.
• Developments ➔
o One of the longest-ruling dynasties in the southern regions of India.
o The medieval period was the era of absolute power and development for the Cholas.
o Cholas made South Asia and South-East India into a military, economic and cultural power.
• Prominent Rulers ➔ Vijayalaya, Aditya I, Rajaraja Chola I, Rajendra Chola I, Rajadhiraja Chola, Virarajendra Chola
and Kulothunga Chola I.

👉 Rajendra Chola I

• Succeeded the mighty Rajaraja Chola.


• Rajendra I was the first to venture to the banks of Ganges. He was popularly called the Victor of the Ganges.
• Built new capital called the Gangaikondacholapuram where he received the title of ‘Gangaikonda’.
• He built the Brihadisvara Temple and upgraded several others.
• His period is referred to as the golden age of the Cholas.

18. INDIAN KNOWLEDGE SYSTEMS (IKS) INITIATIVE


Why in News?
Raja mantri chor sipahi, posham pa, gilli danda etc. are among 75 indigenous games that will be introduced in schools
under IKS initiative.

About

• IKS is an innovative cell under Ministry of Education at All India Council for Technical Education (AICTE).
• It is established to promote interdisciplinary research on all aspects of IKS, preserve and disseminate IKS for further
research and societal applications.
• It aims to spread rich heritage and traditional knowledge in field of Arts and literature, Agriculture, Basic Sciences,
Engineering & Technology, Architecture, Management, Economics, etc.

107 | P a g e
Miscellaneous
1. ONGC Para Games

Why in News?
Government of India inaugurated the 4th Oil and Natural Gas Corporation (ONGC) Para Games in New Delhi.
About

• What is it? ➔ ONGC Para Games is a special Human Resource Initiative to provide Equal Opportunities to Persons
with Disabilities PwDs in all walks of corporate life.
• The intent is to contribute in the holistic development of the Human Resources of the public enterprises, who are
our most valuable asset, in the pursuit of Energy.
• Participants ➔ Employees of eight central oil and gas public enterprises are participating in the 4th ONGC Para
Games, being organized by Oil and Natural Gas Corporation Limited (ONGC) during 2-4 August 2022.
• International Level ➔ ONGC organizes the Para Games in an international format from its very first edition in 2017
with the help of the Paralympic Committee of India, where 120 ONGC PwD employees participated in games like
athletics, badminton, table tennis and wheelchair race.

👉 ONGC

• ONGC is a public sector petroleum company.


• Under the leadership of Pandit Jawaharlal Nehru, the foundation stone of ONGC was laid in 1955 as the Oil and Gas
Division under the Geological Survey of India.
• It may be noted that on 14th August 1956, it was renamed as the Oil and Natural Gas Commission and in 1994 the
Oil and Natural Gas Commission was converted into a corporation.
• In the year 1997 it was accepted as one of the Navaratnas by the Government of India, while in the year 2010 it was
given the status of Maharatna.

👉 Paralympic Games

• The Paralympic Games or Paralympics is the largest international event for disabled athletes and societal change
and takes place shortly after every Olympic Games in the same host city.
• The Paralympic Games are held every two years, alternating between the summer and winter Paralympic Games.
• The Paralympics has grown from a small gathering of British World War II veterans in 1948 to become one of the
largest international sporting events by the early 21st century.
• The first Paralympic Games was held in 1960 in Rome.

2. Aridity Anomaly Outlook(AAO) Index


Why in News?
Recently, Indian Meteorological Department (IMD) has released the Aridity Anomaly Outlook (AAO) Index of July 2022,
which says at least 85% of districts face arid conditions across India.
About

• Developed by ➔ India Meteorological Department (IMD).


• Purpose ➔ The index monitors agricultural drought, a situation when rainfall and soil moisture are inadequate to
support healthy crop growth till maturity, causing crop stress.
o An anomaly from the normal value signifies a water shortage in these districts that could directly impact
agricultural activity.

108 | P a g e
• The index is computed for weekly or two-week periods. For each period, the actual aridity for the period is
compared to the normal aridity for that period.
o Negative values indicate a surplus of moisture while positive values indicate moisture stress.
• Parameters used ➔ Actual evapotranspiration and calculated potential evapotranspiration which requires
temperature, wind and solar radiation values.
o Note: Actual evapotranspiration is the quantity of water that is actually removed from a surface due to the
processes of evaporation and transpiration.
• Potential evapotranspiration is the maximum attainable or achievable evapotranspiration for a given crop due to
evaporation and transpiration.
• Significance of this index ➔ Impacts of drought on agriculture, especially in the tropics where defined wet and dry
seasons are part of the climate regime.
o Both winter and summer cropping seasons can also be assessed using this method.

👉 Key findings of the index

• At least 85% of districts face arid conditions across India.


• Some 196 districts are in the grip of a ‘severe’ degree of dryness and 65 of these are in Uttar Pradesh (highest).
• Bihar had the second-highest number of districts (33) experiencing arid conditions. The state also has a high rainfall
deficit of 45%.

👉 India Meteorological Department (IMD)

• IMD was established in 1875.


• It is an agency of the Ministry of Earth Sciences.
• It is the principal agency responsible for meteorological observations, weather forecasting and seismology.

3. Central Universities (Amendment)Bill 2022


Why in News?
Lok Sabha has passed the Central Universities (Amendment)Bill 2022.
About

• The Bill amends the Central Universities Act, 2009, which provides for establishing central universities in various
states.

👉 Key features of the Bill include:

• Gati Shakti Vishwavidyalaya: The Bill seeks to convert the National Rail and Transportation Institute, Vadodara (a
deemed university) to the Gati Shakti Vishwavidyalaya, which will be a central university.
• The National Rail and Transportation Institute was declared a deemed university under the University Grants
Commission Act, 1956. The Vishwavidyalaya will be sponsored and funded by the central government through the
Ministry of Railways.
• Scope of education: The Bill provides that Gati Shakti Vishwavidyalaya will take measures to provide quality
teaching, research, and skill development in disciplines related to transportation, technology, and management. If
required, the University may also establish centres in India and abroad.

4. Diammonium Phosphate
Why in News?
As part of the Atma Nirbhar Bharat initiative, the Government of India has been supporting the Indian fertiliser companies
including those producing Diammonium Phosphate to strengthen their backend supply chain.

109 | P a g e
About

• Di-ammonium Phosphate (DAP) [(NH4)2HPO4] is the world’s most widely used phosphorus fertilizers.
• It is a preferred fertilizer in India because it contains both Nitrogen (18% N) and Phosphorus [46% P2O5 (20% P)]
that are primary macronutrients and part of 18 essential plant nutrients.
• DAP is manufactured by reacting Ammonia with Phosphoric acid under controlled conditions in fertilizer plants.
• Uses ➔ DAP is an excellent source of P and N for plant nutrition.
o It is highly soluble and thus dissolves quickly in soil to release plant-available phosphate and ammonium.
o To prevent the possibility of seedling damage, care should be taken to avoid placing high concentrations of
DAP near germinating seeds.
o DAP is used in many applications as a fire retardant to prevent a forest from burning. It then becomes a
nutrient source after the danger of fire has passed.
o DAP is used in various industrial processes, such as metal finishing.
o It is commonly added to wine to sustain yeast fermentation and to cheese to support cheese cultures.

5. Udyam Portal
Why in News?
Around one crore Micro, Small and Medium Enterprises (MSMEs) have registered on the Udyam portal within a span of
25 months, according to the Union Minister of Micro, Small, and Medium Enterprises.
About

• Launched on ➔ 1st July, 2020.


• It’s an online system for registering MSMEs, launched by the Union MSME ministry.
• Further, it is linked to the databases of the Central Board of Direct Taxes(CBDT) and Goods and Services Tax Network
(GSTN). 
o GSTN is a unique and intricate IT enterprise that establishes a channel of communication and interaction
between taxpayers, the central and various state governments, and other stakeholders.
• It is fully online, does not require any documentation, and is a step towards Ease of Doing Business for MSMEs.
• Significance ➔
o Udyam registrations are essential for MSMEs to utilize the benefits of schemes for the Ministry of MSME
and for Priority Sector Lending from banks.
o Also, MSMEs contribute to the GDP, exports, and employment generation of the country.
• New Initiatives ➔
o The MSME ministry signed MoUs with the Ministry of Tourism and the National Small Industries Corporation
for the sharing of Udyam data.
o Further, the Digi locker facility for Udyam registration would be linked.

6. Baumol’s Cost Disease


Why in News?
How wages rise even when there is no significant increase in productivity.
About

• Economics theory states that wages rise when there’s greater productivity.
• However, Baumol’s cost disease refers to the increase in the wages of certain labourers even though their
productivity or skill level has not risen commensurately.
• This happens because there is competition between various industries for the limited supply of labour.
• So, even if the productivity of their employees has not risen significantly, employers in many cases have no choice
but to pay higher wages in order to prevent the movement of labourers to other higher-paying industries.
110 | P a g e
• It should be noted that labour is often a kind of non-specific resource that can be used across various industries.

7. World Dairy Summit 2022


Why in News?
India is hosting the World Dairy Summit 2022 in New Delhi.
About

• Background ➔ The last World Dairy Summit was organized in 1974 in New Delhi. After 48 years, India is organizing
the summit again.
• Organized by ➔ International Dairy Federation(IDF).
• Hosted by ➔ India.
• Aim ➔ To provide a forum for the industry experts to share knowledge and ideas on how the sector can contribute
to nourishing the world with safe and sustainable dairying.
• Theme ➔ Livelihood and Nutrition.

👉 Dairy Sector in India

• India is ranked 1st in milk production contributing 23% of global milk production.
• Uttar Pradesh is the highest milk-producing state in India contributing around 18% to the total milk production
followed by Rajasthan, Andhra Pradesh, Gujarat and Punjab.
• Initiated in 1970, Operation Flood was arguably the most ambitious dairy development programme that
transformed India into one of the largest milk producers.
• However, India is yet to join the ranks of major milk exporting nations as much of what we produce is directed
towards meeting domestic demands.

8. Khayyam Satellite
Why in News?
Russia will launch an Iranian remote sensing satellite ‘Khayyam’ into orbit by using its Soyuz 2.1B satellite carrier.
About

• The satellite is named after 11th-12th century Persian polymath Omar Khayyam.
• This satellite is a remote sensing satellite designed and manufactured at enterprises that are part of the Russian
Space Corporation Roscosmos.
• The satellite has high imaging accuracy and is capable of filming the earth's surface in different image spectra. It
aims to
o Monitor the country's borders,
o Enhance agricultural productivity and
o Monitor water resources and natural disasters.
• Russia is putting the satellite into space but it will be guided and controlled from ground stations in Iran.

9. Pyrene
Why in News?
Researchers at the CSIR have identified a fungus capable of removing toxic and carcinogenic Pyrene or Polycyclic
Aromatic Hydrocarbons(PAHs) from the environment.

About

111 | P a g e
• What is it? ➔ Pyrene is a polycyclic aromatic hydrocarbon (PAH) consisting of four fused benzene rings. It has
several carcinogenic and mutagenic properties.
• They originate from multiple sources including combustion of petrogenic fossil fuels and incomplete incineration of
municipal wastes and biomass.
• They get lodged into the environmental matrices like soil, water and atmosphere resulting in widespread
environmental pollution necessitating adequate remediation of contaminated environmental matrices.

👉 What have the researchers found?

• Researchers have found a fungus named white-rot fungus Trametesmaxima IIPLC-32 which has the potential to
cause microbial degradation of pyrene.
• Growing on dead plants, this fungus causes pyrene degradation using special enzymes. This fungus also acts to
decrease the pollution level of the soil.
• Hence, this fungus may prove to be helpful in the remediation of pyrene.

10. Indian Virtual Herbarium


Why in News?
Indian Virtual Herbarium, biggest database of country’s flora, is a global hit.
About

• It is the biggest database of the country’s flora.


• Developed by scientists of the Botanical Survey of India (BSI), the Indian Virtual Herbarium was inaugurated by the
Union Minister of Environment Forest and Climate Change.
• Each record in the digital herbarium includes an image of the preserved plant specimen, scientific name, collection
locality, collection date, collector name, and barcode number. The digital herbarium also includes features to
extract the data State-wise and users can search plants of their own States which will help them to identify regional
plants and in building regional checklists.

11. India ki Udaan Project


Why in News?
Google has launched the “India ki Udaan” Project to mark 75 years of Independence.
About

• It is being implemented by Google Arts and Culture, in association with the Ministry of Culture.
• It celebrates the achievements of India and is based on the theme ‘Unwavering and undying spirit of India over
these past 75 years’.
• The centrepiece of its celebrations is a new online collection titled “India Ki Udaan”. It features India’s rich cultural
diversities, including iconic moments from the last 75 years.
• Significance of the project: This project offers a unique view of India’s remarkable moments and lets people
discover some of the most memorable moments in India’s modern history. These include iconic personalities, and
the proudest scientific and sporting achievements and show how women in India continue to inspire the world.

12. Operation Yatri Suraksha
Why in News?
Railway Protection Force(RPF) has launched a Pan-India Operation under the code name “Operation Yatri Suraksha”.
About

• Launched by ➔ Railway Protection Force(RPF).

112 | P a g e
• Aim ➔ To improve the security of passengers travelling by Indian Railways.
• Feature ➔ Under the operation, the RPF will take various steps including increasing CCTV surveillance, escorting
trains, surveilling active criminals and increasing police presence at stations.

👉 Other operations launched by RPF

• Mission Jeewan Raksha ➔ Under this, RPF personnel have saved 1650 lives from the wheels of the running trains
at railway stations in the last four years.
• Meri Saheli Initiative ➔ It was launched by RPF to provide security to lady passengers in long distance trains
particularly travelling alone or those who are vulnerable to crime.
• Operation Amanat ➔ Under this, RPF retrieved left behind luggage having value of more than 23 Crores belonging
to 12377 passengers and returned to them after due verification.
• Toll Free Helpline Number 139 (24 X 7) ➔ It is a single helpline number launched by Indian Railways for quick
grievance redressal and enquiries by passengers during their train journeys.
• Operation Number Plate ➔ It was launched by the RPF to identify and verify all vehicles parked in Railway
premises, circulating areas, parking lots and even in the ‘No Parking’ areas for a longer duration.
• Operation Thirst ➔ It was launched by the RPF to stop selling of unauthorised Packaged Drinking Water (PDW) in
railway premises.

13. The Sacrifice Zone


Why in News?
Myanmar bears cost of green energy, so it is now called the ‘Sacrifice Zone’.
About

• A “sacrifice zone” or “sacrifice area” is a place that destroys itself for the good of the world.
• It has been permanently impaired by heavy environmental alterations or economic disinvestment, often through
locally unwanted land use.
• These zones most commonly exist in low-income and minority communities.
• The northern forest frontier areas of Myanmar are the source of several key rare earths, often called the vitamins
of the modern world.

14. Ongole Cattle Breed


Why in News?
Ongole breed of cattle had remained indispensable for all farm operations for centuries in Prakasam district of Andhra
Pradesh in view of their draught power.
About

• Ongole cattle are an indigenous cattle breed that originates from Prakasam District in the state of Andhra Pradesh.
• The breed derives its name from the place the breed originates from, Ongole.
• The Ongole breed of cattle Bos Indicus, has a great demand as it is said to possess resistance to both foot and mouth
disease and mad cow disease.
• Special about this breed ➔
o Cattle breeders use the fighting ability of the bulls to choose the right stock for breeding in terms of purity
and strength.
o Ongole cattle are known for their toughness, rapid growth rate, and natural tolerance to tropical heat and
disease resistance.
o It was perhaps the first Indian breed of cattle to gain worldwide recognition.
o Ongole milk is rich in A2 (allele of Beta Casein).

113 | P a g e
o They fetches a premium price of over ₹150 per litre as it enables consumers build immunity against viral
and other diseases.

15. The Zaporizhzhia Nuclear Plant
Why in News?
Russia and Ukraine have accused each other of shelling Europe’s largest nuclear power plant.
About

• The total length of the new rail line will be 116 kms. The project will be
completed by 2026-27.
• Ambaji is a famous important pilgrimage destination and is one of the 51
Shaktipeeths in India and attracts millions of devotees from Gujarat as well
as other parts of the country and abroad every year.
• The construction of this line will facilitate easy travel for these millions of
devotees.
• Further, the devotees visiting the Ajitnath Jain temple (one of the 24 holy
Jain Tirthankaras) at Taranga Hill would also be greatly benefitted by this
connectivity.
• This railway new line between Taranga Hill-Ambaji- Abu Road will connect these two important religious sports with
railway’s main network.
• The alignment of the proposed doubling will traverse through Sirohi district of Rajasthan and Banaskantha and
Mahesana districts of Gujarat. This project will also provide alternative route for existing Ahmedabad-Abu Road
railway line.

16. DigiYatra
Why in News?
The Delhi International Airport Ltd (DIAL), run by GMR, announced the soft launch of the Centre’s DigiYatra initiative,
rolling out the beta version of its app for Android platforms.
About

• The ‘DigiYatra’ is a Biometric Enabled Seamless Travel experience (BEST) based on Facial Recognition Technology.
• It aims to provide a paperless and seamless travel experience to the passengers. “DigiYatra is a unique initiative of
the Government of India, coordinated by the Ministry of Civil Aviation.
• DigiYatra envisages that travellers pass through various checkpoints at the airport through paperless and
contactless processing, using facial features to establish their identity, which would be linked to the boarding pass.
• The DigiYatra Foundation will be the custodian of the passenger ID validation process. It will also define the criteria
for compliance and guidelines for the local airport systems.

17. Carnelian beads


Why in News?
A cluster of 74 carnelian beads was unearthed at Konthagai in the Sivaganga district, which is part of the Keeladi cluster.
About

• In February, the Tamil Nadu State Department of Archaeology commenced the eighth phase of the archaeological
excavations in Keeladi and its cluster sites.
• The third phase of the excavations was underway at the Konthagai burial site.
• Carnelian beads had earlier been unearthed in the Kodumanal and Thandikudi sites.

114 | P a g e
• This is the first time that carnelian beads have been found at this burial site.
• The beads were found inside an urn.
• The carnelian beads signified a trade relationship with western India, particularly present-day Maharashtra and
Gujarat.
• The beads were available only in the western part of the country.

18. PIN Code


Why in News?
The 75th Independence Day marks the 50th year of the introduction of the PIN code.

About

• At the time of independence, there were 23,344 post offices primarily in urban areas in India. But, the country was
growing rapidly, and the postal network had to keep pace.
• The PIN code was then introduced to ease the process of mail sorting and delivery in a country where different
places, often, have the same or similar names and letters written in a wide variety of languages.

👉 How does the PIN Code work?

• The PIN is made up of six digits. The first number indicates the postal region — Northern, Eastern, Western,
Southern; and the number 9, which signifies the Army Postal Service. The second number denotes a sub-region, and
the third represents the sorting district. The remaining numbers narrow the geography further to the specific post
office making the delivery.

👉 Who was the person behind the PIN Code initiative?

• The person behind the initiative was Shriram Bhikaji Velankar, an additional secretary in the Union Ministry of
Communications and a senior member of the Posts and Telegraphs Board.
• Note: Shriram Velankar was also a Sanskrit poet of eminence who had been conferred the President’s Award for
Sanskrit in 1996. One of his most important books of his was Viloma Kavya which is considered a literary masterpiece
because it comprises verses in praise of Lord Rama when read from one side and, when read backwards, it
transformed into verses dedicated to Lord Krishna.

👉 How is the PIN Code still relevant?

• In today’s world where emails have replaced letters, and WhatsApp, Instagram and other social media platforms
have become means of communication, one may think that the ‘pin code’ does not hold the same relevance.
• However, as more and more e-commerce platforms, couriers and food delivery services set up shop, the pin code
is still required to avoid duplication and identify the correct address.

19. Talc
Why in News?
Pharmaceutical giant Johnson and Johnson(J&J) has announced that it will discontinue the sale of its talc-based baby
powder globally by 2023.
About

• What is it? ➔ Talc is the softest known mineral and is mined from underground deposits. Chemically, talc is a
hydrous magnesium silicate.
• It is used in a variety of cosmetic and personal care products like baby powder, lipstick, eyeshadow and foundation.
• When reduced to a fine powder, it is able to absorb moisture and reduce friction which keeps the skin dry, helps
prevent rashes, stops makeup from caking and generally helps improve the feel and texture of a product.
115 | P a g e
👉 Does it cause cancer?

• Asbestos, the name for another group of naturally occurring silicate minerals, can also be found near talc deposits.
• According to the FDA, there is the potential for contamination of talc with asbestos. Asbestos has been used in
construction and manufacturing and is known to cause lung cancer, ovarian cancer, mesothelioma and other health
conditions.

20. Nereguli Paddy


Why in News?
Farmers in the villages of Saidur, Kanale and Hirenallur gram panchayats grow a flood-resistant traditional variety of
paddy.
About

• Nereguli is a flood-resistant traditional variety of paddy.


• Nereguli variety yields about 15 bags of paddy, each weighing about 60 kg per acre.
• Though the yield is less compared to other varieties of paddy, this variety has a higher rate of tolerance to flooding.
• The paddy did not lose its quality even when the field was inundated for 40 days.
• The grass blades rot and what remains are the stalks, which sprout once the water level recedes.
• There are other varieties in Bangladesh and Andhra Pradesh that can withstand floods and grow tall but will collapse
once the water recedes, but Nereguli is quite the opposite, since, it is rooted and grows fast once the water recedes.
• However, due to the extensive use of chemical fertilizers, this variety of paddy too could withstand flood only for
25 days.
• It is liked for its vigour, taste and health quotient.
• This variety is organically grown using traditional methods and is highly nutritious and in great demand, in Kerala
and Goa.

21. Manthan Platform


Why in News?
The Office of the Principal Scientific Adviser(PSA) to the Government of India (GoI) has announced the launch of the
Manthan platform.
About

• What is it? ➔ Manthan is a platform to promote collaboration at scale between industry and the scientific research
and development ecosystem to help meet India’s scientific missions and goals in alignment with the UN-defined
Sustainable Development Goals(SDG) charter.
• Features ➔ The platform will facilitate knowledge transfers and interactions through information exchange
sessions, exhibitions and events to develop a framework for future science, innovation, and technology-led growth.
• It will also provide the necessary fulcrum for transforming the nation through innovative ideas, inventive minds and
public-private-academia collaboration to achieve the country’s sustainability goals.
• Powered by ➔ The platform is powered by NSEIT.

👉 NSIET

• NSIET is a 100% subsidiary of the National Stock Exchange of India. It is a global technology enterprise that offers
a range of digital, data analytics, automation, cybersecurity, and technology-enabled services.

22. Thucydides Trap


Why in News?

116 | P a g e
Is strained China-US relationship spiralling towards a Thucydides Trap scenario?.
About

• Thucydides Trap, or Thucydides’ Trap, is a term popularized by American political scientist Graham T. Allison to
describe an apparent tendency towards war when an emerging power threatens to displace an existing great power
as a regional or international hegemon.
• It was coined and is primarily used to describe a potential conflict between the United States and the People’s
Republic of China.

23. Roshni
Why in News?
The Union Minister has launched India’s first saline water lantern named ‘Roshni’.
About

• What is it? ➔ Roshni is India’s first saline water lantern which uses seawater as the electrolyte between specially
designed electrodes to power the LED lamps.
• Developed by ➔ National Institute of Ocean Technology(NIOT).
• Significance ➔ This is expected to bring ‘ease of living’ to the poor and needy, particularly the fishing community
living along the 7500 Kilometers long coastal line of India.
• Moreover, this lantern can also be used in hinterlands, where seawater is not available, as any saline water or
normal water mixed with the common salt can be used to power the lantern, which is not only cost-effective but
very easy to operate.

24. Federation Internationale de Football Association (FIFA)


Why in News?
Federation Internationale de Football Association (FIFA), suspended the country’s top administrative organization, the
All-India Football Federation (AIFF), for undue influence from third parties.
About FIFA

• Founded in 1904.
• FIFA or the Federation Internationale de Football Association is the highest governing body of football in the world.
• It is the international governing body of association football, futsal, and beach soccer.
• FIFA is a non-profit organisation.
• FIFA was launched to oversee international competition among the national associations of Belgium, Denmark,
France, Germany, Netherlands, Spain, Sweden, and Switzerland. FIFA now comprises 211 member nations.
• It is headquartered in Zurich.
• Six Regional confederations affiliated with FIFA ➔
o The Asian Football Confederation (AFC) is the governing body for Asia and Australia.
o Confederation of African Football (CAF) has 56 members,
o Confederation of North, Central American and Caribbean Association Football (CONCAF) has 41 members,
o Confederation Sudamericana de Fútbol (CONMEBOL) is the South American federation with 10 members,
o Oceania Football Federation (OFC) has 14 members including New Zealand,
o Union of European Football Associations (UEFA) with 55 members is the governing body for Europe.

👉 All-India Football Federation (AIFF)

• Founded in 1937, and gained FIFA affiliation in the year 1948, after India's independence in 1947.
• The All-India Football Federation (AIFF) is the organization that manages the game of association football in India.

117 | P a g e
• Currently, it has an office in Dwarka, New Delhi. India was one of the founding members of the Asian Football
Confederation in 1954.

25. Tilapia Fish


Why in News?
To bring about the blue revolution, the Technology Development Board(TDB), a statutory body under the Department
of Science and Technology is funding its first ever ‘Aquaculture’ project using ‘state of the art’ Israeli technology for
production of Tilapia Fish”.
About

• ‘Tilapia’ has emerged to be one of the most productive and internationally traded food fish in the world.
• The culture of tilapia has become commercially popular in many parts of the world and fishery experts have dubbed
the tilapia as “aquatic chicken” due to its quick growth and low maintenance cultivation.

👉 Aquaculture project for the production of Tilapia Fish?

• In order to facilitate the culture of Tilapia in India in a responsible manner, M/s Fountainhead Agro Farms Private
Limited envisages setting up a complete production line (from breeding to full fish) in Mudhol (Karnataka).
• The company aims to produce 500 tons of Tilapia to be grown from the imported parent broodstock ‘Hermon’ from
Israel.

👉 What is Hermon?

• Hermon is a hybrid of two selected strains of Tilapia, namely Oreochromis Niloticus (Male) and Oreochromis Aureus
(Female).
• It is known for special characteristics such as high growth rate; resistance to low temperature; light (attractive)
colour; all hybrid fry progeny of males only, without the conventional system of usage of hormones.

👉 Significance of this project

• The project will be a great addition to Pradhan Mantri Matsya Sampada Yojana(PMMSY) which aims to double the
export earnings to Rs.1,00,000 crore from the fisheries sector.

26. NIDAAN Portal


Why in News?
India's first portal named NIDAAN' on arrested narco offenders ' gets operational.
About

• The National Integrated Database on Arrested Narco-offenders portal has been developed by the Narcotics Control
Bureau (NCB).
• It is part of the narcotics coordination mechanism (NCORD) portal that was launched in July 2022 during the
national conference on 'Drug trafficking and national security'.
• Key Features ➔
o It sources its data from the ICJS (inter-operable criminal justice system) and the e-Prisons (a cloud-based
application) repository and it is planned to integrate it in the future with the crime and criminal tracking
network system or CCTNS.
o The ICJS, an initiative of the Supreme Court e-committee, was created to enable seamless transfer of data
and information among different pillars of the criminal justice system, like courts, police, jails and forensic
science laboratories, from one platform.
27. Mudhol hounds
118 | P a g e
Why in News?
Mudhol hounds, a breed of hunting dogs native to north Karnataka, could be inducted into the Special Protection Group
(SPG), the elite force protecting the Prime Minister of India.
About

• The Mudhol hounds come from the region bordering Karnataka and Maharashtra.
• According to a popular legend Ch. Shivaji Maharaj bred and used these dogs in his army.
• The breed was named after the erstwhile princely state of Mudhol, which was part of the Bombay Presidency in
British India.
• The last king of the state, King Malojirao Ghorpade, presented a pair of these puppies to King George V when he
visited the UK and it was the English emperor who christened the dogs as Mudhol hounds.
• Mudhol hounds have narrow heads that are disproportionately smaller to their bodies but are tall in height.

28. Forever Chemicals


Why in News?
A recent study found that rainwater from many places across the globe is contaminated with “per- and polyfluoroalkyl
substances,” (PFAs).
About PFAs

• They are called “forever chemicals” because of their tendency to stick around in the atmosphere, rainwater and soil
for long periods of time.
• They are man-made chemicals used to make nonstick cookware, water-repellent clothing, stain-resistant fabrics,
cosmetics, firefighting forms and many other products that resist grease, water and oil.
• They can migrate to the soil, water and air during their production and use.
• Since most PFAs do not break down, they remain in the environment for long periods of time.
• Some of these PFAs can build up in people and animals if they are repeatedly exposed to the chemicals.
• Health risks ➔
o Decreased fertility, developmental effects in children, interference with body hormones, increased
cholesterol levels and increased risk of some cancers.
o Long-term low-level exposure to certain PFAs can make it difficult for humans to build antibodies after
being vaccinated against various diseases.
• Measures ➔
o There are many effective, albeit expensive, methods to remove them from rainwater that have been
collected through various rainwater harvesting methods.
o It can be done with the use of a filtration system with activated carbon.
o Recently, The researchers first placed a PFA compound in a solvent called DMSO (dimethyl sulfoxide).
o They then mixed it with sodium hydroxide (lye) in water. They found that when this mixture was heated
up to boiling temperature, the PFA compound began to degrade.

29. Aqua Bazar App


Why in News?
Recently, The Union Minister of Fisheries, Animal Husbandry and Dairying has launched Aqua Bazar App.
About

• What is it? ➔ It is an online marketplace application that will help the fish farmers and stakeholders to source the
inputs such as fish seed, feed, medicines and services required for fish culture.
• The farmers can also list table-size fish for sale through this app.

119 | P a g e
• Developed by ➔ Bhubaneswar-based ICAR-Central Institute of Freshwater Aquaculture with the funding support
of the National Fisheries Development Board (NPFB).

👉 National Fisheries Development Board(NFDB)

• NFDB was established in 2006 as an autonomous organization under the administrative control of the Department
of Fisheries, Ministry of Fisheries, Animal Husbandry and Dairying.
• Aim ➔ To enhance fish production and productivity in the country and to coordinate fishery development in an
integrated and holistic manner.

30. Kerala Savari


Why in News?
Kerala has soft launched ‘Kerala Savari’, the country’s first online taxi service owned by a State government.
About

• What is it? ➔ It the country’s first online taxi service owned by a State government.
• It is operated by the Motor Workers Welfare Board under the aegis of the Labour Department.
• It aims to ensure fair, safer and decent service to passengers along with fair remuneration to auto-taxi workers.
• The fares are approved by government without any ‘surge pricing’.
• Security related features ➔ It is claimed as a reliable online service for women, children, and senior citizens. This
consideration has been given importance in app designing and driver registration.
o A police clearance certificate is mandatory for drivers joining the scheme apart from the required proper
training.
o A panic button system has been introduced in the app. This button can be pressed in the event of a car
accident or in cases of any other danger.

31. Puros helmet


Why in News?
A Delhi Based Startup has developed a helmet called “Puros”. This helmet can help two-wheeler riders breathe clean air.
About

• What is it? ➔ Puros is an anti-pollution helmet that can help 2-wheeler riders breathe clean air.
• Developed by ➔ Shellios Technolabs with the help of the Department of Science and Technology (DST).
• Features ➔ The helmet has a system set at the back that picks up all particulate matter coming from outside and
cleans the air before it reaches the biker.
• It also has a Bluetooth-enabled app that lets the rider know when it requires cleaning.
• Significance ➔ This helmet will help the two-wheeler riders who were having prolonged daily exposures and that
too, to a double whammy of particulate matter and vehicular emissions in the air that they breathe.

32. Hunger Stones


Why in News?
Europe is suffering from the worst drought in half a millennium. Rivers have dried up so much that ‘hunger stones’ have
been revealed and have gone viral on social media.
About

• Hunger stones or hungersteine in German are a common hydrological marker in central Europe. They date back to
the pre-instrumental era.

120 | P a g e
• These stones were embedded into rivers by ancestors when rivers were subdued to severe levels subsequently
causing famine and food shortages.
• Many of the hunger stones found have unique carvings on them that seek to remind the next generation that if
water levels get to this point, food availability will be affected.
• One famous carving example is in the Elbe river in Děčín, Czech Republic which says “Wenn du mich siehst, dann
weine” (“If you see me, weep”).

33. Coradia iLint


Why in News?
Germany has launched the world’s first fleet of hydrogen-powered passenger trains Coradia iLint.
About

• Coradia iLint trains have a range of up to 1,000 kilometers and a maximum speed of 140 kmph (87 mph).
• By using hydrogen produced with renewable energy the trains will save 1.6 million liters (more than 422,000 gallons)
of diesel fuel a year.
• Hydrogen is currently produced as a byproduct of chemical processes, but German specialty gas company Linde plans
to manufacture it locally using only renewable energy within three years.

34. Azadi Quest


Why in News?
The Union Minister of Information and Broadcasting has launched ‘Azadi Quest’.
About

• What is it? ➔ Azadi Quest is a series of online educational games based on India’s freedom struggle.
• Developed by ➔ Ministry of Information and Broadcasting in collaboration with Zynga India.
• Features ➔ The games are based on the concept of ‘Gamification of Education’ to revolutionize the education
sector in the country. The games are available in both English and Hindi.
• The first game in the series is ‘Azadi Quest: Match 3 Puzzle’, a simple and easy-to-play casual game presenting the
players with the colourful journey of India’s Independence from 1857 to 1947.
• The second game ‘Azadi Quest: Heroes of Bharat’ is designed as a quiz game to test the players’ knowledge about
the heroes of India’s freedom movement through 750 questions spread across 75 levels.
• Significance ➔ These games will impart the knowledge of India’s freedom struggle and the legends of the country’s
great freedom fighters, thereby inducing a feeling of pride and a sense of duty upon the players.

35. Meslin Flour


Why in News?
The Cabinet Committee on Economic Affairs, chaired by the Prime Minister, has approved the the proposal for
amendment of policy of exemption for Wheat or Meslin Flour (HS Code 1101) from export restrictions/ ban.
About

• Meslin is a mixture of wheat and rye that is sown and harvested together. In trade, it is usually classified with wheat.
• It was grown on farms before World War II, but only a few ecological farmers sow it now.
• Meslin flour contains relatively low quantity of wet gluten.
• In 2020, the top exporters of meslin flour were Turkey, Kazakhstan, and Germany.
• In 2020, the top importers of meslin flour were Afghanistan, Iraq, and Netherlands.

36. Indian Boiler Act,1923


121 | P a g e
Why in News?
The Government of India is set to decriminalize the archaic Indian Boilers Act,1923.
About

• Indian Boilers Act-1923 was enacted with the objective to provide mainly for the safety of life and Property of
persons from the danger of explosions of steam boilers and for achieving uniformity in registration and inspection
during the operation and maintenance of boilers in India.

👉 Key changes the government is planning to make to the Indian Boiler Act, 1923?

• Firstly, decriminalize the Boilers Act by removing the provision for a two-year jail time for violation of the rules.
• Secondly, easing the rules by extending the duration of licenses and cutting the frequency of inspections.

37. National Cancer Grid


Why in News?
The National Cancer Grid(NCG) has established the Koita Centre for Digital Oncology (KCDO) to promote the use of digital
technologies and tools to improve cancer care across India.
About

• Background ➔ Established in 2012.


• Established by ➔ Government of India through the Department of Atomic Energy and with the grant support of
Tata Memorial Centre.
• Mandate ➔ To create a network of cancer centres, research institutes, patient groups and charitable institutions
across India.
• With the objective of ➔
o a) developing uniform standards of patient care for prevention, diagnosis and treatment of cancer;
o b) providing specialized training and education in oncology and
o c) facilitating collaborative basic, translational and clinical research in cancer.

38. Vishnugad Project


Why in News?
An independent panel of the World Bank is considering a plea by residents of villages in Uttarakhand to investigate
environmental damage from the under-construction Vishnugad Pipalkoti Hydro Electric Project(VPHEP).
About

• Vishnugad Project is a run-of-river hydroelectric project being constructed on Dhauliganga River in Chamoli District
of Uttarakhand.
• Built by ➔ Tehri Hydropower Development Corporation (THDC), a partially State-owned enterprise.
• Funded by ➔ The project is primarily funded by the World Bank and was sanctioned in 2011.
• Expected completion year ➔ The project is proposed to be completed in June 2023.
• Complaints ➔ Residents of a village (Haat village, Chamoli district, Uttarakhand )have alleged that the muck
threatens a historic temple and construction has affected their livelihood and access to water.

39. Zombie Ice


Why in News?
According to a study, zombie ice from the massive Greenland ice sheet will eventually raise the global sea level by at
least 10 inches (27 centimeters) on its own.

122 | P a g e
About

• Zombie or doomed ice is ice that is still attached to thicker areas of ice but is no longer getting fed by larger glaciers.
That’s because the parent glaciers are getting less replenishing snow.
• Without replenishment, the doomed ice is melting from climate change and will inevitably raise the sea level.

👉 Significance of this study

• The ten inches rise in global sea level predicted by this study is more than twice as much sea level rise as scientists
had previously expected from the melting of Greenland’s ice sheet.
• For instance, the Intergovernmental Panel on Climate Change in its 2021 report projected a range of 2 to 5 inches
(6 to 13 centimeters) for likely sea level rise from Greenland ice melt by the year 2100.

40. India Council for Research on International Economic Relations


Why in News?
India Council for Research on International Economic Relations (ICRIER), in collaboration with India Cellular and
Electronics Association (ICEA) prepared a report titled as ‘Globalise to Localise: Exporting at Scale and Deepening the
Ecosystem are Vital to Higher Domestic Value Addition.
About

• Established in August 1981, ICRIER is a policy-oriented, not-for-profit, economic policy think tank.
• Main focus ➔ to enhance the knowledge content of policy making by undertaking analytical research that is
targeted at informing India’s policy makers and also at improving the interface with the global economy.
• ICRIER conducts thematic research in the following five thrust areas ➔
o Growth, Employment and Macroeconomics (GEM)
o Trade, Investment and External Relations (TIER)
o Agriculture Policy, Sustainability and Innovation (APSI)
o Digital Economy, Start-ups and Innovation (DESI)
o Climate Change, Urbanisation and Sustainability (CCUS)

👉 About ICEA:

• ICEA is the apex industry body for the mobile and electronics industry comprising manufacturers, brand owners,
technology providers, VAS application & solution providers, distributors, and retail chains of mobile handsets and
electronics.
• ICEA is committed to carrying forward its vision of building Indian manufacturing and design in verticals other than
mobile handsets while consolidating the gains made in the mobile handset and components industry.

123 | P a g e
Quick revision (True/False)
1 Nowcasting -> is weather forecasting on a very short term e.g., 2 hrs.

2 INS Vikrant -> Nuclear-powered ballistic missile submarines.

3 Index of Industrial Production -> Published monthly by the Central Statistical


Organization (CSO).

4 International North-South Transport Corridor -> originally decided between


India, China, and Russia in 2000 in St Petersburg.

5 Living Lands Charter -> officially adopted by BRICS countries.

6 Exercise VINBAX 2022 -> bilateral military exercise between India and Vietnam.

7 Seekho aur Kamao Scheme -> Implemented by Ministry of Skill Development and
Entrepreneurship.

8 Loktak lake -> Known for its floating circular swamps, which are called phumdis.

9 Electoral Bonds -> HDFC Bank is authorised to issue and encash these bonds.

10 School Innovation Council -> is an initiative taken by The Ministry of Education’s


Innovation Cell (MIC) and The All-India Council for Technical Education (AICTE).

11 Indian Knowledge Systems -> is an innovative cell under Ministry of Science and
Technology.

12 World Organisation for Animal Health -> Headquarters: Geneva, Switzerland.

13 Exercise Pitch Black -> is a biennial exercise hosted by the Australian Air Force.

14 India Meteorological Department (IMD) -> an agency of the Ministry of Earth


Sciences.

15 Central Waqf Council -> is a non-statutory body.

16 Hambantota Port -> Israel.

17 Diammonium Phosphate -> preferred fertilizer in India because it contains both


Nitrogen and Phosphorus.

124 | P a g e
18 Central Vigilance Commissioner -> Their term is 6 years or 65 years, whichever is
earlier.

19 Bureau of Energy Efficiency(BEE) -> established by Energy Conservation Act 2001.

20 Hasdeo Aranya region -> Kerala.

21 Exercise Yudh Abhyas -> is a military exercise between India and the US.

22 Laser-Guided Anti-Tank Guided Missiles -> Ranges from 1.5 to 5 kilometers.

23 Eklavya Model Residential Schools -> The Ministry of Education in the year 1997-
98 launched the scheme.

24 MP’s privileges on arrests -> member of Parliament cannot be arrested in a civil


case 40 days before the commencement of the session or committee meeting and
40 days thereafter.

25 Ayurveda, Siddha and Unani Drugs Technical Advisory Board -> is a non-statutory
body.

26 Small Satellite Launch Vehicle -> has four stages using solid and liquid propulsion
systems alternately.

27 Ocean Thermal Energy Conversion plant (OTEC) -> established in Kavaratti, the
capital of Lakshadweep.

28 Exercise Vajra Prahar -> India-France Joint Special Forces exercise.

29 Great Barrier Reef -> Located in Atlantic Ocean.

30 India ki Udaan Project -> implemented by Google Arts and Culture, in association
with the Ministry of Culture.

31 Foreigners’ Tribunals -> Powers to constitute tribunals were vested only with the
Centre.

32 Operation Yatri Suraksha -> Launched by Central Reserve Police Force.

33 Quit India Movement -> Slogan ‘Quit India’ was coined by Yusuf Meherally.

34 Bhitarkanika National Park -> Uttarakhand.

35 Vasculitis -> several conditions that cause inflammation of blood vessels.

36 SMILE-75 Initiative -> Launched by Ministry of Women and Child Development.

125 | P a g e
37 Advanced Towed Artillery Gun System -> range of over 45 km.

38 Audio Visual Co-production Treaty -> between India and Sri Lanka.

39 Novel Langya Henipavirus -> is a type of Henipavirus, a category of zoonotic


viruses that can jump from animals to humans.

40 Universal Postal Union -> It is not a United Nations specialized agency.

41 UNMOGIP -> 24 January of 1949 to supervise the ceasefire between India and
China.

42 Digital Lending Norms -> A standardized Key Fact Statement (KFS) must be
provided to the borrower before executing the loan contract.

43 Indian Ocean Dipole (IOD) -> IOD affects the climate of Australia and other
countries that surround the Indian Ocean Basin.

44 PLI Scheme for the Auto Sector -> Launched in 2021 by Ministry of Heavy
Industries.

45 Asiatic Lion -> IUCN Red List: Vulnerable.

46 Chronic Fatigue Syndrome -> causes are still unknown.

47 Exercise Udarashakti -> is a bilateral air force exercise between India and Sri
Lanka.

48 Aurobindo Ghose -> Among his many writings are The Life Divine, The Synthesis
of Yoga and Savitri.

49 Agasthiyarmalai Elephant Reserve -> Kerala.

50 Atal Pension Yojana -> Eligibility- Any Indian citizen in the age group of 20-60
years.

51 International Labour Organization -> not a specialized agency of the UN.

52 Nicaragua -> is the northernmost country in Africa.

53 DigiYatra -> is a Biometric Enabled Seamless Travel experience (BEST) based on


Facial Recognition Technology.

54 Fundamental Duties -> incorporated in Part IV-A of the Constitution by the 44th
Amendment of 1978.

126 | P a g e
55 Rhine river -> rises in two headstreams high in the Swiss Alps.

56 Chitrangudi Bird Sanctuary -> Odisha.

57 Navroz festival -> on the list of UNESCO Intangible Cultural Heritage of Humanity
of India.

58 Super Vasuki -> Is the longest and heaviest freight train ever run by the Railways.

59 Manthan Platform -> The platform is powered by NSEIT.

60 Paalan 1000 Campaign -> Aims to create awareness on environmental issues.

61 Cornea -> Damage to the cornea is one of the leading causes of blindness.

62 STEVE Phenomenon -> Disturbances in the Earth's magnetosphere due to solar


wind.

66 Desert National Park -> The Chinkara or Indian Gazelle (Gazella bennettii) is a
common antelope of this region.

67 Godavari River -> Rises from Trimbakeshwar in the Nashik district of Maharashtra.

68 Bal Aadhar -> Biometrics to establish uniqueness for the Bal Aadhar is not
collected.

69 Tasmanian Tiger -> IUCN Status: Extinct.

70 Kamakhya Temple -> Tamil Nadu.

71 Desert National Park -> The Chinkara or Indian Gazelle (Gazella bennettii) is a
common antelope of this region.

72 Godavari River -> Tributaries: Vamsadhara, lndravati, Pranahita, Pennar.

73 NIDAAN Portal -> developed by the Narcotics Control Bureau.

74 Tasmanian Tiger -> IUCN Status: Critically Endangered.

75 Kamakhya Temple -> Tamil Nadu.

76 Lord Hardinge -> In 1904, he passed the Universities Act (1904) that placed
Calcutta University under government control.

77 Jal Jeevan Mission -> Goa and Dadra & Nagar Haveli and Daman & Diu (D&NH and
D&D) became the first ‘Har Ghar Jal’ certified State and UT.

127 | P a g e
78 Air Quality and Health in Cities report -> Released by WHO.

79 Elements of Mandala in art -> Triangles represent Eight-fold path of Buddism.

80 Grameen Udyami Project -> Funded by National Skill Development Corporation.

81 Paraguay -> bordered by Peru to the south.

82 Facial Recognition Technology -> 1:1 verification of identity used by Law


enforcement agencies such as the Delhi Police usually procure FRT.

83 Tomato Flu -> Bacterial disease.

84 Geographical Indication Tag -> tag is valid for a period of 3 years following which
it can be renewed.

85 National Legal Services Authority -> Attorney-General for India is the Patron-in-
Chief.

86 India's first situational awareness observatory -> located in Hanle near Leh in
Ladakh, India.

87 Acculturation -> concept of acculturation was coined in 1880 by American


geologist John Wesley Powel.

88 Poyang Lake -> Russia's biggest freshwater lake.

89 Garba -> originates from the state of Punjab.

90 Nikshay Poshan Yojna -> aims to support every Tuberculosis (TB) Patient by
providing a Direct Benefit Transfer (DBT).

91 PEN-PLUS Strategy -> Global Guidelines for Early Childhood Education.

92 Sickle cell disease -> is a group of inherited red blood cell disorders.

93 One Nation One Fertiliser Scheme -> The scheme functions under the aegis of
the department of fertilizer, the Ministry of Agriculture and Farmers Welfare.

94 Southern Rice Black-Streaked Dwarf Virus -> transmitted by white-backed plant


hopper(WBPH) in a persistent circulative and propagative manner.

95 Article 143 -> such referral advice by the apex court is binding on the President.

96 Recognition of Prior Learning (RPL) program -> Enrolling the persons in


universities for distance learning programmes.

128 | P a g e
97 Pegasus -> is a tactical surface-to-surface short-range ballistic missile.

98 Lenticular Clouds -> are stationary clouds that form mostly in the troposphere.

99 Convention on International Road Traffic of 1949 -> India is not a signatory.

100 Puga Valley -> Uttarakhand.

129 | P a g e

You might also like